Sie sind auf Seite 1von 102

Psychiatry (adult and child)

1
2
Over the past three months a 54 year-old man has become forgetful, preoccupied, withdrawn, irritable
and disheveled. His physical examination is otherwise normal. The patient has been with his company for
twenty two years and has always been a model employee. The family states that the patient does not
smoke, use alcohol or drugs, has no significant past medical history, and takes no prescription
medications.

Which of the following is the most likely diagnosis?

Multi-infarct dementia

Hypothyroidism

Schizophrenia

Alcoholism

Major depression

Explanation

The correct answer is choice E

While all of the above conditions can result in some or all of the patient’s symptoms, major depression is
the best fit for this patient’s presentation. The diagnosis of major depression requires that the patient must
experience depressed mood most of the day for a period of at least 2 weeks and suffer from 5 or more of
the following symptoms during the same 2-week period: poor appetite or overeating that has led to
significant weight change; insomnia or hypersomnia; poor concentration or difficulty making decisions;
feelings of hopelessness; markedly diminished interest or pleasure in all, or almost all, activities most of
the day, nearly every day; psychomotor agitation or retardation nearly every day; fatigue or loss of energy
nearly every day; feelings of worthlessness or excessive or inappropriate guilt (which may be delusional)
nearly every day (not merely self-reproach or guilt about being sick); recurrent thoughts of death (not just
fear of dying), recurrent suicidal ideation without a specific plan, or a suicide attempt or a specific plan for
committing suicide.

Of note, irritability is a psychomotor hallmark of depression. In addition to the above symptoms, in order
for a person to qualify for a diagnosis of major depression substance abuse must be eliminated as a
possible cause and the symptoms must cause the person either social or vocational difficulties—all of which
are manifest in our example.

Multi-infarct dementia (Choice a) is not a likely diagnosis for this patient because this condition results
from a series of small cerebral arterial infarcts that that occur irregularly over time and cause a step-wise
deterioration of mental and physical function. The relatively rapid onset and progression of our patient’s
symptom complex is not consistent with the slow, progressive course characteristically seen with multi-
infarct dementia. In addition, he has no known risk factors for cerebrovascular disease.

Hypothyroidism (Choice b) is not the best answer for this condition because while hypothyroidism can
result in major depression, hypothyroidism-related depression is usually more insidious in developing and
will likely be accompanied by 1 or more physical exam findings consistent with hypothyroidism, none of
which the patient demonstrates.

Schizophrenia (Choice c) is incorrect because it is extremely unlikely that this condition would develop in a
person of his age (54). Schizophrenia is a chronic disease with an insidious onset that almost always

3
manifests in the 20s and 30s.

Alcoholism (Choice d) makes no sense as a diagnosis for this patient because his family states that he is a
nondrinker, and his “normal physical examination” suggests that he has no stigmata of alcoholism.

While rounding on your patients, a nurse tells you that Mr. Jones, a patient on your medicine service,
"seems suicidal". He is unable to explain more, but urges you to evaluate Mr. Jones immediately. At the
patient's bedside, you do a cursory check to be sure all sharp and dangerous objects are out of reach.

Next, you begin your interview of the patient. Mr. Jones is a married, 66-year-old man with a history of
renal failure secondary to uncontrolled diabetes mellitus type II. He was hospitalized after the amputation
of several toes from foot ulcers. He mentions that he feels like a burden on his wife "and she would be
better off if I was dead".

Which of the following would be the next best step:

Immediately order an SSRI and anxiolytic while continuing to interview the patient to gain
more insight into his suicidal risk
Place the patient on one-to-one watch and review his chart for a more detailed history
Continue with the interview to ascertain his suicide risk level by asking about plan and intent
Immediately place patient on one-to-one watch and continue the interview to ascertain for
suicide risk level
Administer an anxiolytic, place on one-to-one watch and complete interview when patient is no
longer upset

Explanation

The correct answer is choice C

The most appropriate next step is to sensitively steer the interview further to discover his suicide risk. It is
important to ask about plan and intent in order to determine how likely a pending suicide is. After
analyzing the information, you may then decide to place him on one-to-one watch for suicide prevention or
order medications for depression and/or anxiety.

Things to note in the vignette are the patient's age, gender, marital status, and medical condition. These
pieces of information can begin your suicide risk assessment. His gender, male, places him at a higher risk
than a female for suicide completion. Next, his age, over 45, places him at a much higher risk of
completed suicide than those under 45. His marital status is an advantage, as married men tend to be less
likely to commit suicide than single or divorced men. Finally, his medical condition is rather poor with his
history of diabetes and renal failure as well as his recent amputations. All of these pieces of information,
however, are not as important as your impression when asking about plan and intent. A good suicide risk
assessment takes each piece of information into account and uses them to make their best, educated
judgment on the individual's actual risk of attempting suicide.

The first option suggests prescribing an SSRI and anxiolytic (usually a benzodiazepine) before finishing the
interview; this would be a premature step. It is important to complete the interview and review the chart
before prescribing any medication in this situation.

The second option places the patient on one-to-one watch before actually assessing the suicide risk. The
chart will not be able to tell you how realistic the risk of suicide is. That is something that must be done in
person, carefully analyzing body language as well as verbal information. Additionally, the answer choice
suggests leaving the patient after already beginning the interview. This would not only be impolite, but
dangerous considering the last comment he made to you was emotionally charged; a departure by the
physician at this crucial time will erode the patient's trust in the physician and ultimately decrease

4
compliance with treatment.

The third, and correct, choice appropriately delves deeper into the patient's comment to see how serious
and thought-out the comment was. It is a conservative and appropriate measure to take under the
circumstances. Only after doing a careful and thorough assessment, including a suicide risk assessment,
can decisions regarding treatment and plan be made.

The fourth option, like the second option, places the patient on one-to-one watch before thoroughly
assessing the actual level of risk. Therefore, it could be an unnecessary step to take and could easily be
prevented (or confirmed) with a more complete evaluation.

The final choice administers an anxiolytic and places on one-to-one watch without further questioning.
These steps may be unnecessary with only a few more minutes of questioning. Finally, it even suggests
that completing the interview later would be best. This is not polite to the patient, nor is it prudent to
leave after the patient made an emotionally charged statement. Support at this time will gain the trust of
the patient and increase the chances of his honesty regarding suicidal ideation as well as his compliance
with the eventual treatment.

Yesterday you admitted a 27 year-old male suffering from bipolar affective disorder to your service. You
are asked by your senior psychiatry resident to make a presentation at rounds the next morning on the
subject of affective disorders and ways to differentiate between bipolar and unipolar patients.

Each of the following statement about affective disorders is true except:

major depressive illness is more common in females than in males


different responses to lithium are seen in unipolar and bipolar disorder
imipramine is more likely to produce hypomania in bipolar patients than in unipolar patients
patients with bipolar disorder have roughly the same frequency of positive family history as do
patients with unipolar disorder
bipolar affective disorder is far less common than unipolar affective disorder

Explanation

The correct answer is choice D

Affective disorders are those mental illnesses which predominantly affect mood and also have an effect on
thoughts, behaviors and emotions. They include: major depressive disorder (unipolar depression), bipolar
disorder (manic-depressive illness), dysthymia, schizoaffective disorder, schizophreniform disorder and the
anxiety disorders. The rate of a positive family history of affective disorder in patients suffering from
bipolar disorder (80-90%) is much higher than that seen in unipolar disorders (39-69%)

Major depressive illness is more common in females than males (choice A) is correct because it is double
that seen in men.

Different responses are seen to lithium in unipolar and bipolar disorder (choice B) is correct. The mood stabilizer lithium is
one of the drugs of choice in the treatment of bipolar disorder, and receives only rare utilizion in the treatment of unipolar
conditions--most specifically as adjuvant therapy in treatment-resistant depression.

Imipramine is more likely to produce hypomania in bipolar patients than in unipolar patients (choice C) is correct.
Antidepressant therapy must always be approached with caution in bipolar patients because of the risk of
mania induction. This development is rarely seen in unipolar patients. If seen, consideration must be given
to the patient actually having the diagnosis of bipolar disorder.

Bipolar disorder is far less common than unipolar disorder (choice E) is correct because it has a 3-5%
lifetime risk, a figure markedly lower than that seen with seen with unipolar disorder, which carries a 15-

5
20% lifetime risk.

A 62 year old man who was diagnosed with schizophrenia in his early twenties is found wandering around
his halfway house confused and disoriented. He is rushed to a local emergency room, where his serum
sodium concentration is found to be 123 meq/L. his urine sodium concentration is 5 meq/L. The patient
has been treated with risperidone 4 mg a day for the past 3 years with good symptom control. His
roommate reports that the patient makes many trips to the water cooler and often complains of being
thirsty.

What is the most likely cause of this patient’s metabolic unbalance?

renal failure
inappropriate ADH secretion
addison’s disease
psychogenic polydipsia
nephrotic syndrome

Explanation

The correct answer is choice D

The patient’s euvolemic status and hyponatremia suggests that psychogenic polydipsia is the most cause of of
his hyponatremia. Primary polydipsia or psychogenic polydipsia is a special form of polydipsia. It is usually
associated with a patient's increasing fluid intake due to the sensation of having a dry mouth. When the term
"psychogenic polydipsia" is used, it implies that the condition is caused by mental disorders. However, the dry
mouth is often due to phenothiazine medications used in some mental disorders, rather than the underlying
condition

The patient drinks large amounts of water, which raises the pressure of the extracellular medium. As a side
effect, the vasopressin (antidiuretic hormone) level is lowered. The urine produced by these patients will have a
low electrolyte concentration and it will be produced in large quantities (polyuria). If the patient is
institutionalised, close monitoring by staff is necessary to control fluid intake. In extreme episodes, the
patient's kidneys will be unable to deal with the fluid overload, and weight gain will be noted. While
psychogenic polydipsia is usually not seen outside the population of those with serious mental disorders, it may
occasionally be found among others in the absence of psychosis, although there is no extant research to
document this other than anecdotal observations. Such persons typically prefer to possess bottled water that is
ice cold, consume water and other fluids at excessive levels, and may be falsely diagnosed as suffering from
diabetes insipidus, since the chronic ingestion of excessive water can produce symptoms and diagnostic results
that mimic mild diabetes insipidus.

Hyponatremia is an electrolyte disturbance in which the sodium concentration in the serum is lower than
normal. Sodium is the dominant extracellular cation and cannot freely cross the cell membrane. Normal serum
sodium levels are between 135-145 mEq/L. Hyponatremia is defined as a serum level of less than 135 mEq/L
and is considered severe when the serum level is below 125 mEq/L.

Hyponatremia is most often a complication of other medical illnesses in which either fluids rich in sodium are
lost (diarrhea or vomiting) or excess water accumulates in the body at a higher rate than can be excreted (for
example in congestive heart failure, syndrome of inappropriate antidiuretic hormone, SIADH, or polydipsia).
Regarding sodium loss as a cause of hyponatremia, it is important to note that such losses promote
hyponatremia in only an indirect manner. In particular, hyponatremia occurring in association with sodium loss
does not reflect inadequate sodium availability as a result of the losses. Rather, the sodium loss leads to a state
of volume depletion, with volume depletion serving as signal for the release of ADH (anti-diuretic hormone). As
a result of ADH-stimulated water retention, blood sodium becomes diluted and hyponatremia results.

Symptoms of hyponatremia include nausea and vomiting, headache, confusion, lethargy, fatigue, appetite loss,

6
restlessness and irritability, muscle weakness, spasms, or cramps, seizures, and decreased consciousness or
coma. The presence and severity of symptoms are associated with the level of serum sodium, with the lowest
levels of serum sodium associated with the more prominent and serious symptoms.

Neurological symptoms often show for extremely low levels of sodium. When sodium levels in blood become too
low, excess water enters cells and causes the cells to swell. Swelling in the brain is especially dangerous
because the brain is confined by the skull and is unable to expand. Neurological symptoms most often are due
to very low serum sodium levels (usually <115 mEq/L), resulting in intracerebral osmotic fluid shifts and brain
edema. This neurological symptom complex can lead to tentorial herniation with subsequent brain stem
compression and respiratory arrest, resulting in death in the most severe cases.

The imbalance between sodium and water in blood may occur in three primary ways and is based on the
patient’s volume status:

 Hypervolemic hyponatremia, excess water dilutes the sodium concentration, causing low sodium
levels. Hypervolemic hyponatremia is commonly the result of kidney failure, heart failure or liver
failure.
 Euvolemic hyponatremia, normal water levels are combined with low sodium levels. This condition is
commonly due to chronic health conditions, cancer or certain medications.
 Hypovolemic hyponatremia, water and sodium levels are both low. This may occur, for example, when
exercising in the heat without replenishing fluid electrolytes or with marked blood loss.

Renal failure (choice A) is a hypervolemic hyponatremia where excess retained free water water dilutes the
sodium concentration causing low sodium levels. Patient in renal failure would show signs of volume overload
such as ascites or pitting edema.

Inappropriate ADH secretion (choice B) is a paraneoplastic syndrome commonly associated with small cell
carcinoma of the lungs.

Addison’s disease (choice C) or chronic adrenal insufficiency would result in hypovolemic hyponatremia in which
water and sodium levels are both low. Patients would show physical signs such of volume depletion such as
skin tenting and positive orthostatic vital signs.

Nephrotic syndrome (choice D) is a hypervolemic hyponatremia where excess retained free water water dilutes
the sodium concentration causing low sodium levels. Patient with nephrotic syndrome would show signs of
volume overload such as ascites or pitting edema.

7
A 13-year-old boy is referred by his pediatrician to a child and adolescent psychiatrist. The patient reports
having various involuntary, repetitive, stereotyped movements of his arms and neck, which have occurred
every day for the past 5 years. Additionally, for the past 2 years, the patient repetitively and frequently
clears his throat or grunts. The patient’s pediatrician has found no laryngeal pathophysiology to suggest a
medical etiology of the patient’s throat clearing. These behaviors are increasingly interfering with the
patient’s ability to participate in social activities, as he feels embarrassed and ashamed by the behavior.
He has no other symptoms. The patient’s father purportedly exhibited similar symptoms in adolescence.
An adequate therapeutic trial of clonidine has been ineffective in suppressing the patient’s behaviors.

What is the most appropriate treatment for this patient?

Benztropine mesylate
Haloperidol
Reassurance and clinical monitoring
Risperidone and behavioral interventions
ECT

Explanation

8
The correct answer

Tourette's Disorder
over-sensitivity of d
antipsychotic or alp
failed a trial of Clon
the management o
antipsychotics), suc
significantly. Up to

Atypical antipsycho
frequently chosen b
and parkinsonian s
success in the dose
extrapyramidal adv
have limited its use
are commonly used

Behavioral interven
movements that ca
self-monitoring, inc
well as habit revers
together have a syn

Choice A. Benztrop
It would be used to
of antipsychotic use

Choice B. Haloperid
choose atypical ant

Choice C. Reassura
While this is a part
you are competent

Choice E. There ha
Tourette's Disorder

A 27 year old resident complains of increasing anxiety over the past six months. She describes episodes of
sudden onset anxiety, sweatiness, shortness of breath and a fear of losing her mind. These episodes
resolve spontaneously after five to ten minutes, and have occurred both at home and at work. Physical
examination is normal. The most appropriate management is which of the following:

1. paroxetine
2. psychotherapy and possible referral for drug rehabilitation
3. lorazepam
4. cognitive behavioral therapy with gradual desensitization

5. reassurance

9
1,2
1,2,3
1,3,5
1,4,5
All of the above

Explanation

The correct answer is Choice C.

This patient is demonstrating the clinical manifestations of panic disorder. The mainstay of panic disorder
treatment is a combination of serotonergic antidepressants in conjunction with reassuring the patient that
with time the episodes should become less frequent and less severe. Some patients will experience severe
debilitating episodes and may require benzodiazepines such as alprazolam or lorazepam. Efforts should be
made to avoid prescription of such drugs, if possible, because their introduction can lead to addiction and
other related concerns.

Selections 1 and 2 are not correct (Choice A) because, while paroxetine would be an excellent
antidepressant choice for someone suffering from panic disorder, this patient has demonstrated no severe
psychopathology that would merit targeted psychotherapy and has evidenced no substance abuse
concerns.

Selections 1, 2 and 3 (Choice B) are not correct because again, as outlined in the above explanations that
while paroxetine and as-needed lorazepam would be appropriate in the management of panic disorder,
substance abuse and psychotherapy referrals would not be.

Selections 1, 4 and 5 (Choice D) are not correct because, while paroxetine and reassurance would be
advised in this situation, cognitive behavioral therapy is a specific psychotherapy utilized in the
management of some psychological and psychiatric disorders, but not in panic disorder.

All of the above (Choice E) is not correct because we have already established that selections 2 and 4 are
not correct.

A 15-year-old boy is evaluated for depression. Medical causes and substance abuse have been ruled out.
His mother states he has been irritable for almost two years, with poor appetite and difficulty sleeping.
Over the past three weeks, he has been much more irritable. He is eating a lot of junk food and sleeping
excessively, and his school has reported academic problems. He reports that his legs feel like "lead
weights."

Based on this history, which of the following diagnoses is/are likely?

10
Adjustment disorder
Dysthymic disorder
Atypical major depression
Minor depression
B and C
B and D
B, C and D

Explanation

The correct answer is choice E

The diagnostic criteria for dysthymia and major depression are defined in the Diagnostic and Statistical
Manual of Mental Conditions (DSM-IV-TR). Dysthymia, a chronic low-level depressive condition, requires
the patient experience “depressed mood most of the day, for more days than not, as indicated either by
subjective account or observation by others, for at least 2 years” and that during this period he/she suffer
from “at last 2 of the following symptoms: poor appetite or overeating; insomnia or hypersomnia; low
energy or fatigue; low self-esteem; poor concentration or difficulty making decisions; or feelings of
hopelessness”. Irritability is a hallmark of depressive illness in adolescents, overeating (as opposed to
anorexia), and excessive sleeping, or hypersomnia, are characteristics used to differentiate atypical
depression from major depression.

Adjustment disorder (Choice A) is incorrect because the diagnosis of adjustment disorder requires that the
patient develop “emotional or behavioral symptoms in response to an identifiable stressor(s) occurring
within 3 months of the onset of the stressor(s)” and that the “symptoms or behaviors are clinically
significant as evidenced by either of the following: (1) marked distress that is in excess of what would be
expected from exposure to the stressor or (2) significant impairment in social or occupational (academic)
functioning. While our patient is experiencing both distress and impairment, we have not been informed of
an unidentifiable stressor.

We know that dysthymic disorder (Choice B) by itself is correct, but choice E is the preferred answer
because of the related correctness of Choice C.

Minor depression (Choice D) is incorrect because this relatively new and somewhat controversial diagnosis,
defined in the DSM-IV-TR as “characterized by the presence of 2 to 4 depressive symptoms during a 2-
week period, and require(ing) one of these symptoms to be either depressed mood or loss of interest or
pleasure. A history of a variety of disorders, including MDD, excludes an individual from this category”. The
patient in question is eliminated from having this diagnosis because of both the length and number of his
symptoms and the fact that he fulfills criteria for both dysthymia and atypical major depression.

A 40 year-old man has been standing immobile for four days. One of his arms is stretched upward, the
other is wrapped around his neck. The patient does not appear aware of his surroundings and actively
resists any attempt to make him change position.

What is the next step in the management of this patient?

Explain that refusal to cooperate with the nurses limits the amount of help you can provide
him, ultimately delaying his return to a health state of mind.
Gently continue to loosen up his joints and muscles using passive resistance exercises.
Ignore the behavior because this type of personality disorder worsens with increased
attention, albeit negative attention.
Administer 10 mg of intramuscular Ziprasidone.
Administer 2 mg of intravenous Lorazepam.

11
Explanation

The correct answer is choice E.

Lorazepam is the first-line agent of choice for catatonia regardless of the cause. you are told that this
patient held this position for 4 days, making malingering impossible (try holding that position for 10
minutes!). You do not have to learn medication doses for any of the licensing exams.

Other agents include amantadine 100 mg po bid, bromocriptine 2.5-5 mg po bid, memantine 5-10 mg po
bid, and topimarate 100 mg po bid. Electroconvulsive Therapy should be used expeditiously if catatonia is
not responding to medications or if malignant catatonia is present.

In this case, this patient most likely has schizophrenia with catatonic posturing and extreme negativism. A
brief discussion of similar conditions follows. Negativism is motiveless resistance toa ll attempts to be
moved or to all instructions. Catatonic rigidity is the voluntary assumption of a rigid posture, held against
all attempts to be moved. Catatonic posturing is the voluntarily assumption of an inappropriate or bizarre
posture, generally maintained for long periods. Muscle rigidity describes a state in which the muscles
remain immovable, as seen in schizophrenia.

It is worth remembering that catatonia can also exhibit excessive motor activity, such as constant unrest,
screaming, taking off clothes, or running down the hallway.

A. Though posturing is defined as purposeful, the underlying pathology required to make a man hold a
posture for 4 days is profound. Reasoning with him will not help much, if at all, though it is good practice
to explain what you are doing to patients even when you don't think they can understand you.

B. The source of the rigidity originate from upper motor neurons. Massage and exercise may feel soothing
as the patient recovers their senses, but this is not the next step in the management.

C. Clearly this is not a personality disorder presentation. You would have to get the schizophrenia
medically under control to assess for the presence of a personality disorder. This behavior is so painful and
bizarre that no one aware of their senses could do this voluntarily.

D. This make sense because it is an atypical antipsychotic, but the question asked about the next step in
the management. Clearly this patient needs to restart or start antipsychotic medication, but his muscular
rigidity is the predominating feature that needs relief.

You are assigned the management of a 27 year-old African-American male patient by your inpatient
psychiatric attending. The patient was brought to the ED by the police who picked him up when he was
frightening people in an area mall with his odd behavior and strange statements. You diagnose him with
schizophrenia, paranoid type, and elect to institute begin treatment with haloperidol (Haldol).

Common side effects of haloperidol include each of the following except:

1. stiffness
2. akathesia
3. dry mouth
4. tremors

5. hypotension

1 and 3
2, 3, 4
1, 4
3, 5
3, 4, and 5

12
All of the above are common side effects

Explanation

The correct answer is Choice F

Haloperidol’s mechanism of action is via dopamine blockade. This drug has a fairly significant side effect
profile that limits its utility. All of the side effects listed above are seen with some frequency with Haldol
administration. Haldol’s interference with dopamine can lead to cholinergic imbalance, resulting in
extrapyramidal symptoms and xerostomia (dry mouth)

Choosing stiffness and dry mouth (choice a) is incorrect because both stiffness and dry mouth are common
side effects of Haldol. Reduction of available dopamine results in muscular stiffness and rigidity and the
drug's anticholinergic effects often lead to dry mouth.

Choosing akathisia, dry mouth and tremors (choice b) is wrong because akathisia is a side effect that is
commonly seen effect with antipsychotic treatment, including Haldol. Akathisia is a movement disorder
characterized by a feeling of inner restlessness and a compelling need to be in constant motion. People
with akathisia are unable to sit or keep still, complain of restlessness, fidget, rock from foot to foot, and
pace. We have already established that xerostomia is one of Haldol's more side effects.

In addition, tremor is a common issue with patients requiring Haldol therapy. Tremor occurs due to a
haloperidol-induced dopamine deficient state that results in a condition known as pseudoparkinsonism.
This extrapyramidal side effect is usually managed by co-administration of benztropine (Cogentin). One
concern of Cogentin, however, is that it can be additive to xerostomia concerns.

Again, xerostomia is a common side effect of Haldol. Orthostatic hypotension is also a common concern
upon commencement of treatment with Haldol; accordingly, choice d is incorrect. Great care must be taken
to begin treatment at a low dose and to advance the dose slowly to prevent dizziness and falling episodes
that can result in patient injury. This concern often gradually resolves with continued treatment.

A 24-year old male, college sophomore is brought to your office on the insistence of his mother. While he
only engages you in conversation when asked direct questions, his mother is happy to divulge what she
feels is alcoholism in her son. After patiently listening, you invite her to wait outside while you perform an
physical examination.

With the mother out of the room, you do a thorough history and physical exam and find several key
pieces of information:

 he has been using cocaine, alcohol, and amphetamines, all more than he intends to
 he has had several unsuccessful attempts to cut down or stop
 he has been spending hours every day dedicated to obtaining or using the substances
 he has continued his drug and alcohol use despite awareness of harm to his health and his
mother's fears
 it is taking more and more drugs and alcohol to make him feel "high"
 neglect of responsibilities

You diagnose Substance Dependence.

Which of these other behaviors below are included in the criteria for the diagnosis of Substance Abuse?

13
Neglect of responsibilities.

Legal problems.

Conflict with loved ones.


Jeapordizing personal health or safety.

All of the above.

Explanation

The correct answer is Choice E.

All drug and alcohol use is associated with risk. From the very first use of alcohol or drugs, devestating
consequences may occur. In fact, negative consequences are used to partially define Substance Abuse.

Choieces A-D. The diagnostic criteria for Substance Abuse are manifestation one of the following over a 12-
month period:

 Neglect of responsibilities
 Legal problems
 Conflict with loved ones
 Jeapordizing personal health or safety

Notice that neglect of responsibilities is also seen in the criteria for Substance Dependence and Substance
Abuse. The key word to help you distinguish these distinct diagnoses is "Dependence". Whether it is a
physiologic or psychologic dependence, the implications are profound.

Outpatient psychoeducation, cognitive behavioral therapy, and cognitive processing therapy may better
help patients with Substance Abuse, but more radical interventions are often required for those with
Substance Dependence. These interventions may include commitment for evaluation, a detoxification
program, and a subsance dependence rehabilitation program lasting weeks to months.

A 32-year-old woman presents to the primary care physician with the chief complaint of chest pain for 3
months. She describes the pain as diffuse and points to her sternum and upper stomach when describing
the pain. She describes it as a squeezing, tight feeling that does not radiate. She cannot relate the pain
to any activity or time of day, but does admit to feeling "stressed more at work" lately. After a complete
cardiac and gastrointestinal work-up, you decide that the source of her pain is anxiety. She requests
alprazolam, since she tried her friend's and it worked well for her.

Which of the following treatment plans would BEST relieve her symptoms:

14
Alprazolam 0.5 mg daily, see back in two weeks, increase if symptoms still present
Alprazolam 0.5 mg daily, see back in two weeks, switch to SSRI if symptoms still present
Paroxetine 10 mg daily, see back in two weeks, increase to 20 mg daily if symptoms still
present
Paroxetine 10 mg daily, see back in two weeks, add Alprazolam 0.5 mg daily if symptoms still
present
Diazepam 10 mg daily, see back in two weeks, add SSRI if symptoms still present

Explanation

The correct answer is choice C

This question is testing your understanding of the correct use of benzodiazepines in the treatment of
generalized anxiety, as this woman seems to have. Although benzodiazepines work extremely well for the
treatment of anxiety and start working immediately, they can be addicting and are often sold for money or
"loaned" to friends inappropriately. Therefore, it is important to monitor their use carefully. Additionally,
they often are not effective for long-term, but daily-use of SSRIs are. For these reasons, it is best to not
consider a benzodiazepine as a first-line treatment for generalized anxiety. Of the answer choices, four of
the five utilized an SSRI in some way. However, two used SSRIs as second-line, making them incorrect.
Narrowing down the options to the two with SSRIs as first-line, we are left with the following two answer
choices:

Choice C. Paroxetine 10 mg daily, see back in two weeks, increase to 20 mg daily if symptoms still present

Choice D. Paroxetine 10 mg daily, see back in two weeks, add Alprazolam 0.5 mg daily if symptoms still
present

While both of these are technically acceptable, 10 mg is a very low dose of Paroxetine (Paxil), making the
choice that increases the dosage before adding a benzodiazepine the correct answer. It may take a dosage
as high as 40-50 mg of Paxil until the anxiety symptoms are relieved.

Please note that other SSRIs are also successful long-term treatments for anxiety, Paxil was used as an
example only.

A 36-year-old patient with schizophrenia presents to the emergency psychiatric hospital. He was brought
in by the police who found him in a restaurant standing on the table, proclaiming conspiracies regarding
the government. He claims that the government has placed chips into everyone's brains while they were
sleeping and can read all of our thoughts. The patient is clearly agitated and hallucinating.

Based on the most likely acute diagnosis, an elevation in which of the following neurotransmitters could be
causative:

Glutamate
Serotonin
Norepinephrine
Dopamine
Melatonin

Explanation

15
The correct answer is choice D

The most likely acute diagnosis in the schizophrenic patient above is a form of psychosis. The specific type
is not ascertainable due to lack of historical information regarding the patient's schizophrenia. However,
details are not needed to answer the question. Of the neurotransmitters listed, only two have been
implicated in the pathophysiology of schizophrenia: glutamate and dopamine. Of these two, dopamine is
the one that, when elevated, can cause hallucinations, delusions, and the overall loss of reality. While
glutamate also plays a role in psychosis, it seems to cause more problems when there is a reduction in its
transmission at the NMDA receptor.

Glutamate, as stated above, would more likely have problems with transmission as opposed to actual
levels when postulated to be involved in schizophrenia and psychosis.

Serotonin is usually associated with mood disorders. When decreased, depression can result.

Norepinephrine has been shown to be involved in depression and perhaps schizophrenia, however, the
results of some newer studies are still pending, making dopamine the only clear choice for this question.
This neurotransmitter is usually associated with energy and alertness.

Melatonin (which is actually a hormone, not a neurotransmitter) is associated with the sleep-wake cycle
and, when elevated, will cause drowsiness. Therefore, it can often be artificially supplemented to aid in the
treatment of insomnia.

You are an internal medicine resident who has chosen to take an elective inpatient psychiatry rotation
because you are considering working as a part-time medical consultant to an inpatient facility as a
component of your private practice. Lo and behold, both the senior psychiatry resident and the psychiatry
attending are home sick on the same day when you get paged to the emergency department to evaluate a
patient the emergency physician believes is a bipolar in manic phase who merits inpatient admission.

Which of the following classes of medication would be the most useful in the management of an acute
manic episode?

1. neuroleptics
2. anticonvulsants
3. lithium

4. tricyclic antidepressants

1,2,3 only
1,3 only
2,4 only
4 only
all are correct

Explanation

The correct answer is choice A

Manic-depressive illness, also known as bipolar affective disorder, is a condition whose sufferers alternate
between episodes of depression (“down” periods) and mania (“up” periods). Mania is defined as a distinct
period of abnormally and persistently elevated, expansive, or irritable mood, lasting at least 1 week (or
any duration if hospitalization is necessary) and that during this period of mood disturbance, 3 or more of
the following symptoms have persisted (4 if the mood is only irritable) and have been present to a

16
significant degree: inflated self-esteem or grandiosity; decreased need for sleep (ex: feels rested after only
3 hours of sleep); more talkative than usual or pressure to keep talking; insomnia or hypersomnia nearly
every day; psychomotor agitation or retardation nearly every day (observable by others, not merely
subjective feelings of restlessness or being slowed down); flight of ideas or subjective experience that
thoughts are racing; distractibility; increase in goal-directed activity (either socially, at work or school, or
sexually) or psychomotor agitation; or excessive involvement in pleasurable activities that have a high
potential for painful consequences (e.g., engaging in unrestrained buying sprees, sexual indiscretions, or
foolish business investments).

The goal of mania treatment is to return the patient to his or her baseline mental status. Most patients
suffering from a severe manic episodes demonstrate markedly impaired thinking, delusional thinking that
may be bizarre, and may be a danger to themselves or others so will usually require inpatient
management on a locked ward. Medical management options include neuroleptics (another term for
antipsychotics that is no longer in vogue) to help reduce agitation, psychosis and to promote sleep; lithium
(Lithobid, Eskalith) as a mood stabilizer—a product demonstrated to help reduce time to mania resolution
in combination with antipsychotics; and anticonvulsants--products originally designed for the management
of epilepsy that were subsequently discovered to be effective in the managementn of manic-depressive
illness--such as valproic acid (Depakote) or lamotrigine (Lamictal), which are used as mood stabilization
alternatives to lithium for patients who either cannot take lithium due to renal concerns, a history of poor
therapeutic response to lithium, or a history of repeated lithium overdose. Two notes on the answer to this
question: (1) most bipolars require only one mood stabilizer; and (2) while lithium has been the gold
standard mood stabilizer for bipolar disorder for 60 years, it has fallen somewhat out of favor because of
its renal and toxicity concerns, thus leading to increased prescribing of the anticonvulsants for this issue.

Anticonvulsants alone (Choice b) is not the correct answer because both antipsychotics and lithium play an
important role in the management of acute mania (see discussion above).

The combination of neuroleptics and lithium (Choice c) is not the correct answer because anticonvulsants
play a role in the management of acute mania (see discussion above).

Tricyclic antidepressants (Choice d) is incorrect because the last thing a person suffering from mania needs
is a medication that has the potential to raise levels of the neurotransmitters (dopamine, norepinephrine,
epinephrine) whose elevation has resulted in their current manic state (psychiatric equivalent of pouring
gasoline on a raging fire). Tricyclic antidepressants play a very minimal role in the management of persons
suffering from manic-depressive illness because the combination of the impulsiveness that many bipolars
display during manic episodes accompanied by the cardiotoxicity concerns seen with tricyclics can lead to
tragic consequences.

You are summoned to appear as an expert witness in a case involving a previous patient of yours. She is a
37 year-old woman being charged with forgery and has been out of state living under a false identity for
the past nine months.

The facts in the case are that up until one year ago she was living nearby, taught at a local elementary
school, and was involved in the lives of her husband and children. After a tragic vehicle accident, her
entire family was killed and her friends have provided depositions that state she shut down emotionally
after that.

Upon interviewing her, you note what appears to be profound confusion to the point of despair. She insists
that has has not forged any identification documents, despite contrary fingerprint evidence and photo
albums from her presumed parents. She declares that she must have a twin that she wasn't aware of.

Which of the following are true statements regarding dissociative fugue:

17
Rare
Characterized by awareness of the loss of memory
Characterized by behavior that appears extraordinary
to others
Usually long-lasting
All of the above

Explanation

The correct answer is choice A.

There is a prevalence rate of 0.2% in the general population. Like amnesia, dissosiative fugue occurs most
often during wartime, after natural disasters, and as a result of crises with intense internal conflicts.
Although heavy alcohol abuse may predispose persons to dissociative fugue, the cause of the disorder is
thought to be basically psychological. The essential motivating factor seems to be a desire to withdraw
from emotionally painful experiences. Patients with mood disorders and certain personality disorders (i.e.,
borderline, histrionic, and schizoid) are predisposed to develop dissociative fugue [perhaps they're on a
fugue from somewhere else!]. After the person has regained their original identity, they are amnestic to
the fugue itself.

According to the DSM-IV-TR, the predominant disturbance is sudden, unexpected travel away from home
or one's customary place of work, with inability to recall one's past. Confusion about personal identity and
the partial or complete assumption of a new identity are also required to make the diagnosis.

B. Persons with dissociative fugue are not clear about, or are in total unawareness of, their past lives.

C. Persons with dissociative fugue do not seem to others to be behaving in extraordinary ways. They
typically lead quiet, reclusive existences, work at simple occupations, live modestly, and generally avoid
attention.

D. While most episodes of dissociative fugue are measured in days, chronic fugues have been described.
They are usually short-term.

A 64 year old woman presents with lower back pain which at CT scan followed by bone densitometry
testing reveals is the result of tiny compression fractures resulting from osteoporosis. The woman is 12
years post-menopausal and never was given estrogen replacement therapy. Her history is remarkable for
coronary artery disease diagnosed three years ago which treated with atenolol and atorvastatin. However,
since she has noticed an increased tenancy to bruise during the last year, a friend of hers suggested that
all of her symptoms may be related to a nutritional deficiency. Wondering if this may be the case, you
order laboratory tests and find that her hemoglobin is 13.0 g/dL (normal 12.0-15.2), hematocrit is 39.9
percent (normal 37-46), MCV is 94 fL (normal 82-102), platelet count is 333,000/microliter (normal
140,000-450,000), and WBC count is 5,930/microliter (normal 4,100-10,500). PT and PTT both are
prolonged, though bleeding time, factor VIII, and von Willebrand factors all are normal.

Most likely, the patient's condition is related to which of the following?

calcium deficiency
vitamin K deficiency
vitamin D deficiency
von Willebrand disease
disseminated intravascular coagulation

18
Explanation

The correct answer is choice B.

Vitamin K actually is a group of factors that are required for the synthesis of various proteins, most of
which are involved in coagulation. It also plays a role in bone growth and bone density maintenance. Thus,
symptoms and signs of deficiency manifest as bleeding disorders and osteoporosis. Additionally, low levels
of vitamin K2 (menaquinone) are associated with coronary artery disease.

Since vitamin K is required for synthesis of clotting factor VII, deficiency of vitamin K results in an
increased prothrombin time (PT). While partial thromboplastin time (PTT) may be normal in mild cases of
vitamin K deficiency, as other factors drop while the condition progresses, the PTT also will increase.
Bruising, petechiae fit a scenario of vitamin K deficiency, which can result from inadequate absorption of
the vitamin, as may occur in conditions such as celiac disease and Crohn disease. Thus, workup for
gastrointestinal illness may be warranted, though if the patient has no gastrointestinal complaints the
problem may be the result of insufficient dietary intake in which case her diet would require evaluation.

Calcium deficiency (choice A) and vitamin D deficiency (choice C) certainly increase the risk of
osteoporosis, as does being post menopausal. While some studies suggest that vitamin D may be useful in
preventing coronary artery disease, the laboratory results given in the question stem point to vitamin K
deficiency, though since both are fat-soluble vitamins both deficiencies may be present together.

Affecting an estimated one percent of the population, von Willebrand disease (choice D) is the most
common hereditary inherited bleeding disorder. It is named for Erik Adolf von Willebrand, who described
the condition in 1926 as a congenital disorder featuring a tendency to bruising, nosebleeds, and (in the
case of females) menorrhagia, very heavy menstrual bleeding and a prolonged period. It results from a
deficiency or abnormality of von Willebrand factor (vWF), a glycoprotein secreted by capillary endothelial
cells that acts as a carrier for clotting factor VIII (FVIII). This protects FVIII from degradation, and when
vWF is absent or functions poorly, the bleeding time, which normally is no longer than 9.5 minutes, is
prolonged, which is not so in this case. wWF either is below normal, or its activity is reduced.

Disseminated intravascular coagulation (DIC, choice E) is characterized by an activation of the coagulation


cascade. This results in thrombosis and hemorrhagic manifestations. DIC affects both males and females
and can be mild or acute. It can result from trauma, sepsis, malignancy, or various vascular disorders,
snake and insect venom, and in associated with preeclampsia. PT and PTT may be prolonged, but platelet
counts drop.

A 32 year old engineer has been uncharacteristically active for several weeks. He spends most of his time
at work and gets little sleep. He has told another engineer that he is involved “in a research project that
will earn me a Nobel prize”. Expensive research equipment keeps arriving at his office. The engineer is
irritable, and it is hard to hold his attention. A classmate from graduate school recalls that the patient
behaved in a similar manner twice during stressful periods at school.

Long-term drug therapy for this patient would likely include which of the following:

haloperidol
valproic acid
clozapine
ascorbic acid
chlordiazepoxide

Explanation

The correct answer is choice B

The patient in our example appears to be in the throes of a manic episode. Manic-depressive illness, also

19
known as bipolar affective disorder, is a condition whose sufferers alternate between episodes of
depression (“down” periods) and mania (“up” periods). Mania is defined as a distinct period of abnormally
and persistently elevated, expansive, or irritable mood, lasting at least 1 week (or any duration if
hospitalization is necessary) and that during this period of mood disturbance, 3 or more of the following
symptoms have persisted (4 if the mood is only irritable) and have been present to a significant degree:
inflated self-esteem or grandiosity; decreased need for sleep (ex: feels rested after only 3 hours of
sleep); more talkative than usual or pressure to keep talking; insomnia or hypersomnia nearly every day;
psychomotor agitation or retardation nearly every day (observable by others, not merely subjective
feelings of restlessness or being slowed down); flight of ideas or subjective experience that thoughts are
racing; distractibility; increase in goal-directed activity (either socially, at work or school, or sexually) or
psychomotor agitation; or excessive involvement in pleasurable activities that have a high potential for
painful consequences (e.g., engaging in unrestrained buying sprees, sexual indiscretions, or foolish
business investments).

Mood stabilizers are the primary component in the chronic management of manic-depressive illness.
Examples of such products include the correct answer to this question: valproic acid (Depakote), lithium
(Lithobid, Eskalith), carbamazepine, (Tegretol), and newer agents such as quetiapine (Seroquel), and
olanzapine (Zyprexa). Valproic acid, carbamazepine, and lithium all require close dosage monitoring via
serum levels in order to both maximize treatment efficacy and prevent toxicity and side effect concerns.
In addition, the use of both Depakote and Tegretol is discouraged in patients with existing liver disease
due to these drugs’ hepatic concerns, and lithium use should be avoided in patients with chronic renal
disease due to this drug’s exclusive renal metabolism.

Haloperidol (Choice A), brand name Haldol, is an incorrect answer to this question because the question
is seeking a long-term treatment strategy for preventing mania development. Haldol is incorrect because
this potent antipsychotic drug is used in acute mania treatment to reduce the symptoms of mania once it
has already developed.

Clozapine, (Choice C), brand name Clozaril, is incorrect because it is an antipsychotic that receives the
majority of its use in treatment-resistant chronic psychotic patients, the majority of which suffer from
schizophrenia. A unique side effect concern that reduces clozaril prescribing is its association with
possible bone marrow suppression; accordingly, patients who take Clozaril require regular blood count
monitoring to detect development of this potentially serious side effect.

Ascorbic acid (Choice D) is the scientific name for Vitamin C. Inadequate intake of ascorbic acid results in
scurvy, which is the main therapeutic utility for this compound. It has no demonstrated role in the
management of manic-depressive illness so is cannot be the correct answer to this question.

Chlordiazepoxide (librium) (Choice E) is an incorrect answer because this is a member of the


benzodiazepine class and is used primarily as an anxiolytic, but is also utilized in the management of
alcohol withdrawal syndrome, insomnia, acute agitation, and other psychiatric concerns. Other
benzodiazepine class members include lorazepam (Ativan), diazepam (valium), clonazepam (Klonopin),
and alprazolam (Xanax).

A 40-year-old white married woman comes to the clinic with complaints of vague abdominal pain of
three months' duration and the certainty that she has cancer. She has been referred to the clinic after
exhaustive medical examinations, the results of which have always been within normal limits. Despite
numerous tests and hospitalizations, she continues to believe that she has cancer, but "the doctors just
haven't found it yet."

Over the past three months, she has experienced early morning awakening and loss of appetite. She has
lost 12 pounds, which she attributes to the effects of cancer. She is unable to find even momentary
pleasure in anything and has been unable to do her housework. She thinks her family would be better off
without her. She has a sad, fixed affect. Her speech is monotonous and slow. Her sentences often begin
after long, sighing expirations. Tears come to her eyes when she begins to talk about the fact that her
youngest child left for college three months ago.

Previously she had been well. She denies a previous history of similar symptoms and has received no prior
medical or psychiatric help. Although regarded by others as unduly serious, formal, and perfectionistic,

20
she takes pride in the way she is "I guess I was a workaholic, but that's the way I am."

What of the following BEST describes the treatment of choice?

Start her on a Selective Serotonin Reuptake Inhibitor and begin a short course of Cognitive
Behavioral Therapy for relief of major depression.
Schedule weekly visits so that you can reassure her that she will get better, and avoid using the
words "Somatization Disorder" because patients often get the feeling that their problem is "all in
their head".
Encourage group therapy for her to hear the stories of other parents whose children have moved
off to college.
Computed tomographic study with intravenous contrast to cause opacity of the cancer in the
head of her pancreas.
Use exposure therapy to treat her post-traumatic stress disorder in an effort to impart alternate
interpretations of what her child's absense means in her life.

Explanation

The correct answer is choice C.

This patient has Adjustment Disorder and the treatment of choice is psychotherapy, individual or group.
This question asks about treatment and requires you to make the diagnosis yourself. Practice doing that
before you look at the answer choices.

Adustment Disorder is the development of emotional or behavioral symptoms in response to an identifiable


stressor(s) occurring within 3 months of the onset of the stressor. Notice that the question could also be
asked, "how could you confirm the diagnosis?". In that case, the answer would be to have her child move
back home, with the symptoms resolving within 6 months.

The comment about her being serious, formal, and perfectionistic fits a rigid personality, which lends itself
to not adapting easily to changing circumstances.

A. Using an SSRI and CBT is a good idea because CBT is a form of psychotherapy and medications are
symptom-based, regardless of the cause. It is the diagnosis of Major Depression that is the problem. Had
there been no mention of an identifiable stressor, this would have been the correct answer.

B. Somatization is the development of physical symptoms as a result of a psychological problem. This


occurs requently even in a health population. Somatization Disorder is a different category entirely. This
diagnosis requires a history beginning before age 30, lasting several years, and must include four pain
symptoms, two gastrointestinal symptoms, one sexual symptom, and one pseudoneurological symptom.

D. When you are told that a patient has had repeated medical evaluations that yielded no diagnosis, you
must trust that they didn't miss anything. The question stem clearly indicates that she is medically healthy.

E. The literature supports exposure therapy for PTSD, but rare indeed is the psychotherapist that will
actually put a patient through the experience. There is no evidence of PTSD in this question stem.

A 27 year-old African-American female returns for follow-up of major depressive disorder. She has been
on escitalopram 10mg daily for 2 months. She reports some improvement in depression symptoms, more
interest in activities, better sleep, etc., but she still has feelings of hopelessness several times per week
and poor appetite. She reports no adverse side effects from the escitalopram. She denies suicidal or
homicidal ideation. Past medical history is remarkable for seizure disorder, and the patient

21
takes phenytoin 300mg QHS, along with oral contraceptives daily.

Family history positive for a mother with depression who was untreated and committed suicide when the
patient was 20 years old. Her laboratory work up reveals normal chemistries, CBC, and TSH. Her vital
signs today are unremarkable except for a 5 pound weight loss since the last visit. Physical exam reveals
a young female in no acute distress with mildly flat affect. Physical exam is otherwise normal.

All of the following are appropriate next steps in treatment EXCEPT:

Increase escitalopram dosage to 20mg po daily.


Supplement with psychotherapy.
Add aripiprazole 5mg po daily.
Change to bupropion XL 150mg po daily.
Change to sertraline 50mg po daily.

Explanation

The correct answer is choice D.

Bupropion lowers the seizure threshold and is contraindicated in patients with a seizure disorder history.
Otherwise, this would be an acceptable treatment option.

Treatment resistant depression occurs when a patient does not respond fully to an antidepressant.
Clinicians should rule out other causes of incomplete response, such as underlying medical conditions (for
example, hypothyroidism), noncompliance, or incorrect diagnosis (for example, bipolar disorder instead
of major depression). Treatment resistant depression may be managed in a variety of ways, including
trying the medication for a longer period of time. Some patients take longer to respond to
antidepressants than others.

Providers may also increase the medication dosage (A). If the patient is not having adverse side effects
from the medication, a higher dosage may improve the response.

Providers may also augment with psychotherapy (B), which has been shown to improve outcomes in
combination with medication more than either treatment alone.

Supplementing with an additional medication (C) is another option, and aripiprazole has an indication for
this. One can utilize a variety of drug classes for augmentation, including anti-anxiety medications, anti-
seizure medications, mood stabilizers, etc. More than one antidepressant may be used together, also,
provided that the prescriber is aware of the risk of serotonin syndrome (for example, an SSRI and an
SNRI may be used together).

Providers may change to a different antidepressant, also, either in the same class or a different class.
Electroconvulsive therapy can also be used in treatment resistant depression. Although it may have side
effects such as confusion or amnesia, it can at times provide dramatic improvement in patients not
obtaining relief with medications.

A 5-year-old boy presents with his mother to the pediatrician. He has a mental age of 3, but no other
known medical or mental disorders. She reports that nearly every day for the past 3 months her son has
been found eating sand from the sand box in his back yard. When asked why he does this, he replies that
after eating his usual meals his tummy hurts. In addition, at bed time, he appears to be chewing and
swallowing even when he hasn't been near the kitchen or given anything to eat.

What is the most likely diagnosis?

22
Rumination disorder.

Pica of Infancy and Childhood.

Feeding Disorder of Infancy and Childhood.

General medical condition.

B and D.

Explanation

The correct answer is Choice E.

This boy's complaint about his "tummy" hurting after he eats should not be ignored, especially in the
mentally retarded population. He needs to be medically investigated for gastrointestinal disorders, such as
gastroesophageal reflux. This would also explain his rumination at night time.

The concurrent diagnosis of Pica of Infancy and Childhood is simpler to explain and understand. In Pica,
there is persistent or recurrent eating of nonnutritive substances, at least twice per week for at least one
month. The child must not exhibit other mental or behavioral problems in the ICD-10 classification (other
than mental retardation). The child's chronological and mental age must be at least 2 years and the eating
behavior can not be part of a culturally sanctioned practice.

Choice A. The boy in this question appears to be ruminating at night, but Rumination Disorder can not be
due to an associated gastrointestinal or other general medical condition, such as esophageal reflux.
Therefore, that has to be ruled out in this boy because he is complaining about pain after eating.
Rumination involves regurgitation and rechewing of food for a period of at least one month following a
period of normal functioning. Also, the disorder does not occur exclusively during the course of anorexia
nervosa or bulemia nervosa.

Choice C. Feeding Disorder of Infancy or Early Childhood must begin before age 6, which this boy is.
However, the feeding disturbance is manifested by persistent failure to eat adequately with significant
failure to gain weight or significant loss of weight over one month, which this boy does not have mentioned
in his history. Interestingly, feeding disorder can not be due to an associated gastrointestinal or other
general medical condition, such as esophageal reflux. Therefore, the parents should be educated on
monitoring his weight just in case this becomes a problem and reflux is subsequently ruled out.

A 40-year old man is brought to your emergency department with a history of regular and heavy use of
alcohol for ten years and morning drinking for one year. His last alcohol intake was three days ago. There
is no history of head injury or seizures. On examination, there is no icterus, signs of hepatic
encephalopathy or focal neurological findings. His family reports he has been acting very odd and
confused during the past few days. With you he demonstrates coarse tremor, visual hallucinations and
disorientation to time.

Which of the following is the preferred medication to be prescribed for such a patient?

Doxepine
Haloperidol
Metoprolol
Lorazepam

23
Diazepam

Explanation

The correct answer is choice D

The patient in this example is experiencing alcohol withdrawal syndrome and is at risk for developing
delirium tremens. The symptoms of alcohol withdrawal syndrome include tremor, nausea, abdominal
discomfort, vomiting, anxiety, irritability, and moodiness. Untreated alcohol withdrawal syndrome has
potential to lead to delirium tremens (The DTs), which carry significant morbidity and mortality risks.
Drugs in the benzodiazepine class, such as lorazepam (Ativan) and diazepam (Valium) have been used
effectively for many years in the treatment of alcohol withdrawal syndrome. Aggressive treatment of
alcohol withdrawal syndrome reduces the likelihood of its degenerating into delirium tremens.

Most alcohol withdrawal seizures are self-terminating; however, if prolonged, they are usually quickly
terminated with benzodiazepines (eg, diazepam, lorazepam). Lorazepam is preferred because it has a long
redistribution time that enables it to have prolonged effectiveness, protecting the patient from recurrent
seizures. Lorazepam is less dependent on hepatic metabolism than other benzodiazepines, and hepatic
function may be impaired in chronic alcoholics.

Doxepin (Choice A) is a psychotropic agent with tricyclic antidepressant and anxiolytic properties and has
an absolute contraindication in acute intoxication with alcohol, sedatives, analgesics and other
psychoactive drugs and as well, acute delirium tremens.

Neuroleptics like haldol (Choice B) are not used as primary agents because studies have demonstrated the
superior efficacy of sedative-hypnotics in reducing duration of alcohol withdrawal syndrome and associated
mortality. In a severely agitated patient, neuroleptics such as haloperidol 5 mg IV or IM may be added to
sedative-hypnotic agents as an adjunctive therapy and repeated with caution in 30-60 minutes if needed to
control agitation. Caution must be taken because haloperidol has been known to decrease the seizure
threshold as well as prolong the QT interval.

The use of beta-blockers (Choice C) to diminish tachycardia, hypertension, and perhaps anxiety has been
described and are occasionally useful; however, their effects mask the warning signs of autonomic
hyperactivity if the patient develops DT.

A 9-year-old girl presents for routine follow-up with her child psychiatrist. She has been treated for
Obsessive-Compulsive Disorder with a combination of fluoxetine (40 mg/day) and risperidone (1.5
mg/day) for the past 3 years. Although she has had a moderate improvement in her OCD symptoms with
this medication regimen, the patient’s mother has recently observed various involuntary, repetitive
movements of the patient’s face, arms, and neck. The psychiatrist increases the dose of risperidone to 3
mg per day. At follow-up 4 weeks later, the patient’s mother reports that the movements have increased
in frequency and severity.

What is the most likely diagnosis of the patient’s movement symptoms?

Tardive dyskinesia
Withdrawal dyskinesia
Tardive Tourrette's Disorder
Acute dystonia
Tardive parkinsonism

Explanation

24
The correct answer is Choice A.

Tardive dyskinesia is a delayed effect of antipsychotics and rarely begins until after 6 months of
treatment. It is thought to be caused by dopaminergic receptor supersensitivity in the basal ganglia
resulting from chronic blockade of dopamine receptors by dopamine receptor antagonists.

The symptoms include abnormal, involuntary, irregular choreoathetoid movements of the muscles of the
head, limbs, and trunk. Perioral movements are the most common and include darting, twisting, and
protruding movements of the tongue; chewing and lateral jaw movements; lip puckering; and facial
grimacing. Ringer movements and hand clenching are also common. Torticollis, retrocollis, trunck twisting,
and pelvic thrusting occur in severe cases. Respiratory dyskinesia has also been reported. Dyskinesia is
exacerbated by stress and disappears during sleep.

There are three basic approachaes to tardive dyskinesia - prevention, diagnosis, and management. Using
the lowest dose possible of antipsychotics helps prevent tardive dyskinesia and use only when clearly
indicated. Atypical antipsychotics cause tardive dyskinesia less often than typical neuroleptics. Once the
diagnosis is made, clinicians should consider reducing the dosage or stopping the medication.

Choice B. Although tardive dyskinesia can begin on a steady dose of antipsychotic medication, there is an
even greater chance when the dose is reduced. This is known as withdrawal dyskinesia and the patient in
this example had no lowering of her dose prior to the onset of dyskinesia.

Choice C. Even if you don't know what Tardive Tourrette's Disorder is, you could recognize that a vocal
component is one of the requirements for the diagnosis of Tourrette's, which this patient did not have.
Tardive Tourrette's Disorder is reported to be precipitated by discontinuation or reduction of antipsychotic
medication, therefore it's the same scenario as tardive dyskinesia except for the vocal tic component. It
involves orofacial dyskinesia and limb chorea, tics, and vocalization and may terminate after an increase or
decrease in dosage.

Choice D. About 10 % of patients experience acute dystonia within the first few hours or days after
beginning dopamine receptor antagonists, therefore the timeline in this patient's history precludes this
diagnosis. Acute dystonia shares most of the dystonic symptoms of other neuroleptic-induced movement
disorders, with the exception of involvement of the eyes' upward, lateral movement (oculogyric
crisis). Children are particularly likely to exhibit opisthotonus, scoliosis, lordosis, and writhing movements,
which are scary and often lead to noncompliance with medication in general.

A 9 year old boy presents to the emergency department with his grandmother. She reports that earlier
that day the boy began to make facial expressions and repeatedly turn his neck to the right, slightly
raising his right shoulder. At first, this occurred once every 5-10 seconds and over the course of a few
hours decreased in intensity and slowed to once every 10 minutes. This has never happened before and
she is worried that she may have done something to cause this abnormal condition.

The boy has a history of psychosis and is taking medication. A review of the electronic medical record
reveals that he has been taking Risperidone for a year and a half and that there was a recent increase in
the dose within the last 2 weeks.

Physical exam is remarkable only for a very mild, episode twitching of the platysma on the right greater
than the left.

What is the most significant risk factor for the development of tardive dyskinesia?

A drug history
The presence of brain damage
The increasing age of the patient
Time since first exposure to antipsychotic medications
The presence of an affective disorder

25
Explanation

The correct choice is choice D.

The longer a patient takes dopamine receptor antagonists, the more likely he/she will experience tardive
dyskinesia. The key to remembering this is in the name of the disorder, "tardive". It rarely occurs until
after 6 months of treatment.

Between 10-20% of patients experience tardive dyskinesia after 1 year of exposure. Women are more
likely to be affected than men, and patients over 50 years of age, patients with brain damage, children,
and patients with mood disorders are also at high risk. Still, the single biggest risk factor is the length of
exposure to the antipsychotic medication.

A. Little data exists to support the claim that illicit drugs contribute to the development of tardive
dyskinesia. In fact, the pathogenesis may be dopaminergic receptor supersensitivity in the basal ganglia
resulting from chronic blockade. Because 1-5% of schizophrenic patients have similar abnormal
movements, it is possible that tardive dyskinesia may be related to the pathophysiology of schizophrenia
itself, which is still not completely clear.

B. The presence of brain damage is a risk factor for tardive dyskinesia, but it is not the "most significant"
one.

C. Age over 50 years is a risk factor, but not the "most significant" one.

E. The presence of a mood disorder is a risk factor, but not the "most significant" one.

A 12-year-old child presents to the psychiatrist with his parents. The parents' chief complaint is their
son's "odd "behaviors. The father explains that the son insists on his clothing being separated by color in
his closet or he will throw a temper tantrum. The mother agrees and adds that their son will often
organize things by color for hours on end and if interrupted will "explode". Additionally, he must touch
certain colors in a certain order each time he enters or exits a room.

Based on the most likely diagnosis, which of the following medications would BEST treat his compulsions,
rituals, and irritability?

Haloperidol
Carbamazepine
Dextroamphetamine
Fluoxetine
Valproate

Explanation

The correct answer is choice D

The patient in the above scenario most likely has obsessive compulsive disorder (more information is
needed to make a more specific diagnosis). Fluoxetine (Prozac) is the best choice for the above symptoms
of compulsions, irritability, and rituals, as would other SSRIs (such as Zoloft, Paxil, and Lexapro). While
they are usually known for their anti-depressant effects, SSRIs are a good choice when treating patients
with autism spectrum disorders who struggle with irritability and obsessive-compulsive tendencies. Higher

26
doses are often needed to treat the compulsions/obsessions than used for depressive symptoms, so
patience is key when managing these symptoms. Also, patient education regarding the time period
needed to work up to the higher dose is necessary to ensure compliance.

Haloperidol (Haldol) is an antispychotic most often used for the short-term management of agitation. This
would not be an appropriate long-term management medication for the treatment of compulsions and
rituals.

Carbamazepine (Tegretol) is an anticonvulsant used to control epilepsy.

Dextroamphetamine is a stimulant used for ADHD symptoms (such as hyperactivity, distractability,


impulsivity).

Valproate (Depakote) is an anticonvulsant used to control epilepsy. It can also be used as a mood
stabilizer and to help with the symtoms of aggression and irritability.

A 25 year old surgical nurse is concerned he is losing his mind. For the past 6 months he’s been
preoccupied with contamination on his ward. He has been avoiding touching patients, door knobs and has
been washing his hand excessively.

Which of the following treatment is most likely to decrease his preoccupation and his had washing?

fluoxetine
lorazepam
perphenazine
insight-oriented psychotherapy
nifedipine

Explanation

The correct answer is choice A.

This patient is exhibiting features of obsessive-compulsive disorder (OCD), an anxiety condition that as
characterized by intrusive obsessive thoughts, repetitive compulsive behaviors, or both. Rather than
reflecting real-life problems, the intrusive thoughts are inappropriate to the patient's situation. Thus, being
concerned that he might acquire HIV through a needle stick while treating an HIV positive patient, and
consequently taking appropriate precautions, would not be a feature of OCD, even if nurse felt some level
of anxiety caring for the patient. On the other hand, preoccupation with contamination to the point that
one cannot perform one's job qualifies as intrusive thoughts that are inappropriate. It is important to
realize that, while intrusive and disabling, such obsessions are not delusional. As opposed to disporders
involving delusions, in OCD the patient is fully aware that the thoughts are not rational. He knows that he
will not get seriously ill as a result of touching a patient or a door knob, and the same idea holds true with
respect to the compulsive features of OCD. Compulsions are repetitive behaviors such as handwashing in
this case, but they also can manifest as more mental actions, such as recitation certain phrases or prayers.

In the case of a nurse, it would be appropriate to wash one's hands subsequent to working with a patient
(as well as before, just as it is common for people to recite phrases to remind themselves of some
important facts. Medical students and residents, for instance, may whisper limerics, or sexually-suggestive
phrases, to remember the 12 cranial nerves, while an actor or trapeze artist may think of some favorite
word, prior to performing some act that requires a great deal of skill. Similarly, many people may have
thoughts perform rituals that are not particularly helpful; these would qualify as features of OCD. Many
people wash their hands more than is necessary, and many recite prayers or perform religious rituals
which have no practical value. What distinguishes any of these thoughts or actions from those of OCD is
that one's ability to perform normal activities is disrupted in the latter case. Thus, according to the

27
Diagnostic and Statistical Manual of Mental Disorders, Fourth Edition, Text Revision (DSM-IV-TR), this
nurse's symptoms lead to an easy diagnosis of OCD.

It is very important to understand the difference between OCD, which is an axis I disorder, and obsessive
compulsive personality disorder (OCPD). The former is treable through biological therapies such as
selective seratonin reuptake inhibitors (SSRIs) such as fluoxetine (choice A), as well as behavioral therapy
as an adjudant treatment, while features of the latter are built into the patient's persona. People with PCPD
tend to be rigid conformists who place excessive emphasis on schedules and calculations to the point of
being unable to function normally in group settings. In contrast to people with OCD, those affected with
the personality disorder to not feel as if their lives are being disrupted by intrusive thoughts. Rather, the
thought process leading them to stick rigidly to schedules and routines seems normal to them. The
Fraudian term, anal retentive often is used in association with such individuals, and indeed OCPD is
thought to develop at least in part due to rigid parenting. While this may or may not have anything to do
specifically with potty training, the concept to remember is that OCPD develops as a result of people and
events from early in the patient's childhood. Consequently, insight-oriented psychotherapy (choice D),
even psychoanalysis which can take years or decades, is just about the only intervention that can treat the
root cause of OCPD, although such individuals often are medicated, since their disorder can lead to
depression and anxiety. In contrast, insight oriented therapies are totally inappropriate for treating OCD.
The last thing that a person plagued by intrusive thoughts needs is to spend hours upon hours rehashing
the details of events from times long passed. What they need instead, in addition to medication, are quick
behavioral interventions; in the case of this nurse, systematic desensitization would be an excellent
measure. Once he is taking fluoxetine and beginning to respond, he could be reintroduced to touching
patients little by little, until it no longer provokes anxiety.

Lorazepam (choice B) is a benzodiazapene and thus useful as an anxiolytic in the short term. Like all
bensodiazapenes, however, it is highly addictive, nor is it useful in reducing the particular thought patterns
that lead to OCD. Perphenazine (choice C) is an antipsychotic drug and as explained above, OCD is not a
psychotic condition. Nifedipine (choice E) is a calcium channel blocker, useful in treating hypertension and
angina pectoris, as well as various other conditions such as Raynaud phenomenon and high altitude
pulmonary edema.

A 35 year old man presents to the emergency department with suicidal ideation. He describes significant
stress due to recent job loss and financial hardship. Further inquiry reveals a history of repeated job loss,
fraud charges and frequent arm slashing in attempts to decrease stress. The man was not disruptive as a
child.

The most likely diagnosis is which of the following?

adjustment disorder
schizophrenia
borderline personality disorder
antisocial personality disorder
dysthymic disorder

Explanation

The correct answer is Choice C

Persons with borderline personality disorder demonstrate a pervasive pattern of instability of interpersonal
relationships, self-image, and affects, and marked impulsivity beginning by early adulthood and present in
a variety of contexts, as indicated by five (or more) of the following: (1) frantic efforts to avoid real or
imagined abandonment; (2) a pattern of unstable and intense interpersonal relationships characterized by
alternating between extremes of idealization and devaluation; (3) identity disturbance with markedly and

28
persistently unstable self-image or sense of self; (4) impulsivity in at least two areas that are potentially
self-damaging; (5) recurrent suicidal behavior, gestures, or threats, or self-mutilating behavior; (6)
affective instability due to a marked reactivity of mood (e.g., intense episodic dysphoria, irritability, or
anxiety usually lasting a few hours and only rarely more than a few days); (7) chronic feelings of
emptiness; (8) inappropriate, intense anger or difficulty controlling anger (e.g., frequent displays of
temper, constant anger, recurrent physical fights) and (9) transient, stress-related paranoid ideation or
severe dissociative symptoms.

Adjustment disorder (Choice A) is incorrect because this diagnosis requires that the patient develop
“emotional or behavioral symptoms in response to an identifiable stressor(s) occurring within 3 months of
the onset of the stressor(s)” and that the “symptoms or behaviors are clinically significant as evidenced by
either of the following: (1) marked distress that is in excess of what would be expected from exposure to
the stressor or (2) significant impairment in social or occupational (academic) functioning. The patient in
our example has demonstrated significant behavioral problems throughout his adult life and so the time
component, in combination with the wrong symptom combination, renders this answer incorrect.

Schizophrenia (Choice B) is incorrect because in order to qualify for a diagnosis of schizophrenia a patient
must satisfy the criteria seen in the following 3 categories. The patient’s symptoms in our example do not
satisfy these criteria.

1. Characteristic symptoms: Two (or more) of the following, each present for a significant portion of
time during a 1-month period (or less if successfully treated): (1) delusions; (2) hallucinations;
(3) disorganized speech (ex: frequent derailment or incoherence); (4) grossly disorganized or
catatonic behavior; (5) negative symptoms (ex: affective flattening, alogia, or avolition)
2. Social/occupational dysfunction: For a significant portion of the time since the onset of the
disturbance, one or more major areas of functioning such as work, interpersonal relations, or self-
care are markedly below the level achieved prior to the onset (or when the onset is in childhood
or adolescence, failure to achieve expected level of interpersonal, academic, or occupational
achievement).
3. Duration: Continuous signs of the disturbance persist for at least 6 months. This 6-month period
must include at least 1 month of symptoms (or less if successfully treated) that meet Criterion 1
(i.e., active-phase symptoms) and may include periods of prodromal or residual symptoms.
During these prodromal or residual periods, the signs of the disturbance may be manifested by
only negative symptoms or two or more symptoms listed in Criterion 1

Antisocial personality disorder (Choice D) is incorrect because persons with this condition, also known as
sociopathy, are essentially selfish people who demonstrate a pervasive pattern of disregard for and
violation of the rights of others occurring since age 15 years, as indicated by three (or more) of the
following: (1) failure to conform to social norms with respect to lawful behaviors as indicated by repeatedly
performing acts that are grounds for arrest; (2) deceitfulness, as indicated by repeated lying, use of
aliases, or conning others for personal profit or pleasure; (3) impulsivity or failure to plan ahead; (4)
irritability and aggressiveness, as indicated by repeated physical fights or assaults; (5) reckless disregard
for safety of self or others; (6) consistent irresponsibility, as indicated by repeated failure to sustain
consistent work behavior or honor financial obligations; (7) lack of remorse, as indicated by being
indifferent to or rationalizing having hurt, mistreated, or stolen from another. As expected, many persons
with antisocial personality disorder become incarcerated.

Dysthymic disorder, or dysthymia (Choice E), is incorrect because dysthymia is a chronic low-level
depressive condition that requires the patient experience depressed mood most of the day, for more days
than not, as indicated either by subjective account or observation by others, for at least 2 years and
experience only 2 or more of the following symptoms during that 2-year time period: poor appetite or
overeating; insomnia or hypersomnia; low energy or fatigue; low self-esteem; poor concentration or
difficulty making decisions; or feelings of hopelessness. Both the short term of the patient’s symptoms and
the recent severe stressor eliminate this diagnosis as a possibility. Also, the patient’s personality symptoms
and time course are inconsistent with this diagnosis.

A 34 year-old man reports frequent episodes of chest pain, shortness of breath, and diaphoresis
associated with feelings of high anxiety during a visit to his family physician. The patient says that he is a
veteran of the war in Afghanistan, where he served as a helicopter pilot, evacuating wounded soldiers to
mobile surgical hospitals in combat zones, and returned to the United States in 2005. Since attending a
reunion with fellow veterans in August 2010, he has been increasingly troubled by memories of and

29
nightmares of some of the events that he witnessed, and symptoms intensified after he watched a film
about the events of September 11, 2001. Now, he experiences anxiety whenever he hears news about the
Afghanistan war, or any war. He has canceled his newspaper subscription, discarded his television, and
avoids going online, even to check his email, so as to avoid news headlines and photographs. He does not
understand why he is troubled by memories from more than 5 years ago, and wonders if he might suffer
from a physical ailment. A detailed evaluation, including a cardiac work-up, reveals no evidence of any
medical illness.

Which of the following diagnoses is most likely?

PTSD, chronic type


acute stress disorder
adjustment disorder with anxiety
PTSD, acute
PTSD, delayed onset

Explanation

The correct answer is choice E.

Posttraumatic stress disorder (PTSD) is an anxiety disorder that develops as a result of an individual
experiencing, or witnessing, a traumatic event that threatens his/her life of the lives of others, or results in
others being killed. Another key feature is a feeling of helplessness in association with the event. Initially,
the affected individual responds with horror and fear, but then later constantly reexperiences the event
with different manifestations. The nature of the provocative event can vary widely but in adults generally
involves physical violence, while in children inappropriate sexual experience may elicit PTSD as well. Many
cases have been documented resulting from war experiences, as well as from the terrorist attacks of
September 11 2001. If symptoms last less than 3 months, PTSD is considered to be acute (choice D), while
symptoms lasting more than 3 months constitutes chronic PTSD (choice A). Delayed onset PTSD (choice
E) is defined as PTSD which manifests beginning at least 6 months following the provocative event. In this
patient's case, the trauma occurred during the early 2000s when he was a young pilot. Although not stated
unequivocally, it sounds as if he never was injured physically himself, but flying medical evacuation in a
combat zone, surely he must have seen several horrible injuries.

Often hearing about traumatic experiences that others are having can set off an anxiety attack. Occuring in
an estimated 8-10 percent of the population, PTSD is extremely debilitating, as it causes affected
individuals to avoid many normal day-to-day activities. Though publicized most in relation to men returning
from war, incidence actually is higher in women. Nevertheless, various studies involving veterans
participants from the wars in both Afghanistan and Iraq have elucidated a strong correlation between
duration of combat exposure and the development of PTSD. Soldiers from Afghanistan reported less
experience in combat, and thus a a lower incidence of any mental health disorder as compared with
veterans of the war in Iraq, who reported greater combat exposure. Treatment of PTSD can include drugs,
especially selective serotonin uptake inhibitors (SSRIs), but this is valuable only in combination with
psychotherapy.

What acute stress disorder (choice B) has in common with PTSD is that the individual has experienced or
witnessed a life-threatening traumatic event or serious injury to his/herself or others. The timeframe for
symptoms, however, is much shorter, being 2 days to 4 weeks. In contrast, adjustment disorder (choice C)
develops in connection with a change in one's milieu which is not violent, but situational.

The children of a 67 year-old woman asks their family physician for advice about their mother’s behavior 4
weeks after the death of her husband of 40 years. They are concerned because she weeps whenever she
comes upon an object in her home that she associates with him, her appetite has decreased, and she has

30
had 2 kg (4.4-lb) weight loss. In addition, she now awakens 1 hour before the alarm goes off each
morning. She is able to care for herself, but does not leave her home for any social activities and has
made it clear she does not enjoy visits from her family.
Which of the following is the most likely explanation and what would be its appropriate management?

Top of Form
Normal Bereavement- uncomplicated
normal grief reaction, and she would benefit from diazepam therapy
pathologic grief reaction, and she should be treated with an antidepressant
pathologic grief reaction, and she should be treated with psychotherapy
pathologic grief reaction, and she should be encouraged to move in with one of her
children

Explanation

The correct answer is Choice A

Death is a well understood complication of life, and grief is a natural response to death of a loved one who
has been close to a person. However, severity and length of grief symptoms can become abnormal. The
DSM-IV-TRtr states, "As part of their reaction to the loss, some grieving individuals present with symptoms
characteristic of a Major Depressive Episode (ex: feelings of sadness and associated symptoms such as
insomnia, poor appetite, and weight loss).

While accommodation must be made for normal cultural variations of expression of bereavement, the
diagnosis of Pathologic Grief Reaction is generally not made unless the symptoms are still present 2
months after the loss. However, the presence of certain symptoms that are not characteristic of a "normal"
grief reaction may be helpful in differentiating bereavement from a Major Depressive Episode. These
include 1) guilt about things other than actions taken or not taken by the survivor at the time of the death;
2) thoughts of death other than the survivor feeling that he or she would be better off dead or should have
died with the deceased person; 3) morbid preoccupation with worthlessness; 4) marked psychomotor
retardation; 5) prolonged and marked functional impairment; and 6) hallucinatory experiences other than
thinking that he or she hears the voice of, or transiently sees the image of, the deceased person. While the
recent widow’s symptoms in our example would qualify her for the symptom component of complicated
bereavement, the short duration of her symptoms eliminates her from this diagnosis.

Normal grief reaction and she should be treated with an diazepam therapy (Choice B) is incorrect because,
while she is experiencing a normal grief reaction thus far, the usual treatment for simple bereavement is
time and supportive counseling. The last thing an elderly patient needs at this critical juncture is to be
introduced to a potentially habit-forming drug with sedative side effects that could result in falls or other
mishaps.

Pathologic grief reaction, and she should be treated with an antidepressant (Choice C) is incorrect because
as explained above she does not qualify for a pathologic grief reaction. Again, the usual treatment for
simple bereavement is time and supportive counseling.

Pathologic grief reaction, and she should be treated with psychotherapy (Choice D) is not the best selection

31
as an answer to this question because grief usually resolves with time. Supportive therapy usually helps
grief suffers, but long term in-depth psychotherapies such as cognitive behavioral therapy are generally
not required for grief reaction resolution.

Pathologic grief reaction, and she should be encouraged to move in with one of her children (Choice E)
would be an overreaction to simple grief unless the person demonstrated marked incapacity to manage his
or her affairs. One of the things that help grief sufferers heal is consistency in the other aspects of their
now damaged life; introduction of other changes tends to be unsettling and can actually hinder the
patient’s ability to accept their new situation and move on with their life.

A 66 year-old man presents at your clinic, brought by his wife, who explains that over the course of the
past 4 years he has had difficulty recognizing his grandchildren. Additionally, he has been unable to plan
daily activities. He has mispayed bills, and left pots on the stove, causing the smoke alarm to ring.
Occasionally, his wife has found him wandering through the house at night with a "blank" look on her face.
Most recently he has been exhibiting difficulty in recalling the names of common objects. In speaking with
the man, you find that his speech is limited to simple sentences, consisting of two to three words.
Most likely, you will diagnose this patient with which of the following?

Top of Form
amnestic disorder
pseudodementia
vascular dementia
Alzheimer disease
substance-induced dementia

Explanation

The correct answer is choice D.

Criteria for the diagnosis of Alzheimer disease (AD) are laid out in the Diagnostic and Statistical Manual of
Mental Disorders, Fourth Revision, Text Revision (DSM-IV-TR), as well as in the National Institutes of
Health-Alzheimer's Disease and Related Disorders Association (NIH-ADRDA) and the Consortium to
Establish a Registry in Alzheimer's Disease (CERAD). Generally, a history of gradually-progressing memory
loss is required in the absence of other dementia-causing conditions, which must be ruled out. This man
exhibits as gradual progression of symptoms, including pervasive memory deficit, language difficulty, and
extreme impairment of daily functioning.

Thus, a working diagnosis of AD may be warranted while the patient is worked up for conditions such as
vitamin B-12 (cobalamin) deficiency, disorders of thyroid function, hematological disorders, liver
abnormalities, and syphilis, all of which can cause progressive dementia. Since a possible link has been
elucidated between cognitive impairment and vitamin D deficiency, levels of vitamin D should be tested as
well. As far as imaging goes, while MRI, CT, SPECT, and PET cannot reveal abnormalities pathognomonic
for AD, they can be used to rule out lesions that cause similar presentations. Similarly, EEG can be used to

32
assess for signs of Creutzfeldt-Jakob disease.

Amnestic disorder (choice A) is is not correct, because this is purely a memory condition, while this patient
also has difficulty in language and cognitive function. Pseudodementia (choice B) is wrong as well, as this
is not a dementive disorder, but a depressive syndrome. Vascular dementia (choice C) features step-by-
step deterioration, due to gradually-increasing involvement of more and more blood vessels. Substance-
induced dementia (choice E) develops as a result of substance abuse. While no history of abuse is given in
this question, substance-induced dementia should remain in the differential diagnosis until more is known,
since it is the second most common cause of this patient's symptoms.

Julie, a 27-year-old nurse, has extracurricular interests in trekking and painting. She recently broke up
with her long term boy friend. Two months later, she lost interest in her hobbies, became convinced that
she would not be able to work again, and came to the conclusion that life was not worth living so she
consumed 60 tablets of phenobarbital in a suicide attempt. She denies alcohol or drug abuse or history of
psychiatric concerns.

She is most likely suffering from which of the following conditions?

Adjustment disorder with mixed emotional features


Conversion disorder
Depressive disorder
Post-traumatic stress disorder (PTSD)
Pseudo-depression

Explanation

The correct answer is Choice C

In order for a patient to qualify for the diagnosis of depressive disorder, also known as major depression, they
must experience depressed mood most of the day and 5 (or more) of the following (depressive) symptoms
during the same 2-week period: poor appetite or overeating; insomnia or hypersomnia; low energy or fatigue;
low self-esteem; poor concentration or difficulty making decisions; or feelings of hopelessness”. A potential
complication of these symptoms includes suicidality.

Adjustment disorder with mixed emotional features (Choice A) is incorrect because, while the timeline (less
than 3 months from onset of stressor) and the affective symptoms (sadness, low mood, lethargy, suicidality)
would qualify her for the diagnosis of adjustment disorder, she does not display any of the conduct symptoms
required for this subtype of affective disorder. To review: adjustment disorders require that the patient develop
“emotional or behavioral symptoms in response to an identifiable stressor(s) occurring within 3 months of the
onset of the stressor(s)” and that the “symptoms or behaviors are clinically significant as evidenced by either
of the following: (1) marked distress that is in excess of what would be expected from exposure to the stressor
or (2) significant impairment in social or occupational (academic) functioning.

Conversion disorder (Choice B) is incorrect because this bizarre neurobehavioral condition is defined as a
physical change or condition that presents as an alteration or loss of a physical function suggestive of a
physical disorder. Conversion disorder is presumed to be the expression of an underlying psychological conflict
or need. Examples might include unexplained blindness, inability to speak, limb paralysis, etc, in the absence
of any identifiable physical cause.

Posttraumatic stress disorder, known by the acronym PTSD (Choice D), is incorrect because our patient does
not display the symptom pattern of this condition. Posttraumatic stress disorder is a severe anxiety disorder

33
that can develop after exposure to any terrifying event that involves the threat of death to oneself or to
someone else, or to one's own or someone else's physical, sexual, or psychological integrity. Diagnostic
symptoms for PTSD include (1) re-experiencing the original trauma(s) through flashbacks or nightmares,
avoidance of stimuli associated with the trauma, (2) increased arousal – such as difficulty falling or staying
asleep, anger, and (3) hypervigilance. Formal diagnostic criteria (both DSM-IV-TR and ICD-9) require that the
symptoms last more than one month and cause significant impairment in social, occupational, or other
important areas of functioning.

A 37 year-old male presents with disturbing flashbacks and nightmares about his time in during the
September 11 terrorist attacks in New York city. Associated with his anxiety, he also experiences chest
pain, shortness of breath, and diaphoresis, and he is diagnosed with post-traumatic stress disorder.

Which of the following medications would most likely help him with his condition?

Imipramine (Tofranil)
A monoamine oxidase inhibitor (MAOI)
Clonidine (Catapres)
Sertraline (Zoloft)
Prazosin (Minipress)

Explanation

The correct answer is choice D.

Posttraumatic stress disorder (PTSD) is an anxiety disorder that develops as a result of an individual
experiencing, or witnessing, a traumatic event that threatens his/her life of the lives of others, or results in
others being killed. Another key feature is a feeling of helplessness in association with the event. Initially,
the affected individual responds with horror and fear, but then later constantly reexperiences the event
with different manifestations. The nature of the provocative event can vary widely but in adults generally
involves physical violence, while in children inappropriate sexual experience may elicit PTSD as well. Many
cases have been documented resulting from war experiences, as well as from the terrorist attacks of
September 11 2001. If symptoms last less than 3 months, PTSD is considered to be acute, while
symptoms lasting more than 3 months constitutes chronic PTSD. Delayed onset PTSD is defined as PTSD
which manifests beginning at least 6 months following the provocative event.

Often hearing about traumatic experiences that others are having can set off an anxiety attack. Occurring
in an estimated 8-10 percent of the population, PTSD is extremely debilitating, as it causes affected
individuals to avoid many normal day-to-day activities. Though publicized most in relation to men returning
from war, incidence actually is higher in women. Nevertheless, various studies involving veterans
participants from the wars in both Afghanistan and Iraq have elucidated a strong correlation between
duration of combat exposure and the development of PTSD. Soldiers from Afghanistan reported less
experience in combat, and thus a a lower incidence of any mental health disorder as compared with
veterans of the war in Iraq, who reported greater combat exposure.

Treatment of PTSD can include drugs in combination with psychotherapy. First line drugs are selective
serotonin uptake inhibitors (SSRIs), such as sertraline and paroxetine (choice D). Tricyclics such as
imipramine (choice A) such as and MAO inhibitors such as phenelzine (choice B) have been shown to
exhibit some effectiveness. Phenelzine has little effect on hyperarousal, however, and neither class of
drugs is used as first-line treatment. In the case of MAOIs, special monitoring of diet is necessary to avert
hypertensive crises. The α2 adrenergic agonist clonidine (choice C) can be added as well, though not as
first-line treatment, and the α1 blocker prazosin (choice E) has been shown to be useful in reducing
nightmares and other sleep disturbances of PTSD.

34
A 50 year old man develops difficulty walking while receiving drug therapy for paranoid behavior. Physical
examination show masked facies and diffuse muscle rigidity. He is slow in initiating movement and walks
with a shuffling narrow based gait.
Which of the following drugs is the most likely cause of these findings?

Top of Form
barbiturate
benzodiazepine
monoamine oxidase inhibitor
phenothiazine
tricyclic compound

Explanation

The correct answer is Choice D

Awareness of potential side effects of psychotropic medications is paramount in the medical management
of psychiatric illness. The patient in our example is displaying classic pseudoparkinsonism, or drug-induced
parkinsonism, which is a common occurrence with administration of members of the phenothiazine
(antipsychotic) class. This occurs due to the mechanism of action of phenothiazines, which is reduction of
dopamine levels that are elevated in psychosis. Excessive dopamine reduction results in a patient who is
relatively dopamine deficient—the biochemical equivalent of Parkinson’s disease. Clinical manifestations
include sluggish movement, wide-based shuffling gait, masked facies, cogwheeling rigidity, pill-rolling
tremor, etc. Phenothiazines that possess this side effect concern include chlorpromazine, haloperidol,
perphenazine, thioridazine, and fluphenazine, amongst others.

Barbiturate (Choice A) is incorrect because the side effect concerns of this class of drugs do not involve
muscular dysfunction; but rather include oversedation, dizziness, syncope, development of tolerance that
can lead to addiction concerns, and death from respiratory depression in cases of overdose.

Benzodiazepine (Choice B) is incorrect because the side effect concerns of this class are similar to those of
barbiturates and again do include pseudoparkinsonian symptoms. Side effects of benzodiazepines such as
diazepam, lorazepam, and triazolam include oversedation, dizziness, syncope, tolerance leading to possible
addiction, and possible death in overdose if taken with alcohol or other sedative medications.

Monoamine oxidase inhibitors, also known as MAOIs (Choice C), is incorrect because members of this class
of antidepressant, which are used expressly for treatment-resistant depression and other difficult to
manage psychiatric illnesses, are used sparingly because of their unique side effect profile. MAOIs not only
possess numerous drug-drug interaction concerns, they also interact with foods that contain tyramine
(wine, aged cheeses, etc) and can cause a lengthy list of side effects including hypertension, anxiety, heart
palpitations, drowsiness, and dizziness; however, this list does not include Parkinson-like muscular and gait
dysfunction.

Tricylic compound (Choice E), or tricyclic antidepressant, is not correct because the side effect concerns of
this class of drugs are mostly related to their anticholinergic properties, and include dry mouth, sedation,
constipation, sedation, and urinary retention; they do not, however, have the potential to cause the
parkinsonian side effects seen in our example. In addition, overdose may result in potentially fatal

35
cardiotoxicity so great caution must be used when prescribing tricyclic antidepressants to patients with
cardiac concerns or who are at significant risk for suicide attempt.

A 14-year-old African-American girl has a 12-month history of irritable mood, hypersomnia, and poor
concentration. She recently verbalized feelings of self-deprecation to a school counselor after breaking up
with her boyfriend. Physical examination reveals normal vital signs, no neurological findings, and a BMI of
22 which is in the normal range. Her mother notes that the girl has left the dinner table without finishing
her meal several times during the last month.
Which of the following is the most likely diagnosis for this patient's condition?

Top of Form
Adjustment disorder
Dysthymic disorder
Major depressive disorder
Major depressive episode
Obsessive–compulsive disorder

Explanation

The correct answer is choice B.

Although additional information easily could change the answer choice, the question is testing whether you can make
a judgement between dysthymia, depression, and adjustment disorder based on an initial impression. Adjustment
disorder (choice A) can manifest with a range of behavioral or emotional symptoms, but the DSM-IV-TR criteria
require that this occur within 3 months of some stressor. Thus, for instance, if you were to find out that the parents
divorced recently, resulting in the child going back and forth between two houses, adjustment disorder would be in
the differential diagnosis. Since no such information was given, choice A can be eliminated.

Obsessive–compulsive disorder (OCD, choice E) is identified more often today in adolescents than it was in the
past. While its symptoms can overlap slightly with those of mood disorders, and while the same types of drugs can
be used to treat OCD and mood disorders, OCD actually is an anxiety disorder that features intrusive thoughts or
repetitive actions performed in response to the anxiety.

The difference between major depressive disorder (choice C) and major depressive episode (choice D) has to do with
timing, the former being an ongoing condition and the latter being depression during at least a two week period. To be
classified as major depression according to the DSM-IV-TR, whether depressive disorder or depressive episode, a
minimum of five features must be present. These may include irritability, changes in sleep patterns (such as
hypersomnia), poor concentration and low self esteem, all of which this girl has exhibited, but this adds up to four
features as most, with the last feature noted only once when the girl spoke with the school counselor. Additional
features can include weight loss or gain, recurrent thoughts of death, suicidal ideation, suicide attempt, or specific plan
for suicide, diminished interest or pleasure, and fatigue or loss of energy.

Dysthymic disorder, on the other hand, is characterized by a depressed mood throughout most of the day, but requires
only a minimum of two of the above features in addition to the depressed mood. Although in adults diagnoses of
dysthymia requires the symptoms to be present for a minimum of two years, only one year is required for diagnosis in

36
adolescents and children.

Females have approximately double the risk that males have of developing dysthymia, with an overall presence of
dysthymia in the population of 6 percent. At least one study has revealed that dysthymic disorder is more common in
African-Americans and Mexican-Americans compared to Caucasians. Dysthemic disorder is treated by with medication
and psychotherapy. Studies have suggested that selective serotonin reuptake inhibitors (SSRIs), tricyclic
antidepressants, and serotonin norepinephrine reuptake inhibitors (SNRIs) all about equally effective. Interpersonal
psychotherapy and brief supportive psychotherapy have been compared with one another and with drug therapy
(sertraline). It was found that interpersonal therapy combined with sertraline was the most effective approach.

A 39-year-old married mother of two presents for psychiatric treatment, stating, "My rituals are taking over my
life." She reports being afraid that she or her sons will die if activities are not carried out in the "proper way." For
her, this means repeating all household tasks three times. In addition, certain chants must be repeated numerous
times throughout the day in order to keep her children safe. Certain items, such as old clothing or towels are not
discarded for fear that "something terrible will happen." She states that these rituals have consumed all of her time,
such that she is no longer able to work or socialize.

Which one of the following medications would be the most effective choice to treat this patient?

Alprazolam
Amitriptyline
Neuroleptics
Clomipramine
Clonazepam

Explanation

The correct answer is Choice D.

The diagnosis is Obsessive-Compulsive Disorder (OCD), which is characterized by recurrent obscessions OR


compulsions sufficient to cause marked distress to the person. They are time-consuming and interfere significantly
with the person's normal routine, occupational functioning, usual social activities, or relationships.

Tricyclics and SSRI's are FDA-approved for the treatment of OCD. Clomipramine is a tricyclic antidepressant. Though SSRI's are m
tolerable than TCA's regarding toxic effects, no SSRI was listed as an option. Antipsychotics are considered adjunctive.

Other interventions include behavioral therapy, psychotherapy, psychoeducation of family and, as a last resort, bilateral cinguloto

A. Alprazolam is a benzodiazapine that can be relied upon to provide prompt relief of acute anxiety. Dependence
and withdrawal are serious problems with this drug. Because SSRI's and other antidepressants are as effective in
treating anxiety disorders, alprazolam is not considered first-line therapy for anxiety disorders.

B. Amitriptyline is a tricyclic antidepressant also, but it has strong anticholinergic effects. It is used in the
treatment of chronic pain, insomnia and prophylaxis of migraine headaches.

C. The term neuroleptics refers to medications that block dopamine receptors, commonly known as anti-
psychotics. There is no evidence of psychosis or severe mania in this patient so such a strong class of psychotropic
medications is not indicated.

37
E. Clonazepam is also a benzodiazepine. It is approved as an anticonvulsant and is useful in acute mania. It is used
panic disorder, social phobia, and general anxiety.

You are the neurology resident consulted by internal medicine to evaluate a patient for what appears to be a
movement disorder. The consult reads: “37 year-old apparently homeless man admitted for dehydration,
malnutrition, fatigue, and general failure to thrive. His neurological/mental status examinations are remarkable
for odd affect, strange responses to my questions, bizarre lip-smacking movements, and odd choreoathetoid-like
body movements. Neurological disorder suspected. Please evaluate and treat.
This patient’s diagnosis is likely the result of chronic administration of which of the following medications?

Top of Form
fluphenazine
carbidopa-levodopa
nortryptiline
clonidine
diazepam

Click on image to Zoom

Explanation

The correct answer is Choice A

Fluphenazine is an antipsychotic (neuroleptic). Chronic antipsychotic administration is associated with tardive


dyskinesia, a permanent movement disorder. Tardive dyskinesia development is linearly associated with both drug
potency and duration of treatment. The patient in question is likely a schizophrenic who has taken neuroleptics
throughout his adult life.

Clozapine, quetiapine, ziprasidone, olanzapine, and risperidone (< 6 mg daily) exert a favourable effect on TD by
several possible mechanisms. TD remains a complex disorder because its mechanism is not yet fully understood.
Current TD vulnerability models, which are not mutually exclusive but rather complementary, include genetic
vulnerability, disease-related vulnerability, and decreased functional reserve. Mechanisms of TD induction by
typical antipsychotics include prolonged blockade of postsynaptic dopamine receptors, postsynaptic dopamine
hypersensitivity, damage to striatal GABA neurons, and dam- age to striatal cholinergic neurons. Although one
unifying theory may ultimately be discovered, current understanding suggests that several of these mechanisms
may coexist to pro- duce TD or that different mechanisms may predominate in different patients or patient
populations.

Atypical antipsychotics may be associated with a lower risk of TD by decreasing striatal dopamine receptor
sensitivity, either indi- rectly through serotonin or directly by looser binding to the D2 receptor. Further, compared
with typical antipsychotics, atypicals are less likely to cause changes in basal ganglia vol- ume; thus, they are less

38
likely to damage cells directly involved in the pathophysiology of TD.

Levodopa-carbidopa (Choice B) is incorrect because it is used in the treatment of Parkinson’s disease. Excessive
dose could result in constitutional symptoms and twitching, but not the symptoms seen in this patient.

Nortyptiline (Choice C) is incorrect because it is a tricylic antidepressant that is not associated with movement
concerns. Side effect concerns with tricyclic antidepressants are due to their anticholinergic activity and include dry
mouth, constipation, sedation, confusion and urinary retention.

Clonidine (Choice D) is incorrect because it is a centrally-acting hypertensive also used for patients with agitation
concerns. Its major side effect concern is sedation. It is not associated with movement disorders.

Diazepam (Choice E) is not correct because this drug is a member of the benzodiazepine class and its side effect
concerns include sedation, drowsiness, dizziness, and short term memory disturbance along with potential addiction
issues.

A 35-year-old woman with paranoid schizophrenia presents to the emergency department with a recent increase in
auditory hallucinations. She states that the voices are very disturbing, and she feels that her illness has ruined her
life. She is single, lives alone, and is unemployed. Which of the following must be done before making any
emergency treatment decisions?

Top of Form

Assess for suicide risk


Drug of abuse screen
Magnetic resonance imaging of the brain
Pregnancy test
Spinal tap

A 52 year-old Caucasian male who suffers from depression and alcohol dependence and is known as a “frequent
flier” in your hospital’s alcohol treatment unit presents to your emergency department requesting care. He states
that he usually drinks 30 beers per day and that he “ran out of money and friends with beer” 2 days ago.

Which of the following combinations of signs and symptoms would be consistent with alcohol withdrawal symptoms within the fir
24-36 hours?

1. Diaphoresis
2. Alcohol withdrawal delerium
3. Generalized tonic clonic seizures
4. Insomnia

5. Hallucinosis

39
1 and 3 only
1, 4 and 5
1, 2, 4 and 5
1, 3, 4 and 5
2, 3, 4, and 5
All of the above are true

Explanation

The correct answer is Choice D

Persons suffering from alcohol withdrawal syndrome develop a fairly predictable physiologic response to abrupt alcohol cessation.
response usually correlates with the person’s usual daily alcohol intake. Alcohol acts as a depressant on both the autonomic and
peripheral nervous systems. Reflex tachycardia develops when the heart rate, which has become accustomed to being slowed by
alcohol, suddenly has this autonomic counterbalance removed. In addition, gastrointestinal disturbances such as nausea, vomitin
and abdominal pain are often seen by alcoholic patients both due to the direct toxic effects of chronic alcohol intake and as part o
alcohol withdrawal syndrome. Nervous excitability such as irritability, tremor, and anxiety are also regular manifestations of the
syndrome.

Choices 1, 3, 4 and 5 all occur within the first 36 hours. Choice 2, alcohol withdrawal delerium, doesn't present classically till 48-7
hours later.

Figure 1: Diagnostic Criteria for Alcohol Withdrawal

40
Figure 2: Symptoms of Alcohol Withdrawal Syndrome

A 13-month-old boy is brought to your office by his mother because her mother-in-law said her child's speech was delayed. Whil
the boy is playful, generally happy and speaks in one and two-word sentences, her mother has become concerned that he is not
developing as quickly as other children his same age.

Your developmental examination reveals that he can stand on both feet and take steps with assistance. He hugs you and speaks
mostly in inarticulate sounds but you do comprehend a few, disjointed words. His behavior seems goal directed and your medica

41
examination reveals no abnormalities. He can pick up small items with two fingers, but won't scribble with a crayon. You notice t
he doesn't follow one-step commands.

Which of the following statements would be most appropriate to tell his mother?

He should be following one-step commands.

His social skills are progressing nicely, but you would like to see him already walking unassisted.

Children start scribbling on their own at different ages. You want to do more comprehensive testing if he's still not
scribbling by age 16 months.
You are really impressed that he's able to stand and are surprised to see him even trying to stand. It isn't
until 14 months that children usually begin to demonstrate standing and walking behavior.

The reason you were testing if he could pick up small objects with two fingers was to assess for Adaptive
Delay. Normally that task is mastered in the 6-8 month age range and you were concerned that he might be
developmentally delayed. Fortunately, he was able to do the task.

Explanation

The correct answer is Choice C.

You can count on the fact that you will encounter develomental milestones questions on your exam. If you have children or frequ
access to them, you will be at an advantage. In fact, read the milestones while you're babysitting or playing with your children to
imprint them in permanent memory.

This parent is distressed and is, by extension, under your care along with the child. Be careful not to alarm concerned parents
unnecessarily.

In general, children do develop on different schedules and there are frequently large windows of time for a child to develop a
particular skill. You will see far more developmentally normal children than abnormal, both in real life and on your exam. So, it pa
to memorize what is normal.

Developmental milestones are points in time whereby a concern will be raised if a child hasn't acquired a specific ability by the tim
the milestones passes. For example, by age 4 months a child should be able to roll from laying prone to on their back. This doesn
mean that the child is advanced if they could do this at 3 weeks of age. Abnormal children on your exam will likely be pushing the
limits or bordering on normal, but they are not trying to trick you. Read the question carefully. Further testing and referral to a
specialist is indicated if a child is found to be delayed developmentally.

Here are the developmental milestones that will help you answer questions such as these:

MOTOR MILESTONES

 4 mo. - Rolls front to back

42
 6 mo. - Sits with support
 9-10 mo. - Sits alone
 10 mo. - Pulls to a stand
 10-12 mo. - Crawls
 10-18 mo. - Walks alone
 15-24 mo. - Runs
 3 yrs. - Rides a tricycle
 5-7 yrs. - Rides a bicycle

LANGUAGE MILESTONES

 4-6 wks. - Smiling


 3 mo. - Coos
 6 mo. - Babbles
 10-14 mo. - Uses jargon
 12 mo. - Speaks first word
 15 mo. - Follows one-step commands
 22 mo. - Follows two-step commands
 3 yrs. - Follows three-word sentences

ADAPTIVE MILESTONES

 3 mo. - Mouthing
 6 mo. - Transfers objects
 10 mo. - Drinks from cup
 11-12 mo. - Picks up a raisin
 15 mo. - Scribbles
 12-15 mo. - Uses spoon
 <5 yr. - Bladder trained for boys (one year earlier for girls)
 <4 yr. - Bowel trained

Choice A. This child still has 2 months before you need to be concerned that he's not following one-step commands.
This is completely within the realm of normal.

Choice B. Walking unassisted can occur from 10-18 months. He still has 5 months before you need to do futher developmental
testing.

Choice D. Similar to Choice B, the range of ages for standing alone and walking vary widely among children. This
makes this subject important to know well so that you can educate parents on the wide range of normal
development. Even if he doesn't walk unassisted until the 18th month, this is still normal.

Choice E. The pincer reflex is not to be confused with the purposeful, fine-motor movements require to pick up a small object wit
fingers. Picking up a raisin is an easy test that can be done in the office and should be accomplished by 11-12 months of age.
Therefore, it shouldn't surprise you that this 13-month old boy can do it.

A 55 year-old male presents at the ER on account of severe pain in his right eye. Physical examination reveals a red ring
surrounding the iris along with increased intraocular pressure in the right eye. After obtaining a thorough history (with the help o
the patient's family), you decide that this condition is the result of a drug.
Which of the following agents triggered this condition?

43
Top of Form

propranolol
amitriptyline
cimetidine
diazepam
malathion

Explanation

The correct answer is choice B.

This patient seems to be suffering from acute glaucoma, also called narrow angle glaucoma. Such attacks can be
provoked by agents with anticholinergic mechanisms of action working through an antimuscarinic effects on the iric
pupillary constrictor muscle. This leads to pupillary dilation, which in turn narrows the angle within the eye's
anterior chamber. A tricyclic antidepressant, amitriptyline is notorious for its high level of anticholinergic side
effects.

Propranolol (choice A) is a non-selective, beta-antagonist. While it would not cause a glaucoma attack, it may help
in preventing an attack though blocking action on the beta-receptors of the ciliary body, thus decreasing
production of aqueous humor.

Cimetidine (choice C) is an H2 antihistamine. It decreases release of gastric acid, but has no anticholinergic effects,
and thus would not provoke a glaucoma attack.

Diazepam (choice D) also has no anticholinergic effects and does not cause glaucoma or precipitate attacks. It is a
benzodiazepine that works on the chloride channel receptors of neurons.

Used as an insecticide, Malathion (choice E) is an organophosphate cholinesterase inhibitor. If the patient had used
this, acetylcholine levels would be increased, and this would widening the angle, not narrow it.

A 32 year old female presents to your emergency department with a 2 days of steadily worsening nausea, dizziness,
trembling, diarrhea and blurry vision. She denies known recent exposure to anyone with viral illness or excessive
medication use/overdose. You have your medical student evaluate the patient and he proceeds to diagnose the
patient with likely viral gastroenteritis and recommends clear fluids, bed rest, and chlorpromazine suppositories.
After the student completes his presentation you ask the patient to stand and walk across the room. The patient
rises unsteadily then proceeds to walk slowly across the room with great difficulty, almost falling on two occasions.

You ask your medical student if he is absolutely sure that his diagnosis is correct. He hesitates when you offer

44
him a chance to look at the chart and review the patient’s list of current medications prior to his instituting his
treatment plan.

What medication might the patient be taking that could explain her symptoms?

acetaminophen
lithium
valproate
diltiazem
opiod toxicity

Explanation

The correct answer is Choice B

Appropriate lithium level maintenance can be a difficult process—especially for the complicated patient population who is usually
taking it (those suffering from manic-depressive illness). Elevated lithium levels can affect a wide variety of organ systems includ
gastrointestinal (nausea, vomiting, diarrhea), CNS (dizziness, confusion, syncope, staggering gait, muscle twitching or limb tremo
visual disturbances, slurred speech), cardiac (tachycardia or dysthymia), and renal (pedal edema and acute renal failure).

Acetominophen (Choice A) is incorrect because, while excessive acetominophen administration can lead to hepatic and gastrointe
(nausea, bloating, abdominal pain) concerns as well as constitutional symptoms (jaundice), the marked neurological symptoms w
not be present unless the person was in an advance stage of acetaminophen toxicity from overdose.

Diltiazem (Choice D) is not correct because this calcium channel blocker, which is used in the management of hypertension, angin
pectoris, and many other cardiovascular conditions, possesses no significant gastrointestinal side effect concerns.

Table 1: Drugs and physiological states with clinically important interactions with lithium

45
Table 2: Indications for diuresis vs hemodialysis in cases of lithium overdose

An 18 year old African-American male is brought to your office by his grandmother, with whom he lives. She states that he just
“acting right.” He has been more quiet than usual recently, keeping to himself, and he expresses concern about a knot on the ba
of his head. He mentioned once to family that he thought someone had placed something into his head in order to monitor him.

46
confirms this suspicion upon detailed questioning, but he is withdrawn and makes poor eye contact. His vitals are unremarkable.
Exam is remarkable only for mood and behaviors mentioned above.

Lab work-up reveals normal CBC, chemistries, and TSH. A urine drug screen is negative. Family history reveals that the patient’s
mother is an inpatient in a narcotic rehabilitation facility. The grandmother thinks she is being treated for mental problems, also,
does not know more details.

All of the following symptoms, if present in this patient, would support a diagnosis of schizophrenia EXCEPT:

Top of Form

Thinking at times that he has special abilities or powers that others do not have
Hearing police sirens outside his bedroom window that others cannot hear
Spending exorbitant amounts of money on gambling without stopping to sleep
Having no expression on his face and speaking in a monotone voice
Having difficulty understanding information and using it to make decisions

Explanation

The correct answer is choice C.

These symptoms are suggestive of a manic episode, which would support a diagnosis of bipolar disorder or schizoaffective disord
(mood disorder plus schizophrenia symptoms). If a patient has bipolar symptoms or significant depressive symptoms along with
schizophrenic symptoms, the diagnosis would be schizoaffective disorder rather than schizophrenia.

The other answers above describe typical symptoms that can be associated with schizophrenia. Choice A is an example of a delus
or false belief that does not change. There are several types, including delusions of control, as described with the patient in the
question (someone placing something in his head), and delusions of grandeur, as described in choice A.

Choice B is a hallucination, or a false perception. It can occur with regard to any of the senses. Choice B is an auditory hallucina

Choice D repressents a flat affect, a so-called negative symptom of schizophrenia. Positive symptoms are hallucinations, delusion
disorganized speech and behavior, and catatonia. These are often managed with antipsychotics. Negative symptoms are more
difficult to treat, and they include decreased range of facial expression, monotone voice, anhedonia (lack of pleasure in activities)
alogia (poverty of speech).

47
Choice E represents the cognitive difficulties that occurs often in schizophrenia. These may include problems with organization,
concentration, working memory, abstraction, understanding nuances or social interaction, etc. See table below for DSM-IV criter
schizophrenia diagnosis.

Figure 1. DSM-IV-TR Diagnostic Criteria for Schizophrenia

A. Characteristic symptoms: Two (or more) of the following, each present for a significant portion of time during a 1-month perio
less if successfully treated):

1. delusions
2. hallucinations
3. disorganized speech (e.g., frequent derailment or incoherence)
4. grossly disorganized or catatonic behavior
5. negative symptoms, i.e., affective flattening, alogia, or avolition

Note: Only one Criterion A symptom is required if delusions are bizarre or hallucinations consist of a voice keeping up
a running commentary on the person's behavior or thoughts, or two or more voices conversing with each other.

B. Social/occupational dysfunction: For a significant portion of the time since the onset of the disturbance, one or
more major areas of functioning, such as work, interpersonal relations, or self-care, are markedly below the level
achieved prior to the onset (or when the onset is in childhood or adolescence, failure to achieve expected level of
interpersonal, academic, or occupational achievement).

C. Duration: Continuous signs of the disturbance persist for at least 6 months. This 6-month period must include at
least 1 month of symptoms (or less if successfully treated) that meet Criterion A (i.e., active-phase symptoms) and
may include periods of prodromal or residual symptoms. During these prodromal or residual periods, the signs of the
disturbance may be manifested by only negative symptoms or two or more symptoms listed in Criterion A present in
an attenuated form (e.g., odd beliefs, unusual perceptual experiences).

D. Schizoaffective and Mood Disorder exclusion: Schizoaffective Disorder and Mood Disorder With Psychotic
Features have been ruled out because either (1) no Major Depressive, Manic, or Mixed Episodes have occurred
concurrently with the active-phase symptoms; or (2) if mood episodes have occurred during active-phase
symptoms, their total duration has been brief relative to the duration of the active and residual periods.

E. Substance/general medical condition exclusion: The disturbance is not due to the direct physiological effects of a
substance (e.g., a drug of abuse, a medication) or a general medical condition.

F. Relationship to a Pervasive Developmental Disorder: If there is a history of Autistic Disorder or another Pervasive Developmen
Disorder, the additional diagnosis of Schizophrenia is made only if prominent delusions or hallucinations are also present for at le
month (or less if successfully treated).

Classification of longitudinal course (can be applied only after at least 1 year has elapsed since the initial onset of active-phase
symptoms).

Episodic With Interepisode Residual Symptoms (episodes are defined by the reemergence of prominent psychotic symptoms).

Also specify if with:

48
 Prominent Negative Symptoms
 Episodic With No Interepisode Residual Symptoms
 Continuous (prominent psychotic symptoms are present throughout the period of observation); also specify if: With
Prominent Negative Symptoms
 Single Episode In Partial Remission; also specify if: With Prominent Negative Symptoms
 Single Episode In Full Remission Other or Unspecified Pattern

*Reprinted from Diagnostic and Statistical Manual of Mental Disorders, 4th Edition, Text Revision. Washington, DC, American
Psychiatric Association, 2000. Copyright © 2000, American Psychiatric Association.

A 33 year old gay office worker complains of a four month history of “anxiety”. He claims they are discreet episodes that occur w
on the streetcar or at work, while sitting at his computer terminal. The feeling of nervousness are accompanied by sweating, ting
in the back of his head, a pounding heart, and shortness of breath; most symptoms subside after 10 minutes, with the exception
a mild headache. For this patient, which of the following is the most appropriate management?

1. psychotherapy exploring issues of sexuality and anxiety


2. medical work-up to rule out organic disease
3. gradual desensitization regarding his fear of taking the streetcar
4. imipramine
5. lorazepam

1, 2
1, 2, 3
2, 4, 5
2, 4
All of the above

Explanation

The correct answer is choice C.

There are a few conditions which could account for this "anxiety attacks". On the one hand, he could have panic
disorder. This condition features spontaneous panic attacks can include tachycardia and dyspnea, such as the
patient describes. Their frequency may vary from a few episodes per year to several episodes per day. Such panic
attacks can occur as part of other anxiety disorders, however. Panic disorder is diagnosed when they develop
without discernible, predictable precipitating factors.

49
Attacks featuring similar symptoms and sensations also can occur as manifestations of organic disorders, ranging
from mere hypoglycemia to temporal lobe seizures. Thus, a complete medical and neurological workup (option 2) is
essential. The tricyclic antidepressant imipramine reasonable to include in the management plan, since it is used
successfully to treat panic disorder, which is at the top of the differential diagnosis, even as the medical workup
proceeds. Another option would be a selective serotonin reuptake inhibitors (SSRI). Opinions on the value of
benzodiazepines such as lorazepam (option 5) are split, but the position of the American Psychiatric Association is
that they can be used effectively as one element of panic disorder treatment.

Psychotherapy exploring issues of sexuality and anxiety (option 1) is completely inappropriate. Based on the question
stem, there is no reason to assume that this patient is insecure about, or questioning or exploring, his sexual
orientation. Secondly, his attacks occur while at his computer terminal and while on the streetcar, suggesting that
they have no relation to the fact that he is homosexual.

Gradual desensitization (option 3) is a technique employed in treating a specific phobia. Thus, for instance, if an
individual has a fear being around dogs, he desensitization might begin by showing him photographs of dogs, for
instance, the introducing him to real dogs little by little. While it certainly is possible to develop a phobia of riding
in a streetcar, this man's symptoms also develop in other situations, such as while he is seated at his computer
terminal. Thus, specific phobia is not a likely diagnosis.

A muscular 24-year-old man presented himself to the admitting office of a state hospital. He told the admitting physician that he
taken thirty 200 mg tablets of chlorpromazine (Thorazine) in the bus on the way over to the hospital. After receiving medical
treatment for the "suicide attempt," he was transferred to the inpatient ward.

On mental status examination the patient told a fantastic story about his father, a famous surgeon, who had a woman he was
operating on die in surgery and who was then killed by the husband of the woman. The patient then stalked his father's murdere
several thousand miles across the United States and, when he found him, was prevented from killing him, at the last moment, b
the timely arrival of the man's 94-year-old grandmother. He also related several other intriguing stories involving his $64,000 sp
car, which had a 12-cylinder diesel engine, and about his children, two sets of identical triplets. AIR those stories had a grandios
tinge, and none of them could be confirmed. The patient claimed that he was hearing voices, as on television or in a dream. He
answered affirmatively, to questions about thought control, thought broadcasting, and other Schneiderian first-rank symptoms;
also claimed depression. He was oriented and alert and had a good range of information except that he kept insisting that it was
Germans (not the Russians) who had invaded Afghanistan. There was not evidence of any associated features of mania or
depression, and the patient did not seem elated, depressed, or irritable when he related the stories.

On the ward the patient bullied the other patients and took food and cigarettes from them. He was reluctant to be discharged, a
whenever the subject of his discharge was brought up, he renewed his complaints about “suicidal thoughts" and “hearing voices
was the opinion of the ward staff members that the patient was not truly psychotic but merely feigned his symptoms whenever t
subject of the disposition of his case came up. They thought that he wanted to remain in the hospital primarily so that he could b
the other patients and be a "big man" on the ward.

What is the diagnosis?

50
Factitious Disorder
Malingering
Confabulation
Delusional disorder
Bipolar disorder

Explanation

The correct answer is Choice A.

This man certainly has outlandish stories, but determining if they are true, determine if he realizes they are false, and determinin
these beliefs rise to the level of psychosis or delusions are key in determining the proper diagnosis.

Clearly these stories are wild enough to be almost impossible to be true. The fact that the question stem tells you the opinion of t
staff is that this is not psychosis is important. You can assume the statements to be false and it appears he has some secondary
in trying to be the "big man" on the wards. Recall, too, that secondary gain is unconscious.

B. The diagnosis of factitious disorder requires that the man be unconscious of the fact that his statements are
false. This means that he's not lying or malingering because he believes his statements are true. In practice, it can
be difficult to discern malingering from factitious disorder and patients are often misdiagnosed. If the question
stem had mentioned any admission or acknowledgement on the part of the patient that his statements weren't
true, this would definitely be malingering. To err on the side of caution means that you need to use the diagnosis
of malingering as an absolute last resort, never assuming it on the USMLE

C. Confabulation is a rather unique term that describes the unconscious filling of gaps in memory by imagined or
untrue experiencdes that a person believes but has no basis in fact. Most often, it is associated with organic
pathology, such as Korsakoff's Dementia.

D. This is an understandable answer choice, however delusions are defined as deeply held beliefs that are held
onto despite evidence to the contrary. Usually, the delusion involves one or two circumstances and doesn't always
affect other parts of a person's life, such as believing aliens are in the closet and feeling fine when not at home
(assuming there are no aliens in the closet). The prominent feature in this case are numerous, wild stories of
adventure and intrigue; more of a manic flavor than delusional.

E. Had there been any suggestion of insomnia, increased energy, or other bipolar-type symptoms, this would have
been the answer. Notice that only outlandish stories were provided and that was all you had to go on. The
diagnoses will be pretty easy to figure out on the USMLE, so spend a little time learning the first- and second-line
therapies for each diagnosis.

51
A 10 year-old girl is brought to your clinic by parents on account of decreased performance in school. She is very social, energe
and very talkative child, and testing by school counselors has revealed no learning disability. Despite being aided by private tuto
her performance in school has declined, and both she, her parents, and teachers believe this is the result of mere impatience. Sh
has difficulty falling asleep, yet is difficult to wake in the morning, and while she feels hungry often she gets up during means,
unable to remain at the table. Since physical examination reveals a healthy child in the 75th percentile for height, weight, and h
circumference, you diagnose her with attention-deficit hyperactivity disorder and explain to the parents that methylphenidate is
likely to improve the child's condition. But the parents are concerned about side effects.

Side effects of methylphenidate therapy for hyperactivity include each of the following, except:

anorexia
insomnia
tachycardia
dependence (addiction)
growth suppression

Explanation

The correct answer is choice D.

ADHD is notorious for being overdiagnosed, as well as overtreated, but this girl's history and presentation suggests
that it is a perfectly appropriate diagnosis in her case. The hyperactivity component clearly is a characteristic of the
child, as noted by all of her teachers. Nevertheless, hyperactive children are difficult to distinguish from other
children through preschool and the early part of grade school, because all young children have very short attention
spans.

Since the skills taught and the assignments given are age-appropriate, such children can perform at average
levels, or even above average, but then start to fall behind as the the concentration level and the amount of time
to complete an assignment increases as they get older. In many cases, such children have higher than average
intelligence and have difficulty paying attention or concentrating, because they are bored and actually would do
better in an academic environment that is more challenging. Thus, often special classes for gifted and talented
students actually include some children diagnosed with ADHD. ADHD is about twice as common in boys as in girls
and is present in all ethnic groups. With treatment, it is likely that this girl's performance in school will improve
significantly. However, it also is likely that the condition will persist into adulthood as occurs in 30-80 percent of
children with ADHD.

ADHD can be treated with stimulants such as methylphenidate, which is not addictive if prescribed for this purpose
(choice D). Common doses for children are 0.3-0.5 mg/kg/dose PO tid/qid, and thus the initial dose is 2.5-5 mg for
the immediate release preparation. Methylphenidate is thought to work by increasing dopamine levels. On overall
effect on increasing alertness results in increased attention. However, since the effect is stimulatory, insomnia
(choice B) and anorexia (choice A) can result. since dopamine is a catecholamine, supraventricular tachycardia
(choice C) also has been documented. While the effects of methylphenidate on growth (choice E) has been

52
controversial, studies have supported a growth suppressive effect of this drug.

A 38 year old woman is brought to the emergency room by her husband. He states that she lost her job 3 months ago and has b
withdrawn and irritable ever since. Her appetite has diminished and she has lost over 10 pounds. She stays in bed most of the d
and has lost interest in her usual social activities.

Which one of the following statements about the treatment of depression is NOT correct?

Selective serotonin reuptake inhibitors (SSRIs) should be used with caution in the first few weeks of
treatment because they have been associated with a worsening of suicidal ideations
SSRIs are associated with more adverse effects than tricyclic antidepressants
Venlafaxine (Effexor) is more effective than tricyclic antidepressants
Cognitive therapy has been shown to effectively improve depressive symptoms
Electroconvulsive therapy should be considered if there is no response to several trials of antidepressant drug therapy

Explanation

The correct answer is Choice B.

Depression is a state of low mood and aversion to activity. Patients with depression have a higher rate for suicide
than the general public. The negative symptoms of depression (such as lack of energy) tend to prevent patients
from acting out their suicidal ideation. However, suicidal ideation is relatively common at the start of
antidepressant treatment since treatment reduces negative symptoms which in turn increase a person’s energy
level (choice A); unfortunately, it takes antidepressants longer to eliminate suicidal ideation. During this interval,
the patient has the energy required to act out his suicidal thoughts. Therefore, it is recommended that family
members monitor these patient's behavior for any signs of suicidal ideation during the first eight weeks of therapy.

Patients with depression feel:

 Sad
 Anxious
 Empty
 Hopeless
 Helpless
 Worthless
 Guilty
 Irritable

53
 Restless

Symptoms of depression:

 Loss of interest
 Loss of appetite or overeating
 Difficulty in concentrating
 Suicide ideation
 Insomnia
 Excessive sleeping
 Fatigue
 Loss of energy

SSRIs work by preventing the reuptake of serotonin and typically have fewer adverse effects than tricyclics (choice B), although
drowsiness, dry mouth, nervousness, anxiety, insomnia, decreased appetite, weight gain and sexual dysfunction may occur.

Venlafaxine (choice C) is a serotonin-norepinephrine reuptake inhibitors (SNRIs) and works on both norepinephrine and serotonin
SNRIs have fewer adverse effects than tricyclics and the side effects are similar to SSRIs.

Cognitive therapy (choice D) is a type of psychotherapy used in depression and is developed to help the patient overcome difficul
by changing dysfunctional thinking, behavior and emotional responses. It develops skills for modifying beliefs and distorted think

Electroconvulsive therapy (choice E) is a medically controlled seizure used among patients with serious and unresponsive depress
It is quite effective in patients who do not respond to trials of different antidepressant drug therapies.

Figure 1: Side effects of Antidepressants Classes

54
A 23 year-old female comes to see you in your psychiatry office. She was referred to you from a recent inpatient stay for a moto
vehicle accident. The patient states she was looking at her nose in the rear-view mirror while driving when she accidentally ran a
stop light and was hit by another car. She has had two rhinoplasty surgeries, but she still feels that her nose is big and unattract
She has a plastic surgery appointment later this week. Vital signs are normal, and her physical exam is unremarkable. You see

55
nothing out of the ordinary with regard to her nose. You recommend psychotherapy and advise her to cancel her plastic surgery
appointment.

In addition, what medication would be most helpful for this patient?

Fluoxetine 40 mg daily
Escitalopram 30mg daily
Paroxetine 20mg daily
Clomipramine 25mg daily
Fluvoxamine 100mg daily

Explanation

The correct answer is choice B.

All of these medications may be used to treat the patient, but only escitalopram is listed at a high enough
recommended dose for the patient's diagnosis. This patient likely has Body Dysmorphic Disorder. In this disorder,
patients become obsessed with a real or imagined physical defect to the point that it interferes with functioning. It
has significant overlap with other psychiatric disorders, but it is considered a separate somatoform disorder. It is
often missed or misdiagnosed.

SSRIs and other antidepressant medications often have to be used at high doses in order to benefit these patients.
According to psychiatric research, these are some of the average dosages shown to be effective in these patients
(note: off-label use of these medications):

56
These patients also seem to benefit from psychotherapy, particularly cognitive behavioral therapy. Patients often
seek cosmetic procedures from dermatologists or plastic surgeons to improve their symptoms, but this does not
seem to be successful.

Amongst the patients you are managing on your inpatient psychiatry service is a patient who suffers from schizophrenia,
disorganized type. The attending asks you to make a presentation on the different symptoms that can be present in schizophren
patients, including problems with thought form and content.

Examples of disturbances in thought content in schizophrenia include which of the following:

1. phobias
2. hallucinations
3. illusions

4. delusions

Top of Form

57
1,2,3 only
1,3 only
2,3,4 only
4 only
All of the above are correct

Explanation

The correct answer is Choice E

All of the above disturbances in thought content in schizophrenia are true.

Patients with schizophrenia and other psychotic illnesses often demonstrate disturbances in both thought content and form. Thou
process refers to the quantity, tempo (rate of flow) and form (or logical coherence) of thought.Thought content is defined, literall
the thoughts that the patient has or demonstrates. Accordingly, disturbances in thought process would result in irregularities of
speech (too fast due to racing thoughts, too slow due to impaired thinking—defined as paucity of thought).

However, disturbances in thought content would include irregular thoughts that, while not true, the patient believes or experience
be true. Examples would include hallucinations—seeing or hearing things that are not present and that other others cannot see o
hear; illusions—misperceiving sensory input (ex: believing a telephone wire to be a giant snake that has come to eat the patient)
delusions—a fixed false belief from which the patient cannot be shaken (ex: the FBI has tapped the phones of a paranoid
schizophrenic and are using the information they are gathering from these taps to develop a strategy to capture and incarcerate
patient).

You have been treating a 7-year old boy with Attention Deficit Hyperactivity Disorder in your primary care
clinic for the past four months. He has no other medical or psychiatric diagnoses. Despite compliance with
taking the stimulant you prescribed, he remains unable to function both in school and at home.

What is your next step in the management of this patient?

58
Switch to a different class of medicine, such as tricyclic antidepressants or buproprion.

Explain to the parents that boys have ADHD ten times more often than girls and offer
reassurance.

Offer investigational drugs, such as modafinil, tacrine, and donepezil and get them enrolled in a
study.

Encourage behavioral strategies that rely less on rewarding positive behavior and offer a less
structured environment.

Refer to a child psychiatrist.

Explanation

The correct answer is Choice E.

Attention Deficit Hyperactivity Disorder (ADHD) is a maladaptive pattern of clinically significant inattention,
hyperactivity, or impulsivity. It begins before age 7 and may persist into adulthood. Symptoms may not be
apparent in a structured setting but must be apparent in more than one setting (i.e., school and home).

Stimulants remain a good, first-line treatment despite the debate about over-prescribing. In up to 90
percent of cases, stimulant medication helps children improve their approach to schoolwork, get more
focused and organized, think before acting, get along better with others and break fewer rules. They often
seem happier, too.

There are three, very common indications for referral of a child with ADHD to a child psychiatrist:

1. Age younger than 6 years at presentation.


2. Comorbid psychiatric, neurologic, or medical conditions.
3. Lack of response to initial treatment with a stimulant or atomoxetine.

He was otherwise healthy, but requires refferal for lack of response to initial treatment with a stimulant.
The boy in this question stem was 7 years old and, therefore, did not present with ADHD younger than 6
years old. Caution should be used with stimulants in children younger than six.

Choice A. Tricyclicc, buproprion, clonidine and guanfacine hydrochloride are alternate first-line medications
for the treatment of ADHD in school-aged children. Choice A, therefore, is a valid treatment protocol but
this case will be better managed by a child psychiatrist.

Choice B. The prevelance of ADHD is estimated between 2-16% of the population. The male-to-female
ratio is 4:1 for predominantly hyperactive type and 2:1 for predominantly inattentive type.

Choice C. Modafinil, tacrine, and donepezil are investigational drugs so this is a valid option, only better
managed by a child psychiatrist.

Choice D. Behavioral strategies are an important part of the treatment of children with ADHD. Strategies
include an increased structure and rewarding positive behavior.

A 22 year-old college student presents to the university clinic complaining of 10 days’ of slowly worsening
insomnia, fatigue, malaise, difficulty concentrating, and feeling terribly sad. Lengthy discussion reveals

59
that her grandmother died 2 weeks ago and that this has upset her greatly. She admits to feeling lost
without her as she both greatly admired her and considered her a confidante. She denies decreased
performance in either her schoolwork or social schedule, although she states that things are harder to
enjoy right now than they usually are.

What is her correct diagnosis?

Normal bereavement (grief reaction)


Adjustment disorder with depressed mood
Acute Major Depression
Dysthymia
Either B or C could be correct

Explanation

The correct answer is Choice A

Grief is a natural human response to loss of someone or something greatly treasured. Grief only becomes
pathological (alternative psychiatric diagnosis should be entertained) when it is disproportionate to the
loss, results in impaired social or vocational performance, or lasts greater than 12 months.

Adjustment Disorder with depressed mood (Choice B) is incorrect because even though the patient did
experience a loss and her symptoms did develop in response to this loss, she is not experiencing social or
vocational dysfunction and her symptom presence has been too brief.

Major Depression (Choice C) is also not correct because she has an obvious inciting cause for her symptom
development and has not developed impairment in social or vocational functioning.

Dysthymia (Choice D) is incorrect because dysthymia is defined as a state of generally low mood that has
lasted for at least 2 years. The briefness of this patient’s episode eliminates dysthymia as a correct
diagnosis.

Either B (adjustment disorder with depressed mood) or C (acute major depression) could be correct
(Choice E) has to be incorrect because the diagnostic criteria of these two conditions are different. The
diagnostic criteria for dysthymia and major depression are defined in the Diagnostic and Statistical Manual
of Mental Conditions (DSM-IV-TR). Both length of episode and number of symptoms that must be present
differentiate the two diagnoses. Dysthymia, a chronic low-level depressive condition, requires the patient
experience “depressed mood most of the day, for more days than not, as indicated either by subjective
account or observation by others, for at least 2 years”; major depression, however, only requires that
patients experience “five (or more) of the following (depressive) symptoms during the same 2-week
period”. Number of symptoms also differentiates the two conditions: Dysthymia requires the “presence,
while depressed, of 2 (or more) of the following: poor appetite or overeating; insomnia or hypersomnia;
low energy or fatigue; low self-esteem; poor concentration or difficulty making decisions; or feelings of
hopelessness”. As seen above, major depression requires that 5 of these symptoms be present during the
appropriate time interval.

A 30 year old Caucasian female is referred to your psychiatry office from her OB-Gyn for postpartum
depression symptoms. She is 3 months postpartum and has been crying several times a week. She
frequently has feelings of worthlessness and hopelessness throughout the week. She denies suicidal or
homicidal ideation. She has difficulty concentrating on the baby, although she wants to be more involved,
and she cannot seem to find enjoyment in her family or outside activities. She has no psychiatric history

60
prior to this, and her family is supportive. You recommend that she begin individual cognitive behavior
therapy and a trial of oral medications.

Which of the following medications is most effective in post-partum depression?

paroxetine
escitalopram
fluoxetine
bupropion
imipramine

Explanation

The correct answer is choice A

Of the choices, paroxetine is the preferred antidepressant for postpartum depression based on
randomized-controlled trials. Other therapies that have evidence supporting their effectiveness for
postpartum depression include:

 Nondirective counseling
 Individual cognitive behavioral therapy
 Interpersonal psychotherapy
 Psychodynamic psychotherapy

Other treatments that have not been sufficiently studied, and therefore have not been proven beneficial,
include light therapy, group cognitive behavioral therapy, mother-infant interaction coaching,
psychoeducation with partner, and telephone-based peer support. These studies may have a single small
clinical trial or no trials at all in their favor.

There are 2 main kinds of postpartum depression:

 postpartum or maternity “blues,” a mild mood problem of short duration


 postpartum major depression, a severe and potentially life-threatening illness.

Postpartum blues affect 50%–80% of new mothers. Symptoms usually begin 3–4 days after delivery,
worsen by days 5– 7, and tend to go away by day 12. The new mother may have mood swings with times
of feeling tearful, anxious, or irritable, interspersed with times of feeling well; and she may have trouble
sleeping. If symptoms last longer than 2 weeks, it is important to seek medical attention, since 1 in 5
women (20%) with postpartum blues goes on to develop postpartum major depression.

Postpartum major depression can begin anytime in the first days or weeks after delivery and is far more
serious than post-partum blues. It is a type of mood disorder, a biological illness caused by changes in
brain chemistry, and is not the mother’s fault or the result of a “weak” or unstable personality. It is a
medical illness which professional treatment can help. The symptoms of postpartum major depression
include a depressed mood most of the day, nearly every day, for at least 2 weeks and losing interest or
pleasure in activities one used to enjoy. Other symptoms include fatigue, feeling restless or slowed down, a
sense of guilt or worthlessness, difficulty concentrating, insom- nia, and recurring thoughts of death or
suicide. The woman may also be very anxious about her baby’s health. Some women with very severe
postpartum depression develop psychotic thoughts (hallucinations or delusions), and there is a small but
real chance that they could harm their children.

About 10%–15% of new mothers develop postpartum major depression, but it is often not diagnosed until

61
several months after the birth. Sometimes the new mother puts off seeking medical help because of lack of
energy caused by the illness or fear of what others will think. The new mother may feel guilty about being
depressed when she is supposed to be happy. Family members and physicians may also fail to recognize
the symptoms as depression, believing instead that the mother’s mood is a normal reaction to the stress of
caring for the infant.

The most important risk factor for postpartum depression is having had a similar episode before. Over half
of the women who have had a previous depression after the birth of a child will become depressed again
when they give birth. If a woman has been depressed at any other time in her life, her risk of developing a
postpartum depression also increases, from about 10% (risk for a woman with no history of depression) to
25%. Women with manic-depressive illness (also known as bipolar disorder) are also at very high risk.
Women are also more vulnerable if they have been depressed during pregnancy, if they had significant
premen- strual mood symptoms before they were pregnant, or if they have close relatives with depression
or bipolar disorder. It is very im- portant for a woman with a personal or family history of a mood disorder
to talk to her doctor so that she can be monitored closely. Stressful situations (e.g., health problems in the
baby, marital discord, not having a partner) may also place a woman at an increased risk for postpartum
major depression.

For postpartum depression in a breast-feeding mother, the experts recommend serotonin reuptake
inhibitors (SSRIs). Their top choice among these is Zoloft (sertraline), the most widely studied
antidepressant in breast-feeding mothers and their infants. While small amounts enter breast milk, little or
no medication can be detected in infants, and there appear to be no adverse effects. Paroxetine (Paxil) is
also a highly-rated second choice. Paroxetine is not detectable in breast milk or nursing infants. Two other
widely used SSRIs, fluoxetine (Prozac) and citalopram (Celexa), enter breast milk in small amounts but are
viewed as acceptable alternatives. If a mother took fluoxetine or citalopram during her pregnancy and
needs to stay on medication after delivery, experts do not think it is necessary to change to another drug.
Tricyclic antidepressants, an older type of medication, are also viewed by experts as an appropriate choice
for breast-feeding mothers. Imipramine (Tofranil) and nortrip- tyline (Pamelor) are 2 examples. Tricyclics
usually cause more side effects in the mother than SSRIs but are sometimes more effective.

A 41 year-old woman working as an Executive Director in a corporation is convinced that the management
has denied her promotion, by preparing false reports about her competence. Additionally, she asserts that
the management has forged her signature on sensitive documents, in order to convict her. She files a
complaint at the police station and requests for security. This behavior notwithstanding, she attends to her
work and manages the household very well.
She is suffering from which of the following disorders?

Top of Form

paranoid schizophrenia
late-onset psychosis
persistent delusional disorder
obsessive compulsive disorder
schizoid personality disorder

Explanation

62
The correct answer is choice C.

According to the Diagnostic Manual of Mental Disorders, Fourth Edition, Text Revision (DSM-IV-TR),
delusional disorder is the appropriate diagnosis, based on the presence of nonbizarre delusions in the
absence of psychotic symptoms, or mood symptoms. In psychopathological terms, to be given delusional
status, a belief not only must be based on incorrect inference about external reality despite evidence to the
contrary, but also must not be accepted by other members of an individual's culture. Thus, beliefs
associated with religions, for example, are exempt from being labeled delusional in the DSM-IV-TR, and
this definition will continue in the DSM V. Nonbizarre delusions refers to delusions regarding conditions that
could occur in real life - being followed by enemies, or loved by a secret admirer, as opposed to being
chosen by a god or goddess to deliver a message to humanity, or having the power to become invisible,
which would be categorized as bizarre delusions.

Patients also must not exhibit simultaneous hallucinations, disorganized speech, or negative symptoms
such as affective flattening, or grossly disorganized behavior. These would be criteria for a diagnosis of
schizophrenia (choice A) if present for more than 6 months, or schizophreniform disorder if present from 1
to 6 months. The fact that this patient is fully functional in all aspects of life not connected with her
delusion, however, means that features of schizophrenia other than the delusion are not likely to be
present. For the same reason, late onset psychosis (choice B) can be eliminated.

Obsessive-compulsive disorder (OCD, choice D) is an anxiety condition that as characterized by intrusive


obsessive thoughts, repetitive compulsive behaviors, or both. Compulsions are repetitive behaviors, such
as hand washing, but they also can manifest as more mental actions, such as recitation certain phrases or
prayers. Rather than reflecting real-life problems, the intrusive thoughts are inappropriate to the patient's
situation. It is important to realize that, while intrusive and disabling, such obsessions are not delusional.
As opposed to delusional disorder, in OCD the patient is fully aware that the thoughts are not rational. This
patient, on the other hand, believes that her thoughts about what people are doing to her are reasonable,
and this is the nature of paranoia. Individuals with OCD, on the other hand, are not paranoid.

What schizoid personality disorder (choice E) has in common with schizophrenia is not the delusional state,
but rather the lack of interest in social relationships. This can manifest as a tendency to a solitary lifestyle,
emotional coldness, and secretiveness. Incidence of this personality disorder is higher in families prone to
schizophrenia.

An 11-year-old girl asked her mother to take her to a psychiatrist because she feared she was going
crazy. Several times during the past two months she had awakened confused about where she was until
she realized that she was on the living room couch or in her little sister's bed, even though she went to
bed in her own room. When she woke up in her older brother's bedroom, she became concerned and felt
guilty about it. Her younger sister said that she had seen the patient walking during the night, looking like
"a zombie," that she did not answer when called, and that she had walked at night several times but
usually went back to her bed. The patient feared she had amnesia because she had no memory of
anything happening during the night. The patient had no history of seizures or of similar episodes during
the day.

Electroencephalogram and physical examination results were normal. The patient's mental status was
unremarkable except for some anxiety about her symptoms and the usual early adolescent concerns. Her
school and family functioning were excellent.

What is the most likely diagnosis?

63
Nightmare disorder
Familial sleep paralysis
Somniloquy
Sleepwalking disorder
Sleep terror disorder

Explanation

The correct answer is Choice D.

Sleepwalking, also known as somnambulism, consists of a sequence of complex behaviors that are initiated
in the first third of the night during deep NREM (stage 3 and 4) sleep and frequently, although not always,
progress - without full consciousness or later memory of the episode - to leaving bead and walking about.

According to the DSM-IV-TR, Sleepwalking Disorder is diagnosed by repeated episodes of rising from bed
during sleep and walking about, occuring during the first third of the major sleep episode. While
sleepwalking, the person has a blank stare, is relatively unresponsive to the efforts of others to
communicate with him or her, and can be awakened only with great difficulty. On awakening (either from
sleepwalking or the next morning), the person has amnesia for the episode. Within several minutes after
awakening from the sleepwalking episode, there is no impairment of mental activity or behavior (although
there may initially be a short period of confusion or disorientation). The sleepwalking causes clinically
significant distress or impairment in social, occupational, or other important areas of functioning. The
disturbance is not due to the direct physiological effects of a subsante or generel medical condition.

A. Nightmares are long, frightening dreams from which persons awaken scared - which did not happen in
this case. Like other dreams, nightmares almost always occur during REM sleep and usually after a long
REM period late in the night. Some persons have frequent nightmares as a lifelong condition; others
experience them predominantly at times of stress or illness.

The DSM-IV-TR defines Nightmare Disorder by repeated awakenings from the major period of sleep or nap
with detailed recall of extended and extremely frightening dreams, usually involving threats to survival,
security, or self-esteem. The awakenings generally occur during the second half of the sleep period. On
awakening from the frightening dreams, the person rapidly becomes oriented and alert (in contrast to the
confusion and disorientation seen in sleep terror disorder and some forms of epilepsy).

B. Familial sleep paralysis is characterized by a sudden inability to execute voluntary movements either
just at the onset of sleep or on awakening during the night or in the morning. None of this was reported in
this patient's history.

C. Sominoloquy is also called sleeptalking. This is common in children and adults, and occurs during all
stages of sleep. The talking usually involves a few words that are difficult to distinguish. Long episodes of
talking involve the sleeper's life and concerns, but sleeptalkers do not relate their dreams during sleep, nor
do they often reveal deep secrets. Episodes of sleeptalking sometimes accompany night terrors and
sleepwalking. Sleeptalking alone requires no treatment.

E. Sleep terrors differ from nightmares in the fact that the person has no memory of a bad dream, but
simply awake in terror. Occassionally people will recall a single frightening image, as opposed to the long,
frightening and realistic nature of a nightmare. About 1-6% of children have the disorder, which is more
common in boys than in girls and which tends to run in families.

64
A 68-year-old married woman is referred for psychiatric evaluation by her family physician. She complains
of 10 months of "anxiety episodes," which are characterized by hyperventilation, feeling cold, chest
tightness, difficulty breathing, sweating, and intense fear. The first episode occurred while she was
awaiting an MRI scan to evaluate a knee injury. Since then, these events have occurred several times
each week-generally upon awakening in the morning and occasionally while at work. The patient has
begun to greatly fear having an episode while at work, though she continues to go. She is also afraid to go
to sleep at night due to fear of awakening with one of these episodes.

Which of the following DSM-IV diagnoses BEST describes this patient's symptoms?

Panic disorder with agoraphobia


Panic disorder without agoraphobia
Generalized anxiety disorder
Posttraumatic stress disorder
Adjustment disorder with anxiety

Explanation

The correct answer is Choice B.

This question involves the process of elimination and understanding the definition of agoraphobia,
according to the DSM-IV.

Her symptoms are discrete, acute events that don't happen all of the time. Therefore, panic disorder fits
better than a generalized anxiety. The question, really, then is "what is agoraphobia?" When you can
decipher the question underneath the question, you have progresses well, indeed.

A. Patients with agoraphobia rigidly avoid situations in which it would be difficult to obtain help. This
patient fears an attack at work, but continues to go. The difference is a pathologic response and, like most
psychiatric diagnoses, rigid avoidance is an unhealthy response to this condition (unless she operates
heavy machinery or is involved in other potentially dangerous work).

C. Generalized anxiety is exactly what it sounds like; fear and trepidation about practically anything and/or
everything. This patients anxiety were a surprise and, except for fearing the next panic attack, we are not
told that she spends her time worrying and fretting about everything in general.

D. There is no evidence of trauma, which would be necessary to make this diagnosis. If the question had
indicated a death at work, say due to a machinery accident, this would be a good option.

E. We are not told of a specific stressor, which is necessary and a give-away for the diagnosis of
adjustment disorder. Whenever you can point to a reason for the pathology, think adjustment disorder.
Just make sure it occurs within 3 months and resolves within 6 months of the removal of the stressor.

A 45-year-old woman presents to her primary care physician with the chief complaint of increased stress.
Upon further questioning, you elicit that the sensation of being stressed is only present during her
performances as a concert pianist. She describes feeling a sense of fear, a rapid heartbeat, and tremors.
These symptoms interfere with her job by causing her to cancel concerts. At this point, she has already
tried deep breathing, hypnosis and mental imagery. She wants to know if you have anything else to offer
her. You explain that there is a medication she can take one hour before her performances that will most

65
likely alleviate her symptoms.
Which of the following conditions is a contraindication for this commonly used medication:

Top of Form

Asthma
Atherosclerotic disease
High blood pressure
Vasovagal syncope
Cirrhosis

Explanation

The correct answer is choice A

Clinicians may forget to ask about general medical conditions when diagnosing psychiatric disorders.
Therefore, this question was assessing your ability to make a psychiatric diagnosis, choose the first-line
medication, and assess for contraindications before prescribing.

In this patient, the correct diagnosis would be social phobia. The first-line treatment for a social phobia, as
seen in the above patient, would be a beta blocker given one hour before the performance. Although the
newer beta blockers are cardioselective, meaning they have more beta 1 activity (found mainly in the
heart) than beta 2 activity (found in the lungs, as well as other organs), asthma is still an important
contraindication to be aware of, making choice a, asthma, the correct answer. Some other
contraindications for beta blockers would be bradycardia, heart failure, AV blocks, and pheochromocytoma.

Atherosclerotic disease is not a concern with beta blockers, it may even be alleviated by it, as some studies
suggest using beta blockers for intermittent claudication.

High blood pressure would actually be alleviated with maintenance therapy of beta blockers.

Beta blockers may occasionally be used to treat vasovagal syncope and are therefore not contraindicated.

Cirrhosis is not a problem since beta blockers are almost exclusively renally metabolized. However, this
may have been our answer choice if you chose a benzodiazepine in your treatment plan, as they are
hepatically metabolized. Due to their abuse potential among other concerns, benzodiazepines would be a
second-line treatment for the above patient.

A 61 year old man presents at your clinic on account of abdominal pain which he has been experiencing
for the last 2 days. He also reports diarrhea and nausea. His medical history is unremarkable for any

66
major illness, though his family history is significant for hyperlipidemia. He takes no drugs and has no
known drug allergies and reports no major changes in his diet. His exercise routine consists of watching
sporting events from his living room. He has suffered no trauma, nor has he experienced similar
symptoms in the past.

On physical exam, you find him to be overweight with a BMI of 28. His blood pressure is 132/89, his pulse
88 beats/minute, and his temperature is elevated at 38.4 degrees C (normal 36.5-37.5). There is mild
abdominal distention, reduced bowel sounds, abdominal tenderness in the left lower quadrant. There is
slight guarding, but no rebound tenderness. A CBC reveals no anemia, but the white blood cell count is
elevated at 12,000 (normal 4,100-10,500). Abdominal CT confirms a diagnosis of diverticulitis.

All of the following management options are correct for this patient, except:

Top of Form

a liquid diet
amoxicillin/clavulanate
avoidance of popcorn, corn, nuts, and seeds
metronidazole
surgery, only if he experiences another episode

Explanation

The correct answer is choice C.

Abdominal pain with tenderness in the left lower quadrant, nausea, fever, and an elevated white blood cell
count suggests diverticulitis, the most severe manifestation of diverticular disease. Diverticular disease
occurs almost entirely in developed countries and rare in developing countries and in Asia even in
developed countries. Incidence is very low in people under 40 years of age but rises sharply thereafter
such that as much as 60 percent of the population of the US will develop it by age 80. Increasing incidence
has been noted in people under age 50 with the main risk factors being elevated body mass index (BMI),
physical inactivity, and low dietary fiber. Most commonly, the condition involves the left side of the colon,
with most patients affected in the descending and sigmoid colon.

The etiology of diverticular disease is thought to involve a defect in the wall of the colon resulting from
elevated intraluminal pressure. This happens in association with diets low in fiber and high in fat that are
common in Western countries and results in stools that are less bulky. Nevertheless, in 80 percent of those
with the defect, diverticulosis is asymptomatic. In the other 20 percent, the condition manifests either as
symptomatic diverticulosis, or diverticulitis. The former condition features changes in bowel patterns,
episodic pain, but no inflammation, and thus may be diagnosed as irritable bower syndrome. It also can
mimic diverticulitis if there is melena or hematochezia, in which case it has to be differentiated from colon
carcinoma and other causes of lower gastrointestinal bleeding.

Appropriate treatment now that the diagnosis is confirmed is a clear liquid diet (choice A) and 7-10 days of
oral antibiotics, broad-spectrum so that aerobes are covered, such as Streptococcus, Enterobacter, E. coli,

67
Klebsiella, and Proteus, and anaerobes such as Claustridium, peptostreptococcus, and Bacteroides fragilis.
Amoxicillin/clavulanate (choice B) is one possibility, metronidazole (choice D) is another, generally in
combination with trimethoprim-sulfamethoxazole or ciprofloxacin.

Following an initial episode of diverticulitis approximately 1 in 4 cases treated medically will suffer an
additional episode and each time this happens the risk of yet another episode increases. Thus, for quite a
while, for patients younger than 50 years of age, various surgical procedures (which have evolved over the
years) were recommended following just one episode for younger patients, and following two episodes of
diverticulitis for older patients (choice E). These recommendations were grounded in studies suggesting
that the condition was more severe in young patients who also were at greater risk on account of the
longer lifespan. Subsequent studies, however, have demonstrated that the relationship between risk, age,
and other factors is more nuanced. Consequently, the benefit surgery varies from patient to patient.

Once the diverticulitis resolves after the liquid diet, a high fiber diet is recommended for all individuals with
diverticular disease. For many years, patients also were advised to avoid popcorn, corn, nuts, and seeds
(choice C), but by the early 21st century studies had demonstrated that such foods do not actually lead to
complications. Thus, patients no longer are advised to avoid them.

A previously healthy 17 year old girl is brought to the physician for evaluation because of loss of appetite,
sleeplessness, and extreme irritability for 3 weeks. After missing many practices, she quit the softball
team that she previously enjoyed. She often feels tired and has difficulty sitting still and concentrating on
school work. Her menses occur at regular intervals. She weighs 50kg (110lb) and is 168 cm (66in) tall.
Her blood pressure is 110/70 mm Hg, pulse is 74/min and respirations are 16/min.

Which of the following is the most likely diagnosis?

adjustment disorder with mixed disturbance of emotion


and conduct
anorexia nervosa
attention-deficit / hyperactivity disorder
dysthymic disorder
major depressive disorder

Explanation

The correct answer is Choice E

The patient in this example demonstrates classic major depression. Sufferers of major depressive disorder
experience “depressed mood most of the day” and “5 (or more) of the following (depressive) symptoms
during the same 2-week period”: poor appetite or overeating; insomnia or hypersomnia; low energy or
fatigue; low self-esteem; poor concentration or difficulty making decisions; or feelings of hopelessness”.

Adjustment disorder with mixed disturbance of conduct and emotion (Choice A) is incorrect because the
diagnosis of adjustment disorder requires that the patient develop “emotional or behavioral symptoms in
response to an identifiable stressor(s) occurring within 3 months of the onset of the stressor(s)” and that
the “symptoms or behaviors are clinically significant as evidenced by either of the following: (1) marked

68
distress that is in excess of what would be expected from exposure to the stressor or (2) significant
impairment in social or occupational (academic) functioning." Not only did the patient in the example not
have any known stressor, she has demonstrated no conduct irregularities such as skipping school,
violence, or behaviors that resulted in detention by the police.

Anorexia nervosa (Choice B) is incorrect because the patient has demonstrated no significant weight loss
and evidenced no body dysmorphism (in this instance she would need to perceive herself to be overweight
when her weight was normal and acceptable). Her appetite concern is explained by the fact that appetite
disturbance is an almost universal finding in major depression.

Attention deficit hyperactivity disorder (Choice C) is incorrect because the patient in the example has not
demonstrated any of the hyperactivity, inattentiveness or impulsivity symptoms required of this diagnosis
(ADHD) and has no documented prior history of this condition. ADHD patients usually manifest their
symptoms relatively early in childhood and it would be odd for new onset at age 17.

Dysthymic disorder (Choice D) is incorrect because dysthymia is a chronic low-level depressive condition
that requires the patient experience “depressed mood most of the day, for more days than not, as
indicated either by subjective account or observation by others, for at least 2 years” and experience only 2
or more of the following symptoms during that 2-year time period: poor appetite or overeating; insomnia
or hypersomnia; low energy or fatigue; low self-esteem; poor concentration or difficulty making decisions;
or feelings of hopelessness”. The recent symptom onset eliminates this diagnosis as a possibility.

A patient is having recurrent flashbacks after experiencing an ambush while serving in the military. He
presents with difficulty concentrating and feelings of detachment. He denies wanting to hurt himself or
others.

Which one of the following pharmacologic treatments is most appropriate for this patient?

An intramuscular antipsychotic
Long-term benzodiazepine therapy
Selective serotonin reuptake inhibitor
Prazosin
Monoamine oxidase inhibitors

Explanation

The correct answer is choice C

Post-traumatic Stress Disorder (PTSD) is a commonly seen concern in both the military and civilian
settings. It occurs after a person is exposed to a traumatic event where the person either (1) experienced,
witnessed, or was confronted with an event or events that involved actual or threatened death or serious
injury, or a threat to the physical integrity of self or others or (2) his/her experience involved intense fear,
helplessness, or horror.

In addition, PTSD patients often develop concerns involving re-experiencing the trauma, such as
nightmares; increased arousal symptoms such as irritability, restless, and sleeplessness that were not
present prior to the trauma; and many will develop persistent avoidance of stimuli associated with the

69
trauma and numbing of general responsiveness.

Studies demonstrate that selective serotonin reuptake inhibitors (SSRIs) help reduce PTSD symptoms. The
current recommendation for achieving maximal success in the treatment of PTSD is appropriate
pharmacology (SSRI) plus targeted psychotherapy, such as cognitive behavioral therapy (CBT).

An intramuscular antipsychotic (Choice A) is not the correct answer because intramuscular antipsychotics
are essentially used only for persons with chronic psychotic illnesses such as schizophrenia and manic-
depressive illness. Our soldier demonstrates no symptoms of psychosis. These products’ lengthy duration
of action benefits patients with reliability and compliance concerns. Recently, atypical antipsychotics have
been used for patients who do not respond to antidepressants. Patients with severe PTSD symptoms are
likely to need longer treatment to experience beneficial results of taking these medications.

Instituting long term benzodiazepine therapy (Choice B) is incorrect because one sound principal of
prescribing is that physicians should always avoid prescribing drugs with dependence concerns if other
reasonable options are available. All benzodiazepines, while they do have a role in the management of
many conditions, can create tolerance and addiction concerns for patients who take them on a regular
basis. That being said, benzodiazepine use on as-needed basis can be very helpful for some PTSD patients
with significant anxiety issues.

Prazosin (Choice D) is the not the best answer for this question because, while this drug that was created
as an antihypertensive (brand name Minipress) is utilized in the management of PTSD. It is used in the
management of nightmares associated with PTSD in children mainly—a concern this soldier has not voiced.

Monoamine oxidase inhibitors (phenelzine) (Choice E) have been involved in a few controlled medication
trials for PTSD. In one trial, phenelzine significantly reduced intrusion (reexperiencing) symptoms but had
no effect on avoidance. Usually reserved for patients who do not tolerate or respond to traditional cyclic or
second-generation antidepressants.

A 45 year old man is brought to the physician by his spouse. He has been drinking heavily since he was
passed over for a job promotion 3 days ago. He stayed in bed over the weekend. He has no personal
history of psychiatric disorders and no personal or family history of alcohol abuse. He is crying and states,
“I can’t believe it” when addressed. When asked what he will do he states, “I don’t know, but if I don’t go
back to work tomorrow, I’ll lose my job.”

Which of the following is the most likely diagnosis?

dysthymic disorder
major depressive disorder
alcohol dependence
adjustment disorder with depressed mood
bipolar disorder

Explanation

Choice A is the correct answer D.

Adjustment disorder (Choice D) is the correct answer because the diagnosis of adjustment disorder

70
requires that the patient develop “emotional or behavioral symptoms in response to an identifiable
stressor(s) occurring within 3 months of the onset of the stressor(s)” and that the “symptoms or behaviors
are clinically significant as evidenced by either of the following: (1) marked distress that is in excess of
what would be expected from exposure to the stressor or (2) significant impairment in social or
occupational (academic) functioning."

Dysthymic disorder (Choice A) is incorrect because dysthymia is a chronic low-level depressive condition
that requires the patient experience “depressed mood most of the day, for more days than not, as
indicated either by subjective account or observation by others, for at least 2 years” and experience "2 or
more of the following symptoms during that 2-year time period: poor appetite or overeating; insomnia or
hypersomnia; low energy or fatigue; low self-esteem; poor concentration or difficulty making decisions; or
feelings of hopelessness”. Both the short term of the patient’s symptoms and the recent severe stressor
eliminate this diagnosis as a possibility.

Major depressive disorder (Choice B) is also incorrect due to this diagnosis’s unique time and number of
symptom requirements. In order to fulfill diagnostic criteria for major depression patients must experience
“depressed mood most of the day” and “5 (or more) of the following (depressive) symptoms during the
same 2-week period: poor appetite or overeating; insomnia or hypersomnia; low energy or fatigue; low
self-esteem; poor concentration or difficulty making decisions; or feelings of hopelessness”. Again, our
patient’s 3 day symptom time period alone eliminates this diagnosis as a correct answer.

Alcohol dependence (Choice C) is incorrect because the patient’s history reveals no evidence of excessive
use of alcohol or problems related to alcohol use and he has only been drinking alcohol heavily for the past
3 days in response to the stressor. His alcohol-related behaviors would be classified as a dysfunctional
response to the stressor and the heavy drinking as alcohol abuse. Below find the diagnostic criteria from
the Diagnostic and Statistical Manual of Mental Disorders (4th ed.) which states that alcohol dependence is
defined as "a maladaptive pattern of alcohol use, leading to clinically significant impairment or distress, as
manifested by three or more of the following seven criteria, occurring at any time in the same 12-month
period:

 Tolerance, as defined by either of the following:

1. A need for markedly increased amounts of alcohol to achieve intoxication or desired effect.
2. Markedly diminished effect with continued use of the same amount of alcohol.

 Withdrawal, as defined by either of the following:

1. The characteristic withdrawal syndrome for alcohol (refer to DSM-IV for further details).
2. Alcohol is taken to relieve or avoid withdrawal symptoms.

 Alcohol is often taken in larger amounts or over a longer period than was intended.
 There is a persistent desire or there are unsuccessful efforts to cut down or control alcohol use.
 A great deal of time is spent in activities necessary to obtain alcohol, use alcohol or recover from
its effects.
 Important social, occupational, or recreational activities are given up or reduced because of
alcohol use.
 Alcohol use is continued despite knowledge of having a persistent or recurrent physical or
psychological problem that is likely to have been caused or exacerbated by the alcohol (e.g.,
continued drinking despite recognition that an ulcer was made worse by alcohol consumption). "

Bipolar disorder (Choice E) is incorrect as bipolar disorder is a chronic disabling affective disorder
characterized by vascillations between periods of elevated mood, known as mania and low mood, known as
depression. The diagnosis of bipolar disorder is not possible in this patient because there is no evidence of
a manic episode in his history or in his current complaints.

To review, diagnosis of mania requires the patient experience “a distinct period of abnormally and
persistently elevated, expansive, or irritable mood, lasting at least 1 week (or any duration if
hospitalization is necessary) and that during this period of mood disturbance, 3 or more of the following

71
symptoms have persisted (4 if the mood is only irritable) and have been present to a significant degree:
inflated self-esteem or grandiosity; decreased need for sleep (e.g., feels rested after only 3 hours of
sleep); more talkative than usual or pressure to keep talking; insomnia or hypersomnia nearly every day;
psychomotor agitation or retardation nearly every day (observable by others, not merely subjective
feelings of restlessness or being slowed down); flight of ideas or subjective experience that thoughts are
racing; distractibility (i.e., attention too easily drawn to unimportant or irrelevant external stimuli);
increase in goal-directed activity (either socially, at work or school, or sexually) or psychomotor agitation;
or excessive involvement in pleasurable activities that have a high potential for painful consequences (e.g.,
engaging in unrestrained buying sprees, sexual indiscretions, or foolish business investments).”

Two days after being admitted to the hospital for alcohol dependence and alcohol withdrawal syndrome as
well as other findings including alcoholic hepatitis, the patient in the above scenario is heard screaming
that his room is full of giant bugs and that they have come to take him away. When you arrive at his room
you see the terrified man trying to hide behind the door. He is shaking, screaming and gesticulating wildly
at the walls and ceiling.

What is his diagnosis and what medical risks does it entail?

Delusional Parasitosis: essentially no risk except for skin injury from scratching
New-Onset Paranoid Schizophrenia: very dangerous because paranoid schizophrenics commit
a great number of crimes, including murder
Mania: no great danger because if you provide him with 40 mg of Haldol he will become
sedated and when he wakes up the bugs will be gone
Delirium Tremens: carries up to a 35% mortality rate if untreated
Panic Disorder: increased risk of myocardial infarction

Explanation

The correct answer is Choice D

Delirium tremens (DTs) is a severe potential complication of alcohol dependence and alcohol withdrawal syndrome.
Manifestations include hallucinations, tremors, hypertension, tachycardia, seizures, hyperthermia, confusion, and
seizures. Prior delirium tremens episodes increase the risk for the development and severity of future DT episodes, a
phenomenon known as kindling. Aggressive consistent alcohol withdrawal tapers both help improve the patient’s
comfort during the acute stage of his alcohol abstinence and help reduce the likelihood of DT development.

Delusional Parasitosis: essentially no risk except for skin injury from scratching (Choice A) is not correct. Delusional
Parasitosis is a mono-delusional state wherein otherwise mentally stable persons develop the belief that their body is
infested with parasites. These patients cannot be shaken from this belief regardless of the number of physicians who
testify that that their laboratory and physical examinations demonstrate no evidence of parasitic infestation. The only
danger that this condition presents to this patient would be were they to submit to dangerous or ill-advised treatment
from an unscrupulous medical provider.

New-Onset Paranoid Schizophrenia: very dangerous because paranoid schizophrenics commit a great number of
crimes, including murder (Choice B) is very unlikely to be correct because most paranoid schizophrenics present with
their first psychotic break at a young age. Also, the context of the alcohol with this patient indicates that the substance
abuse issue must be addressed before other definitive diagnoses can be made. The second part of this choice is also
incorrect, as most schizophrenics, including paranoid schizophrenics, pose little danger to society. Bipolar affective
disorder patients who are in a manic phase—especially those who are experiencing command hallucinations—pose the
greatest danger to the public.

72
Mania: no great danger because if you provide him with 40 mg of Haldol he will become sedated and when he wakes
up the bugs will be gone (Choice C) is also not correct. Although persons experiencing mania often experience
hallucinations, the fact that this patient entered the facility experiencing alcohol withdrawal syndrome and is being
treated for this issue makes it clear that this episode of confusion and psychosis is most consistent with delirium
tremens. In addition, this person has no documented history of bipolar disorder and it would be extremely unlikely
that a person of his age would have his initial psychotic break at this age. That being said, antipsychotic therapy is
often employed in persons experiencing significant psychotic symptoms during alcohol withdrawal syndrome episodes.

Panic Disorder: increased risk of myocardial infarction (Choice E) is incorrect both because the symptoms
demonstrated by the patient in our example or not consistent with panic disorder and also because panic disorder
poses no lethality threat from a physical perspective. Panic attacks are a rapidly developing episode of anxiety that
may be characterized by chest pain and pressure, sense of dyspnea, tachycardia, heart palpitation, tingling and
numbness of the face and extremities, weakness, dizziness and may progress to a sense of depersonalization.
Episodes are usually brief and resolve without treatment, but may last as long as 10-15 minutes.

Figure 1: Diagnostic criteria for alcohol withdrawal

Figure 2: Stages of alcohol withdrawal

73
A 50-year-old divorced woman presents for psychiatric evaluation in January 2009. She complains of
depressed mood since October 2008, becoming progressively worse. In addition, she complains of
tearfulness, impaired sleep, impaired concentration, anxiety, overeating with weight gain, anhedonia, and
decreased libido. She adds that she has experienced similar symptoms each year from October through
May over the last four years since relocating to the Midwestern United States. Prior to 5 years ago, the
patient resided in the southwestern United States and was asymptomatic.

Which of the following best describes this patient's DSM-IV diagnosis?

Major depressive disorder (single episode, severe)


Adjustment disorder with depressed mood (chronic)
Major depressive disorder, (recurrent, severe, with seasonal
pattern)
Bipolar II disorder, (depressed, severe, with seasonal pattern)
Posttraumatic stress disorder, (chronic, with delayed onset)

Explanation

The correct answer is Choice C.

The litany of major depressive symptoms clearly identifies depression as the prominent feature of this
case. If you did not read all of the answer choices, you may have chosen A, so it this question is a good
reminder to read all of the answer choices even if you think you know the right answer.

A. The problem with this answer choice is the specifier "single episode", when half of the question stem
explains the recurring pattern.

B. Recall that adjustment disorder can be "proven" if, when you remove the stressor (in this case,
potentially the move to the Midwest) the symptoms go away. Her pattern is that the symptoms resolve
while she remains in the Midwest, so that factor isn't causing her symptoms directly. Not enough

74
information is given about what she left behind in the Southwest and any troubling memories or feelings of
loss that recur in the winder months.

D. There is no evidence of mania, so this question is an easy one to eliminate.

E. There is no evidence of trauma. On the USMLE, they would not require you to infer things like that. The
exception would be some vague hints of child abuse or neglect. In that circumstance you are to be
hypervigilant, but the question-writing process requires some tangible evidence in the question stem.

A 32 year old man comes to the physician because of a long-standing belief that he is being followed by
foreign intelligence agents. He has not had hallucinations of any type. On mental status examination, no
formal thought disorder or change in mood is detected. His level of functioning at work has remained high.

Which of the following is the most likely diagnosis?

obsessive compulsive disorder


panic disorder
schizophrenia paranoid type
schizotypal personality disorder
delusional disorder, persecutory type

Explanation

The correct answer is Choice E

Delusions are fixed false beliefs from which a person cannot be dissuaded. Persons suffering from
delusional disorder, persecutory type demonstrate at least 1 non-bizarre delusion but can otherwise appear
and be mentally normal and maintain high social and vocational functioning. In contrast, persons with
paranoid schizophrenia often possess bizarre delusions and tend to develop social and vocational problems
as a result of their behavior.

Obsessive compulsive disorder (Choice A) is incorrect because persons with obsessive compulsive disorder
(OCD) are not delusional. While their obsessions and compulsions may seem odd, persons suffering from
OCD have insight into their condition.

Panic disorder (Choice B) is incorrect because persons with panic disorder do not experience delusions.
Persons with panic disorder experience a discrete period of intense fear or discomfort that develops
abruptly, usually reaches a peak within 10 minutes, occurs in the absence of real danger, and is
accompanied by at least 4 of the following symptoms: palpitations, pounding heart, or accelerated heart
rate; sweating; trembling or shaking; sensation of shortness of breath or smothering; feeling of choking;
chest pain or discomfort; nausea or abdominal distress; feeling dizzy, unsteady, lightheaded, or faint
derealization (feelings of unreality) or depersonalization (being detached from oneself); fear of losing
control or going crazy; fear of dying; paresthesias (unexplained numbness or tingling sensations) and chills
or hot flushes.

Schizophrenia, paranoid type (Choice C) is incorrect as per the discussion related to Choice A. Additional
symptoms that may be experienced by paranoid schizophrenia sufferers include auditory or visual
hallucinations, anxiety, anger, aloofness, verbal confrontations and increased risk for suicidal thoughts and
behavior

Schizotypal personality disorder (Choice D) is not correct because persons with this psychiatric disorder--
while they may experience very strange thinking-- tend not to experience delusions; their greatest concern
is usually difficulty with interpersonal relationships and disturbances in thought patterns, appearance, and

75
behavior. The severity of the behavior disturbance ranges from mild to severe, but those with marked
disturbances may resemble schizophrenics, making maintaining vocational and social relationships
problematic.

A 15 year-old girl presents to the Emergency Department, brought by her parents because she has been
screaming and fighting with her parents all afternoon. At the admissions desk, she threatened to strike
one of the nurses and refused to answer any questions. Presently, she is sitting before you more calmly,
but has guarded, closed body language. When you calmly ask her questions, you find that she is alert and
oriented to time, person, and place.

She denies any substance abuse, hallucinations, suicidal or homicidal ideation, and does not exhibit any
bizarre behavior or thought content. She can point to no reason for the arguments she had that day, but
her parents add that she has been getting increasingly hostile over the last 7-10 days. The patient begins
to get agitated. You ask that the parents leave the room and she begins to calm down again. The patient
tells you that there has been no violence, including sexual assault.

A complete blood count, comprehensive chemistry panel, urine toxicology and serum drug screen are all
within normal limits. Her physical exam is only remarkable for bruises and scratches on her knuckles,
which appear to be recent injuries. She tells you that she punched the wall a few days ago.

What is the next best step in the management of this patient?

Place the patient in seclusion and restraints


Administer 10 mg of Haloperidol intramuscularly
Admit the patient to the psychiatry service for short-term evaluation
Administer 5 mg of Haloperidol PO plus 1 mg of lorazepam PO
Explain to the family that the patient has Oppositional Defiant Disorder

Explanation

The correct answer is Choice C.

This patient still becomes agitated and argumentative when the parents are present. Therefore the
argument for admission is to allow a cooling off period, since the parents were unable to achieve this
themselves and soght medical attention. Not enough information is given here to determine the underlying
cause of agitation, which further supports admission.

Approximately 25% of adolescent psychiatric emergencies are because of aggressive behavior. Behavioral
management of the agitated child includes:

 Safety first - the staff and the patient! If this child was actively causing harm, physical and
pharmacologic restraing would be appropriate.
 Speaking and approaching the child in a calm, reassuring manner.
 Asking the family member to stay if their presence is calming or soothing (in this case it wasn't).
 Offering a drink, snack, or toys if available (yes, there are even video games for teenagers to play
in some hospitals).

76
 Provide a time-out room with little stimulation.

There are both psychiatric and nonpsychiatric causes of agitation in children.

NONPSYCHIATRIC CONDITIONS:

 Poisoning or exposure to toxin


 Acute infection
 Delerium
 Meningitis
 Space-occupying lesion
 Seizure
 Acute confusional migraine
 Head trauma

PSYCHIATRIC CONDITIONS:

 Psychosis
 Mania
 Severe anxiety
 Acute stress disorder
 PTSD
 Substances: intoxiation or withdrawal
 ADHD
 Conduct disorder
 Developmental delay

Choice A. Other than threating a nurse, you were able to calm the patient with your demeanor and asking
the parents to leave the room. At that point, there was no acute agitation, no imminent threat to herself or
others. Therefore, restraints are not indicated at this moment. Be aware that agitated patients can be
emotionally volitile so 5 minutes later you may have to put them in restraints. However, without an acute
threat and not understanding the reason for the agitation, be careful not to overreact to hostile behavior.
Many times, especially with adolescents, a calm but firm tone is all that is required. A little respect goes a
long way.

Choice B and D. Haloperidol may be used in acute agitation if the patient is over 16 years old. There are
some physicians that would medicate first and ask questions later, however, the USMLE exam focuses on
the current and immediate presenting symptom. Don't overlook the fact that the patient was sitting calmly
upon your evaluation.

You will not be required to know doses of medications on any of the USMLE exams. 10 mg of Haloperidol
would be the upper limit, within the acceptable dose range in children (0.05 - 0.15 mg/kg), assuming the
patient is 70 kg. However, the patient's weight was not provided. In general, Haloperidol is indicated
psychotic acute agitation and off-label for nonpsychotic agitation. You will not be choosing any off-label
uses of medications on any of the USMLE exams. Besides, the child doesn't actively warrant
pharmaceutical intervention.

Other pharmacologic management concepts you need to be familiar with include the fact that a clinician
requires permission from a parent or guardian to administer nonemergency medication. Acute, violent
agitation is considered an emergency. Use as-needed (PRN) medications or an additional dose of the child's
own standing medication if available. Other medication choices for acute agitation in children include:

1. Diphenhydramine 1.25 mg/kg PO or IM


2. Clonidine 0.05 - 0.1 mg PO depending on the child's age and size
3. Risperidone 0.5-1 mg PO depending on the child's age and size

77
Choice E. This patient's behavior has only occured for 7-10 days. Note the DSM-IV criteria for Oppositional
Defiant Disorder are the following:

 A pattern of negativistic, hostile, and defiant behavior lasting at least 6 months, during which 4 or
more of the following are present:

1. often loses temper


2. often argues with adults
3. often actively defies or refuses to comply with adults' requests or rules
4. often deliberately annoys people
5. often blames others for his or her mistakes or misbehavior
6. is often touchy or easily annoyed by others
7. is often angry and resentful
8. is often spiteful or vindictive

Note: Consider a criterion met only if the behavior occurs more frequently than is typically observed in
individuals of comparible age and developmental level.

 The disturbance in behavior causes clinically significant impairment in social, academic, or


occupational functioning.
 The behaviors do not occur exclusively during the course of a psychotic or mood disorder.
 Criteria are not met for conduct disorder, and if the individual is 18 years or older, criteria are not
met for antisocial personality disorder.

Keep this diagnosis distinct from Conduct Disorder, which focuses more on harm done to others and
societal norms. The DSM-IV criteria for Conduct Disorder are as follows:

 A repetitive and persistent pattern of bahavior in which the basic rights of others or major age-
appropriate societal norms or rules are violated, as manifested by the presence of three (or more)
of the following criteria in the past 12 months, with at least one criterion present in the past 6
months:

1. Agression to people and animals


2. Destruction of property
3. Deceitfulness or theft
4. Serious violations of rules

 The disturbance in behavior causes clinically significant impairment in social, academic, or


occupational functioning.
 If the individual is age 18 years or older, criteria are not met for antisocial personality disorder.

A 15-year-old girl presents to the primary care clinic with her parents. In the office she is withdrawn and
downcast. Her mother is sitting close to her daughter, with her arm around her, and the girl is leaning
into her mother. The parents describe their daughter as extremely tearful, irritable, and discouraged for
the past several months. The girl says that she has "no friends" and that "everyone hates" her. The
parents are concerned about depression.
Looking at the following medications, which one would you suggest trying first in this patient, assuming
she does indeed have major depressive disorder (MDD) requiring pharmacological therapy?

78
Top of Form

Paroxetine
Venlafaxine
Sertraline
Citalopram
Fluoxetine

Explanation

The correct answer is choice E

Major depressive disorder (MDD) is a common psychiatric disorder. To diagnose depression, the patient
must have depressed mood or loss of pleasure or interest in previously enjoyable activities most of the
day, most days of the week, for at least 2 weeks and represent a change in behavior. Finally, the
symptoms must cause significant impairment in functioning to qualify for the diagnosis.

At this time, only Prozac (Fluoxetine) is considered safe in children and adolescents. Although many
physicians use other anti-depressants with few side effects, the risks of suicide attempts and suicidal
ideation have been suggested to increase with the use of other anti-depressants. Both the United States
Food and Drug Administration (FDA) and British Medicines and Healthcare Products Regulatory Agency
have officially concluded that only Prozac has been shown to be safe and effective in children. However,
that does not mean it is without caution that you should prescribe it to minors.

Paroxetine (Paxil) is a typical SSRI. It is used in adults for generalized anxiety disorder, depression, and
social phobia among others. It does have discontinuation effects and should not be stopped suddenly.
Due to this fact, it is the least prescribed to children.

Venlafaxine (Effexor) is an SNRI. This medication acts on both serotonin and norepinephrine receptors,
and also has some dopaminergic effects. This medication has a black box warning for minors (possible
increase in suicidal ideation/attempts), but is occasionally used off-label for childhood depression. This is
usually considered a second-line antidepressant, after a trial with an SSRI has failed.

Sertraline (Zoloft) is a typical SSRI. This medication is a typical, first-line anti-depressant that acts on
serotonin receptors. It also has the black box warning for minors (possible increase in suicidial
ideation/attempts), but is occasionally used off-label. This medication usually does well in females who
have an ansiety component to their depression.

Citalopram (Celexa) is an SNRI. This medication has action on serotonin and norephinephine receptors.
This one also has the black box warning for possible increase in suicidal attempt/ideation and should not be
used in minors. This is usually used after a trial with an SSRI has failed. Patients who did not tolerate
Effexor are often switched to this medication due to its similar effects.

79
An 8-year-old boy presents to the primary care clinic with his parents. His parents complain that their son
has been hostile and disobedient for the past few months. The boy's teacher describes him as "suddenly
shy and immature" after being a normal, happy boy in her class the previous year. The boy is seated
quietly in the chair during the interview and does not make eye contact with his parents. When asked
direct questions, he will raise his head, glance at you, and answer in short, concise phrases.
Which of the following are you MOST concerned about in this child:

Top of Form

Mood Disorder
Age-appropriate behavior
Oppositional Defiance Disorder
Child Abuse
Hearing Loss

Explanation

The correct answer is choice D

The most important thing to notice in this vignette is the disparity between the parent's comments, the
teacher's comments, and the boy's behavior in your office. This disparity tells you that it is probably not
oppositional defiance disorder (ODD). Also, when these types of children are in your office, they rarely are
able to sit quietly. The comment regarding eye contact is important to note. This is often seen in abused
children. These types of non-verbal cues are important to notice when dealing with children. They often do
not know how to express their feelings; especially with parents present who will usually do most of the
talking for them.

This question also could have asked you "what information would you want to elicit next?" In this situation,
you could look for changes in the child's life--parental conflict, arguing, relative moving into the home,
etcetera. These things would point you towards possible abuse or help explain the sudden change in
behavior. Remember, it is very important to rule out child abuse before moving on to diagnosing other
disorders or prescribing medications.

To diagnose a mood disorder in this child, you would want to take a more detailed family history. For a
mood disorder to show up in a child this young, there is often a first-degree relative with a mood disorder
as well. While a mood disorder would be on your differential, it would be important to rule out other
causes of the problem behavior that would be more likely in an 8 year-old child.

One should always be hesitant to tell parents that their concerns regarding their child are simply "age-
appropriate behavior". If there is a sudden change in behavior, even if it is still on the spectrum of
"appropriate", it is worthy of concern. Always take changes in behavior seriously lest you miss something

80
serious, such as abuse.

To diagnose ODD, you must notice a recurrent pattern of disobedience and defiance towards authority for
at least six months that causes significant social or academic impairment. The child will exhibit excessive
"anger, vindictiveness, arguing with and disobeying adults, loss of temper, annoying others, and blaming
others for poor behavior and mistakes". While the boy's parents may hint at this kind of behavior, it is
important to notice the teacher's focus on other traits. In the case of ODD, both parents and teachers
would be concerned about the overly angry and defiant disposition of the child. That is not the case in this
question. However, it could still be further down on the differential.

Hearing loss in children can often be diagnosed as a learning disability or a disobedient child. While this is
something to consider in this child, child abuse would be more important to rule out with further
questioning and possibly interviewing the child without the parents present. Also, hearing exams are often
done annually at most public schools.

Although not an answer choice, CD should have been on your differential. To diagnose CD, you must notice
a repetitive and persistent pattern of behavior where the child intentionally ignores the rights of others or
basic age-appropriate societal norms. These behaviors can be categorized into four groups: destruction of
property, theft and/or deceitfulness, serious violations of rules, and aggression directed towards animals
and/or people.

Finally, although not an answer choice, ADHD would have also been on your differential. To diagnose
ADHD, you must notice six signs of hyperactivity/impulsivity and six signs of inattention. Although the boy
may have some signs of ADHD, and this would most definitely be in your differential diagnosis, abuse is
the better choice for this presentation. Remember, if child abuse is suspected, it should always be at the
top of your list due to its importance.

Some warning signs for abuse in children to look for: overly withdrawn, anxious or scared of getting into
trouble; decreased or no eye contact; extremes in behavior (such as extreme passivity/compliance or
aggression/hostility); unattached to parents or caregiver(s); acts overly mature (taking responsibility for
other children) or immature (thumb-sucking or throwing tantrums); always watchful, as if waiting for
something bad to happen; dislikes being touched, flinches easily or dislikes going home.

A 30-year old unmarried woman from a poor family becomes convinced that a rich boy staying in her
neighborhood is deeply in love with her. The boy clearly denies any such thing. Still, the young woman
insists that his denial is a secret affirmation of his love towards her. She makes desperate attempts to
meet the boy despite resistance from her family. She also develops sadness at times when her effort to
meet the boy does not materialize. She is able to maintain her daily routine; however, she remains very
preoccupied with the thoughts of this boy.

She is most likely suffering from which of the following conditions:

81
Mania
Depression
Delusional disorder
Schizophrenia
Paraphilia

Explanation

The correct answer is choice C

The definition of delusional disorder is a mental condition characterized by the presence of persistent, non-
bizarre delusions. Delusions are fixed false beliefs from which one cannot be shaken. Delusions can either
be non-bizarre, meaning that despite the fact that whatever the person mistakenly believes it is a thought
that is theoretically plausible in other circumstances; or they can be bizarre, meaning that there is no way
that such a thought could ever be valid. The unfortunate woman in our example is demonstrating a non-
bizarre delusion because, while it is untrue that THE wealthy young man down the street has no interest in
her, it is plausible that A man down the street could develop an interest in her. Conversely, were she to
believe that the young man down the street was pregnant with 25 baby mountain lions and that he
planned to name them all after her because of his devotion to her, that would obviously not be possible
because (1) men don’t get pregnant and (2) pregnant humans don’t give birth to mountain lions. Bizarre
delusions are more likely to be seen in persons suffering from mania or schizophrenia.

Mania (Choice A) is incorrect because while manics often do experience delusions, their delusions are often
bizarre, which hers is not. In addition, she is also not demonstrating the other diagnostic criteria required
for a definitive diagnosis of mania, which are a distinct period of abnormally and persistently elevated,
expansive, or irritable mood, lasting at least 1 week (or any duration if hospitalization is necessary) and
that during this period of mood disturbance, 3 or more of the following symptoms have persisted (4 if the
mood is only irritable) and have been present to a significant degree: inflated self-esteem or grandiosity;
decreased need for sleep; more talkative than usual or pressure to keep talking; insomnia or hypersomnia
nearly every day; psychomotor agitation or retardation nearly every day (observable by others, not merely
subjective feelings of restlessness or being slowed down); flight of ideas or subjective experience that
thoughts are racing; distractibility; increase in goal-directed activity (either socially, at work or school, or
sexually) or psychomotor agitation; or excessive involvement in pleasurable activities that have a high
potential for painful consequences (e.g., engaging in unrestrained buying sprees, sexual indiscretions, or
foolish business investments). One final disqualifier for mania is that she is not experiencing any significant
life impairment as a result of her symptoms. Mania categorically impairs the life of both the patient and the
people in their life.

Depression (Choice B) is not correct because even though the woman has experienced some down
moments related to her delusional relationship situation, she does not qualify for the diagnostic criteria for
depression, which are that the during the same 2 week period the patient must (1) experience depressed
mood most of the day and (2) suffer from 5 or more of the following symptoms: poor appetite or
overeating that has led to significant weight change; insomnia or hypersomnia; poor concentration or
difficulty making decisions; or feelings of hopelessness; markedly diminished interest or pleasure in all, or
almost all, activities most of the day, nearly every day; psychomotor agitation or retardation nearly every
day; fatigue or loss of energy nearly every day; feelings of worthlessness or excessive or inappropriate
guilt (which may be delusional) nearly every day (not merely self-reproach or guilt about being sick);

82
recurrent thoughts of death (not just fear of dying), recurrent suicidal ideation without a specific plan, or a
suicide attempt or a specific plan for committing suicide.

Schizophenia (Choice D) is not a likely diagnosis for this woman because her delusion is paranoid as one
would see with a paranoid schizophrenic, and she evidences none of the symptomotolgy consistent with
the other schizophrenia subtypes as her delusions are not bizarre and she demonstrates no social
deterioration or withdrawal.

Paraphilia (Choice E) is incorrect because the paraphilias are a group of sexual disorders and sexual
perversion whose sufferers experience extreme arousal to deviant stimuli. Examples of paraphilias include
pedophilia, exhibitionism, frotteurism (rubbing against strangers), fetishism (desire for inanimate or
bizarre objects) sadism, sadomasochism, voyeurism, etc. While the young lady in our example may be
confused and psychotic, she is not demonstrating a paraphilia, as a female desiring a male is not
considered deviant behavior in our society.

An 8-year-old boy presents to the primary care clinic with his parents. They are concerned that he may
need medication. His teacher describes him as "excessively active and restless" and says that "he
disrupts other students and shows low tolerance for frustration in school work". His parents describe him
as "easily distracted, except when playing video games".

On the basis of the most probable diagnosis, which of the following drugs may appropriately be tried?

Carbamazepine
Haloperidal
Pimozide
Ethosuximide
Atomoxetine

Explanation

The correct answer is choice E

The most likely diagnosis is attention deficit hyperactivity disorder (ADHD). ADHD affects about 3-5% of
children and is diagnosed in 2-16% of school-aged children. It is considered a life-long disorder, with
approximately 60% of those diagnosed in childhood continuing to have symptoms as adults. It is more
commonly diagnosed in boys; however, there is some debate over whether this is due to the bias seen in
teacher referrals and the physical manifestations present in boys. ADHD often occurs with other disorders:
it is seen with oppositional defiance disorder (ODD) 35% of the time, conduct disoder (CD) 26% of the
time, and bipolar disorder 25% of the time. WIth these comorbidities, it is easy to misdiagnose
ADHD. Care must be taken to analyze each symptom with each disorder to find the one (or more than
one) that best fits the child's behavioral profile.

To diagnose ADHD, one must notice, before the age of 7, six signs of hyperactivity/impulsivity (fidgeting,
excessive talking, inappropriate running) and six signs of inattention (distractability, forgetfulness). When
diagnosed after the age of 7, a thorough interview will often reveal the signs/symptoms were present
much earlier.

Of the medications listed, atomoxetine (Strattera) is the only one used to treat ADHD. Other popular
medications include: Ritalin (methylphenidate), Adderall (amphetamine and dextroamphetamine), Vyvanse

83
(lisdexamfetamine), and Focalin (dexmethylphenidate). Strattera is a second-line, non-stimulant
medication that is used to treat ADHD. Full effects may take up to a week to notice, unlike the stimulant
medications which can be felt the first day of administration. Strattera is an excellent choice for children
who cannot tolerate the stimulants or who did not respond to well to the other first-line medications.

Carbamazepine (Tegretol) is used in the treatment of certain seizures.

Haloperidol (Haldol) is an anti-psychotic used for the management of acute psychosis or agitation.

Pimozide (Orap) is used in the treatment of motor/verbal tics.

Ethosuximide (Zarontin) is used in the treatment of absence seizures.

A 6-year-old boy presents with his mother to the pediatrician. She reports that nearly every day for the
past 2 years her son has displayed episodes of sudden, rapid, nonrhythmic eye blinking during times of
heightened anxiety. She also states that over the past several months her son has been making frequent
repetitive nonrhythmic facial grimaces. The child is not bothered by the behavior, but the mother is
concerned.

What is this patient’s most likely diagnosis?

Transient tic disorder.


Vocal tic disorder.

Chronic motor tic disorder.

Gilles de la Tourette’s syndrome, also known as Tourette's disorder.

Tic disorder not otherwise specified.

Explanation

The correct answer is Choice C.

While this case sounds very much like Tourette's Disorder, vocal tics are required for Tourettee's.
Therefore, the diagnosis is Chronic Motor Tic Disorder, which is defined as the presence of either motor tics
or vocal tics but not both. The tics occur many times nearly every day or intermittently throughout a period
of more than one year. During this period there was never a tic-free period of more than three consecutive
months. The onset must occur before age 18 and the disturbance can not be due to a medical condition.

As a review, tics are defined as rapid and repetitive muscle contractions resulting in movements or
vocalizations that are experienced as involuntary. The various diagnoses are mutually exclusive. That is, a
patient will best fit into only one diagnosis.

Choice A. Transient Tic Disorder is defined as the presence of a single tic or multiple motor or vocal tics or
both. The tics occur many times a day for at least 4 weeks, but no longer than 12 months.

84
Choice B. Vocal Motor Tic Disorder and Chronic Motor Tic Disorder differ only by the specific muscles
affected. If there is only a vocal component, then the patient is diagnosed with Vocal Motor Tic Disorder.
The other criteria are the same as Chronic Motor Tic Disorder.

Choice D. Goerges Gilles de la Tourette first described a patient with what was later named Tourette's
Disorder in 1885. Both multiple motor and one or more vocal tics have been present at some time during
the illness, although not necessarily concurrent. The other criteria match Chronic Motor Tic Disorder.

Choice E.Tic Disoder Not Otherwise Specified is a residual diagnosis for patients that meet none of the
above diagnostic criteria. Perhaps patients begin symptoms after age 18 or the symptoms last less than a
year.

A 38-year old married woman had complained of nervousness since childhood. She also said she was
sickly since her youth, with a succession of physical problems doctors often indicated were caused by her
nerves or depression. She, however, believed that she had a physical problem that had not yet been
discovered by the doctors. Besides nervousness, she had chest pain and had been told by a variety of
medical consultants that she had a nervous heart. She also consulted doctors for abdominal pain and had
been told she had a spastic colon. She had seen chiropractors and osteopaths for backaches, for pains in
her extremities, and for anesthesia of her fingertips.

Three months previously, she was vomiting and had chest pain and abdominal pain, and she was admitted
to a hospital for a hysterectomy. Since the hysterectomy, she had had repeated anxiety attacks, fainting
spells that she claimed were associated with unconsciousness that lasted more than 30 minutes, vomiting,
food intolerance, weakness, and fatigue. She had been hospitalized several times for medical workups for
vomiting, colitis, vomiting of blood, and chest pain. She had had a surgical procedure for an abscess of
the throat. She said she felt depressed, but thought that it was all because her "hormones were not
straightened out." She was still looking for a medical explanation for her physical and psychological
problems.

What is the most likely diagnosis?

Somatization disorder

Conversion disorder

Hypochondriasis
Dysthymic disorder

Pain disorder

Explanation

The correct answer is Choice A.

Somatization disorder presents with symptoms involving multiple organ systems with no apparent
physical/medical cause. There is Seven-Symptom Screeding test for Somatization Disorder:

"Somatization Disorder Besets Ladies And Vexes Physicians" =

85
 Shortness of breath
 Dysmenorrhea
 Burning in sexual organs
 Lump in throat
 Amnesia
 Vomiting
 Painful extremities

B. Conversion disorder usually presents with neurological complaints, such as perceived paralysis with no
underlying medical/physical cause. The title reflects a hypothesis that a patient's somatic symptoms are
due to unconscious psychological conflict. Roughtly 25% of these patients will have a second episode
within one year.

C. Hypochondriasis is the preoccupation with or the fear of having a serious medical disease based on a
person's misinterpretation of bodily sensations and/or functions. Do not be dismissive of their concerns,
but encourage psychiatric care along with medical care.

D. Dysthymic disorder involves a depressed mood most of the day, more days than not, for at least 2
years, along with a smattering of depressive symptoms.

Susan is a 15 year of patient in your family practice. Today, she presented to your office complaining of
abdominal pain, not feeling herself and bouts of tearfulness for approximately 1 ½ weeks. She has just
begun grade 9. As she begins to discuss her class schedule and workload she breaks into tears. She has
found the change to high school quite daunting and confusing. Her sleep has been disrupted by worry and
her fears of failure. Her current presentation is a change form the vivacious teen that you have know for
the last several years.

What is the first-line therapy for her condition?

Selective Serotonin Reuptake Inhibitors


Psychotherapy
Tricyclic Antidepressants

Psychoeducation
Serotonin-Norepinephrine Reuptake Inhibitors

Explanation

The correct answer is Choice B.

Psychotherapy remains the treatment of choice for adjustment disorder. Group, family, and individual
therapy can help elucidate the meaning of the stressor. After successful therapy, the patient can
sometimes emerge from an adjustment disorder stronger than in the premorbid condition.

A, C, E. There are no studies asessing the efficacy of pharmacological interventions in individuals with
adjustment disorder, but it may be reasonable to treat specific symptoms for a brief period. Be sure to
thoroughly assess suicidality before prescribing any antidepressant to an adolescent, especially SSRI's.

86
D. Psychoeducation is a specific form of education. It is aimed at helping persons with a mental illness or
anyone with an interest in mental illness, to access the facts about a broad range of mental illnesses in a
clear and concise manner. It is also a way of accessing and learning strategies to deal with mental illness
and its effects.

Psychoeducation is not a treatment. It is designed to be part of an overall treatment plan. For example,
knowledge of one's illness is crucial for individuals and their support network to be able to design their own
relapse prevention plans and strategies.

Research has shown that the more a person is aware of their illness and how it affects their own lives and
that of others, the more control that person has over their illness. This means that, with appropriate
knowledge and techniques, episodes of mental illness occur less often and are usually less severe in
intensity and duration.

A 32-year-old female patient, who is 9 weeks pregnant, presents to the primary care clinic with the
complaint of anxiety. She admits to feeling stressed and nervous for the last 6 months. She experiences
these symptoms several days out of the month at predictable intervals--before deadlines at work or OB
appointments. She has attempted to manage the anxiety with deep breathing techniques and
psychotherapy. At this time, she has been referred to you by her therapist for medication. The therapist
has suggested an SSRI long-term and is requesting your expertise. After doing a complete history and
physical, you agree that medication is warranted.

Which of the following treatment plans would be the best option for this patient:

Fluoxetine daily

Lorazepam as needed

Alprazolam as needed

Sertraline daily

Citalopram daily

Explanation

The correct answer is choice B

It is important to understand how to treat the pregnant patient with psychiatric symptoms and concerns.
While it may seem daunting to treat a pregnant patient with psychopharmacology, untreated mental
illnesses during pregnancy can have serious ramifications for mother and child and must be taken
seriously. In this case, the mother's anxiety is deemed serious enough to treat despite the delicate stage
of organogenesis in the first trimester.

Therefore, your goal is to determine the safest way to medicate this patient in order to adequately manage

87
the symptoms while minimizing adverse effects to both the mother and fetus.

While an SSRI may be a good choice, there are several downsides to that medication in this situation.
SSRIs take several weeks to take effect and must be taken daily. While benzodiazepines may not always
be the best first-line medication to use for anxiety, they begin working at the first dose and can be taken
as needed. Therefore, the fetus can be exposed to less medication.

Additionally, stress and anxiety can increase in pregnancy and, left untreated, can cause low birth weight,
premature delivery and a higher risk of c-sections.

Of the answer choices, A, D, and E are all SSRIs while B and C are benzodiazepines. Based on the above
discussion, you can rule out A, D, and E. Of the two benzodiazepines listed, only lorazepam is considered
safe in pregnancy.

Therefore, this patient would be given lorazepam on an as-needed basis. Considering that her anxiety
tends to occur at predictable intervals, she may even elect to take the medication before an anxiety-
provoking situation to avoid the anxiety altogether.

A 13-year-old girl is brought to the hospital by her parents. Her parents found her writing a letter to her
boyfriend suggesting he would be better off with her dead. She had several bags of stolen items next to
her. During your interview with the girl, she repeatedly tells you that you have no self-control and calls
you a "klepto".

Which of the following is she most likely exhibiting?

Compensation
Projection
Displacement
Rationalization
Abstraction

Explanation

The correct answer is choice B

The girl in the vignette is projecting her own lack of self-control and personal identification as a
kleptomaniac (one who cannot control their impulse to steal things) onto the interviewer. This is a
common defense mechanism seen in many psychiatric patients. Specifically, projection is a defense
mechanism whereby the patient shifts one's own unacceptable thoughts, feelings and impulses onto
someone else, such that the other person is perceived as possessing the unacceptable thoughts.

Compensation is the defense mechanism exhibited when the patient downplays a weakness in favor of
over-emphasizing a perceived strength. For example, emphasizing athletic prowess to cover up for a lack
of academic accomplishments.

Displacement is a defense mechanism that shifts sexual or aggressive impulses to a more acceptable or
less threatening target in order to avoid dealing directly with what is frightening or threatening. This is
seen when an employee who is mad at his or her boss takes lashes out at their spouse.

Rationalization is where a person convinces him or herself that there is/was nothing wrong with a certain
behavior/situation through illogical reasoning. For example, a person may justify stealing from a large

88
business by telling him or herself that the company won't even notice the object is gone.

Abstraction is the ability to utilize abstract reasoning. The lack of abstraction is often seen in patients with
schizophrenia. This can be seen by their literal interpretation of questions. For example, when asked,
"What brought you in to the clinic today?" they will answer "the bus".

A 24 year old female presents to your family practice clinic complaining that she is being threatened with
termination from her job because of habitual tardiness. Through tears she explains that the reason she is
arriving late to work is that she has been unable to leave the house on time because of her need to check
and re-check to ensure that the house is locked tightly. She admits that this issue is worsening and that
on some days it takes her 2 hours to feel able to get in her car to leave for work.

What is her most likely diagnosis?

Panic Disorder
Obsessive Compulsive Disorder
Capgras Syndrome
Trichotillomania
Schizoaffective Disorder

Explanation

The correct answer is Choice B

Persons suffering from obsessive compulsive disorder (OCD) display both the characteristics of obsession
(inappropriate and excessive thinking about an issue) and compulsion (feeling the need to perform an
action to allay their anxieties related to their obsession). In order for them to qualify for this diagnosis the
problem must be causing vocational or social impairment. OCD patients tend to improve with SSRI
treatment and psychotherapy, but complete recovery is rare.

Panic disorder (Choice A) is incorrect because the patient is not experiencing panic attacks (tachycardia,
sense of dyspnea, chest pain/pressure, sense of imminent doom, tingling/numbness of mouth or limbs).

Capgras Syndrome (Choice C) is incorrect because this is the name given to the rare delusional
misidentification syndrome where a person believes that a person in their life has been replaced by another
person. This is most frequently seen in schizophrenia, but can occur in persons suffering from organic brain
concerns or dementia.

Trichotillomania (Choice D) is incorrect because this is the medical term for the obsessive need to pull out
one’s own hair.

Schizoaffective Disorder (Choice E) is incorrect because this psychiatric diagnosis is characterized by


recurring episodes of elevated or depressed mood, or of simultaneously elevated and depressed mood,
that alternates with, or occur together with, distortions in perception. Schizoaffective disorder most
commonly affects cognition and emotion, and many persons with this condition experience auditory
hallucinations, paranoia, bizarre delusions, or disorganized speech and thinking with significant social and
occupational dysfunction.

A 15-year old, previously healthy girl sees you in your clinic and complains of unexplained feelings of
remorse for the last three weeks. She has not wanted to get out of bed and go to school, which she

89
normally enjoys, yet has difficulty actually staying asleep at night.

In addition, she feels fidgety but her overall energy level is decreased. You have to ask her twice to
describe her mood, which she states flatly is "sad". She denies hallucuinations and suicidal ideation. Her
physical exam is unremarkable.

Based on this patient's history, she most likely has which of the following?

A parent that died before age 11.


A family from a lower socioeconomic status.
A 10% chance of contemplating suicide.
Light-colored skin.
None of the above.

Explanation

The correct answer is Choice A.

Circumstances that may increase risk for Major Depressive Disorder (MDD) are being single, living in a
rural area, divorced, losing a parent before age 11 years, experiencing the death of a spouse, and
unemployment. The lifetime risk of MDD is 20-25% in women and 7-12% in men.

A screening tool for Major Depression (Mnemonic: M SIGE CAPS)

 Depressed Mood
 Sleep decreased (Insomnia with 2-4 am awakening)
 Interest decreased in activities (anhedonia)
 Guilt or worthlessness (Not a major criteria)
 Energy decreased
 Concentration difficulties
 Appetite disturbance or weight loss
 Psychomotor retardation/agitation
 Suicidal thoughts

Choice B and D. Socioeconomic status and race have no correlation with MDD (Notice this is different for
the risk of suicide where a white race is a risk factor. Socioeconomic status is not a risk factor for MDD or
suicide).

Choice C. Two-thirds of people with a Major Depressive Episode contemplate suicide and 10-15% commit
suicide.

Sadly, there is a 50% recurrence rate after the first episode of MDD, a 70% recurrence rate after the
second episode, and a 90% recurrence rate after the third episode.

There is only a drop in recurrence to 40% with an adequte single trial with an antidepressant; the majority
of the rest of the patients show some improvement. However, 15-30% of patients do not improve after a
single adequate trial.

The most common reasons for failure are inadequate dosing and inadequate duration. No single
antidepressant medication is universally accepted as more effective. Therapy is guided by side effects,

90
drug interactions, dosing schedule, cost, and history of response (even in family members).

For a single episode of major depression, treat for a minimum of 6 months after resolution of symptoms or
for the length of the previous depressive episode - whichever is longer. Before declaring treatment failure,
enxure maximum titration and minimum of 4-6 weeks of treatment after achieving the maximum dose (full
response guaged at 8-12 weeks).

A 23-year-old white man presents to the emergency department complaining of feeling depressed for the
past week. He also reports difficulty sleeping, lack of appetite, decreased energy, decreased
concentration, a loss of interest in golfing and suicidal thoughts. He is very surprised that he hasn't
wanted to play golf, because this is his favorite hobby.

On further questioning, you learn that on 2-3 occasions over the past 2 years he has also experienced
episodes marked by an elevated mood and increased energy that last for several days. During these
episodes, the patient felt great despite sleeping 2-3 hours per night. He was able to complete his
schoolwork and even wrote a novel in 2 nights. The patient recalled that during these episodes, he had
"nonstop ideas circling through my head." He said that he drank heavily, engaged in sexual intercourse
indiscriminately, and gambled excessively. Although he recognized that these behaviors were odd for him,
he insisted that he was able to function well during these episodes.

When asked about substance use, he acknowledged heavy drinking twice a month. He also reported
occasional marijuana and cocaine use. He clarified that his marijuana and cocaine use was an attempt to
"self-medicate" during period of depression and anxiety.

What are the best initial treatment for this patient?

Lithium plus an atypical antipsychitoc, OR valproate plus an atypical


antipsychotic
First-line: SNRI's and SSRI's. Second-line: TCA's and MAOI's.
Inpatient detoxification from marijuana and cocaine use
Monotherapy with lithium, valproate, or an atypical antipsychotic
Carbamazepine or oxcarbazepine

Explanation

The correct answer is Choice D.

Bipolar disorder type II, currently depressed, is the full diagnosis because the most recent episode is used
to clarify an initial diagnosis. A history of depressive episodes in addition to hypomanic episodes (he was
still able to function) warrant a diagnosis of bipolar disorder type II. Indications of hypomania included
inflated self-esteem, decreased need for sleep, increased goal-directed activity, excessive involvement in
goal-directed activities, racing thoughts, and pressured speech.

Pharmacotherapy is guided by the severity of the mania. In acute mania, double therapy is indicated with
lithium plus an atypical antipsychitoc, OR valproate plus an atypical antipsychotic. In less severe mania,
monotherapy with lithium, valproate, or an atypical antipsychotic will usually suffice.

Choice A. The patient's symptoms never escalated to the point of not functioning, psychosis, or bizarre
delusions. Therefore, dual therapy is not indicated.

91
Choice B. This approach is for anxiety disorders in general, not bipolar disorder.

Choice C. The patient's substance use does not seem to be temporally related to his onset of hypomania.
So, while abstinence will help him, this is not the best INITIAL treatment for his mania.

Choice E. These are second-line drugs in the treatment of mania.

A 27-year-old woman seeks evaluation for her "depression" in an outpatient clinic. She reports episodic
feelings of sadness since adolescence. Occasionally she feels good, but these periods seldom last more
than 2 weeks. She is able to work but thinks she is not doing as well as she should. In describing her
problems she seems to focus more on repeated disappointments in her life and her low opinion of herself
than on discrete depressive symptoms.

In your differential diagnosis at this point, what is the most likely diagnosis?

Major depression with melancholia


Adjustment disorder with depressed mood
Cyclothymia
Bipolar disorder, type II
Dysthymia

Explanation

The correct answer is choice E.

Even if you weren't sure of the diagnosis after reading the question stem, a basic knowledge of DSM-IV-TR
criteria allows for a simple process of elimination of answer choices.

Dysthymia requiers at least 2 years of constant or constantly recurring depressed mood. Intervening
periods of normal mood rarely last longer than a few weeks, and there are NO episodes of hypomania
(analagous to Choice D). None or very few of the individual episodes of dpression within such a 2-year
period should be sufficiently severe or long-lasting to meet the criteria for recurrehnt mild depressive
disorder.

Finally, during a least some of the periods of depression at least three of the following should be present:

1. reduced energy or activity


2. insomnia
3. loss of self-confidence or feelings of inadequacy
4. difficulty in concentrating
5. frequent tearfulness
6. loss of interest in or enjoyment of sex and other pleasurable activities
7. felling of hopelessness or despair
8. a perceived inability to cope with the routine responsibility of everyday life
9. pessimism about the future or brooding over the past
10. social withdrawal
11. reduced talkativeness

A. As you know, the diagnosis of a Major Depressive Episode does have a 2 week stipulation in it, and this

92
case does reveal that some of her episodes last longer than 2 weeks. This overlap between diagnoses
highlights the importance of you coming up with your own diagnosis before looking at the answer choices.
The overal flavor for this case history is a very long course, from adolesence to age 27. Another key
differentiating feature is that this patient is still able to function; in this case she can still work. During
Major Depressive Episodes, patients become unable to function in their usual capacity, at least to some
degree and this question stem clearly states that she can still work.

B. A requirement for the diagnosis of Adjustment Disorder is an identifiable stressor, which is not present
in this question stem. As a review, Adjustment Disorder is the development of emotional or behavioral
symptoms occuring with 3 months of the onset of an identifiable stressor (such as internship). These
symptoms are clinically significant evidenced by either marked distress that is in excess of what would be
expected from exposure to the stressor, or causing significant impairment in social or occupational
(academic) functioning. This does not include bereavement. Once the stressor has terminated, the
symptoms do not persist for more than an additional 6 months.

C. As the name implys, Cyclothymia has an alternating mood that fluctuates between depression and
hypomania. This question step did not indicate any manic symptoms, which should be another thing in
your diagnostic radar as you read questions about patients with mood disorders. Cyclothymia lasts at least
2 years and the symptoms don't meet the criteria for Major Depression or Bipolar Disorder. During the 2-
year period (1 year for children and adolescents), the patient has not been without symptoms for more
than 2 months.

D. Bipolar II differs from Bipolar I in the intensity of the mania. In Bipolar II, hypomania is needed to make
the diagnosis. The other criteria are the same. This patient had no history of mania at all, so this choice is
an easy one to rule out.

A 25 year-old Asian female who suffers from depression and migraine headaches is brought to to your
urgent care by her roommate who reports that the patient was home with a “migraine” yesterday and
then earlier today the patient became “confused, was acting strangely, and just wasn’t herself”. Her vital
signs are as follows: O2 sat: 98%, Pulse 120, Temperature 102.6, Blood Pressure 118/78 and Respiratory
Rate 20. On physical examination she is pale, diaphoretic, confused, mildly agitated and shivering. Her
lungs are clear and she demonstrates no cardiac exam irregularities other than tachycardia, but when you
begin to palpate her abdomen she jumps off of the table and mutters “bathroom”. You and the nurse then
help the patient to the bathroom where the patient proceeds to have an episode of explosive diarrhea.

After the patient is returned to the examination room you perform laboratory studies that reveal a normal
CBC, electrolytes, and urine. Just as you are preparing to ask the other physician who is working with you
to assist you in your diagnostic quandary, you ask the nurse to tell you again what medications the
patient is taking. The nurse points you to the sack of medications on the counter. You take out the 3
bottles and see that she takes paroxetine 20 mg daily, sumatriptan 100 mg prn, and there is a bottle of
tramadol that was filled yesterday.

What is her likely diagnosis?

Malignant Hyperthermia
Heat Stroke
Pyelonephritis
Serotonin Syndrome
Neuroleptic Malignant Syndrome

Explanation

93
The correct answer is Choice D

Serotonin Syndrome is a condition of usually abrupt onset resulting from excessive serotonin intake. The
markedly elevated serotonin levels result in central nervous system toxicity that lead to the kind of
impressive generalized presentation see in this patient. Signs and symptoms of the condition include
agitation, confusion, sweating, tachycardia, muscle twitching and/or loss of muscle coordination, papillary
dilatation, diarrhea, shivering, and piloerection. Severe manifestations may include seizures, coma and
death.

Both psychiatric and non-psychiatric medications can contribute to the development of serotonin
syndrome, including SSRIs, TCAs, MAOIs, SNRIs, buproprion, pain medications (i.e., tramadol, fentanyl,
etc), Kytril, Zofran, over the counter compounds containing serotonin (St. John’s Wart), and some over the
counter cold preparations. The likely medication addition that pushed this patient into the episode was the
addition of tramadol.

Malignant Hyperthermia (Choice A) is not correct. It is a rare acute neurological disorder resulting from
general anesthesia with significant attendant morbidity and mortality.

Heat Exhaustion (Choice B) is not correct. Although some of her symptoms are seen in heat exhaustion
(marked diaphoresis, dizziness, fatigue/lethargy, tachycardia, muscle cramps, nausea, headache,
confusion) she had not had any recent significant heat exposure.

Pyelonephritis (Choice C) cannot be correct because her urinalysis is normal.

Table 1: Comparison of Serotonin Syndrome and NMS

Table 2: Drugs that increase serotonin

94
You are in an emergency medicine preceptorship as a medical student and your resident requests you run
the show while he goes to lunch. 10 minutes later an odd-appearing disheveled male who appears to be in
his late 40s staggers into the room and collapses on the floor. Your team quickly moves him to bed and
performs vitals which reveal pulse 144, respirations 28 and labored, blood pressure 100/70, and a
temperature of 104.5 F. His oxygen saturation is 97%. You get the nurses started placing lines for
supportive care and obtain blood for standard laboratory testing. His physical exam is remarkable for
profound diaphoresis, mental status that ranges from confused to obtunded, tremor of his extremities,
and his body seems to be almost rigid. A myriad of diagnoses rushes through your mind and then, in a
moment of true epiphany, you start digging through the patient’s clothing and find a pill bottle that
provides your diagnosis. You STAT page your resident and request he respond to help you with a patient
experiencing what you believe to be an extremely life-threatening situation.

What medication did you find that provided the clue to this patient’s likely diagnosis?

1. metoclopramide
2. amitriptyline
3. hydroxyzine
4. risperdone

5. haloperidol

1 and 3 only
1, 2 and 3 only

1 and 2 only
2, 3, and 4 only
1, 2, 3, 4 and 5

95
Explanation

The correct answer is Choice E

This patient presents with a fairly classic case of neuroleptic malignant syndrome, a rare and potentially
fatal condition characterized by severe hyperthermia, muscular rigidity and autonomic dysfunction that is
usually a consequence of chronic antipsychotic (neuroleptic) administration. High level of suspicion in
combination with thorough reveal of the patient’s list of current medications is required in order to make
this diagnosis. Treatment is rapid institution of supportive measures in combination with dantrolene or
bromocriptine. A helpful diagnostic pneumonic for neuroleptic malignant syndrome is FEVER.

F – Fever ; E – Encephalopathy ; V – Vitals unstable;

E – Elevated enzymes (elevated CPK) R – Rigidity of muscles

Metoclopramide and promethazine (Choice A) can be causative of neuroleptic malignant syndrome, but so
can the other 3 compounds listed as possible answers to this question, so this question is incorrect In
addition, co-administration of more than 1 of these products increases the likelihood of development of the
syndrome.

Metoclopramide, amantadine and promethazine (Choice B) can be causative of neuroleptic malignant


syndrome; but, again, so can the other 2 compounds listed as possible answers to this question.
Amantadine/s mechanism of action is via its role in the reduction of dopamine levels associated with anti-
Parkinsonian medications.

Metoclopramide and amantadine (Choice C) is also incorrect because the other 3 selections can cause
neuroleptic malignant syndrome.

Amantadine, promethazine and risperdone (Choice D) is also incorrect because even though all 3 of these
medications can be causative of neuroleptic malignant syndrome, so can the other 2 options listed.

Table 1: Drugs That Can Cause Neuroleptic Malignant Syndrome

96
Table 2: NMS vs Seratonin Syndrome

97
A 52 year-old Asian female is followed by her internist for hypertension and hyperlipidemia. Her most
recent lipid levels were normal and her blood pressure has been borderline elevated at her last two visits.
During today’s exam the patient complained of fatigue and insomnia that has been worsening over the
past four months. She was diagnosed with Major Depression.

Which of the following medications should this patient’s physician avoid prescribing for her depressive
concerns?

escitalopram
paroxetine
venlafaxine
fluoxetine
Any of the above would be fine

Explanation

The correct answer is Choice C

Because of its adrenergic properties, induction or exacerbation of hypertension is a possible side effect
concern with venlafaxine. A member of the combination Selective Serotonin Reuptake Inhibitor-
Norepinephrine class of antidepressants, venlafaxine is best avoided in patients being treated for
hypertension.

Escitalopram (Choice A) has a relatively benign side effect profile. It has no known blood pressure
concerns.

Paroxetine (Choice B) has side effect concerns that include weight gain, sexual dysfunction, and drug-drug
interactions. Blood pressure concerns, however, are not amongst its list of regularly noted side effects.

Fluoxetine (Choice D), like most of the SSRIs, demonstrates side effect concerns including possible
sedation, anxiety and sexual dysfunction that can range from mild to marked, but is not known to
demonstrate significant effects on blood pressure.

Any of the above would be fine (Choice E) is incorrect because of the concerns stated above related to
venlafaxine and hypertension.

Table 1: Adverse effects of antidepressant drugs, based on mechanism of action

98
A 12-year-old boy with newly-diagnosed ADHD presents to the primary care clinic with his parents. The
boy's chief complaint is insomnia. He describes restless sleep and not feeling refreshed upon wakening.
Additionally, he describes a "throbbing" headache during the morning hours of his school day. Upon
further questioning, you discover that he was recently put on 10 mg twice daily of methylphenidate for his
ADHD and takes it in the morning before school and at lunchtime.

99
Which of the following options is the next best step in this boy's treatment plan?

Switch from methylphenidate to atomoxetine


Educate on sleep hygiene and suggest acetaminophen for headache management
Change dosage of methylphenidate to 10 mg once daily
Add melatonin 3 mg at bedtime as a sleep aid and ibuprofen 600 mg once daily for headache
management
Decrease dosage of methylphenidate to 5 mg twice daily

Explanation

The correct answer is choice E

This question is testing your knowledge of the management and recognition of stimulant
adverse effects. The headache and insomnia are both typical adverse effects of stimulants, particularly the
short-acting ones, such as the one used (you can tell this because of the twice-daily dosing only a few
hours apart). The longer-acting stimulants can last up to 12 hours, and are not usually given twice daily.

The insomnia may decrease with usage and the headaches may actually worsen. Therefore, the
best action to take is to decrease the dosage and reassess the ADHD symptoms. If the side effects then
decrease but the ADHD symptoms return, the appropriate next step would then be to switch to
atomoxetine (answer choice 2). In general, it is usually a good idea to decrease the dosage of a new
medication before switching altogether when experiencing minor side effects. A slower titration up to the
therapeutic dose will often decrease or eliminate adverse effects.

While educating on sleep hygiene and suggesting acetaminophen for headache management are good
options, they are not appropriate long-term solutions to the medication's side effects. Long-term use of
acetaminophen can cause ulcers and prolonged bleeding times.

Changing the dosage of methylphenidate to 10 mg once daily would not be an adequate solution. The
medication would not last the entire school day with this regimen. Therefore, the ADHD symptoms would
return before he was done with schoolwork.

Adding melatonin 3 mg at bedtime as a sleep aid and ibuprofen 600 mg once daily for the headaches is
similar to the acetaminophen suggestion above. While melatonin is an excellent sleep aid that will not
interract with most psychopharmacological agents, taking ibuprofen long-term for headaches is not a
good treatment plan.

A young mother is extremely focused on the health of her 16 month old child. She cleans constantly in an
effort to keep her house immaculate for fear that dirt will harm her baby, checks the lock on the door at
least 10 times before retiring to bed, and gets up 8-10 times/night to check that her child is still
breathing. She realizes that her fears are irrational but persists with these behaviors.All this effort and
anxiety is wearing her out, but she feels she must persist with the activities in order to protect her child
from harm.

The most likely diagnosis is which of the following:

100
paranoid delusions not otherwise specified
post partum depression
obsessive compulsive disorder
generalized anxiety disorder
paranoid personality disorder

Explanation

The correct answer is Choice C

In obsessive-compulsive disorder (OCD) sufferers experience distressing intrusive obsessive thoughts that
may or may not be followed by the person’s feeling the need to perform repetitive acts in order to reduce
their anxiety about the obsession: these actions are called compulsions. The diagnosis of OCD is made by
cataloguing the patient’s symptoms and then comparing them to the diagnostic criteria listed below:

The patient must display either obsessions or compulsions with obsessions defined by (a), (b), (c), and (d)
and compulsions defined by (e) and (f):

 (a) recurrent and persistent thoughts, impulses, or images that are experienced, at some time
during the disturbance, as intrusive and inappropriate and that cause marked anxiety or distress
 (b) the thoughts, impulses, or images are not simply excessive worries about real-life problems
 (c) the person attempts to ignore or suppress such thoughts, impulses, or images, or to neutralize
them with some other thought or action
 (d) the person recognizes that the obsessional thoughts, impulses, or images are a product of his
or her own mind (not imposed from without as in thought insertion)
 (e) repetitive behaviors (e.g., hand washing, ordering, checking) or mental acts (e.g., praying,
counting, repeating words silently) that the person feels driven to perform in response to an
obsession, or according to rules that must be applied rigidly
 (f) the behaviors or mental acts are aimed at preventing or reducing distress or preventing some
dreaded event or situation; however, these behaviors or mental acts either are not connected in a
realistic way with what they are designed to neutralize or are clearly excessive

Paranoid delusions not otherwise specified (Choice A) cannot be correct because the patient is not
delusional. The definition of a delusion is a fixed false belief from which one cannot be shaken. The patient
in our example has insight into her obsessive compulsive behaviors, whereas a delusional person (ex: a
young man who believes that he is the leader of a space colony located on Planet Neptar) does not.

Postpartum depression (Choice B) is incorrect because the patient is demonstrating anxiety issues, not
depressive. Symptoms of depression include low mood, sadness, sleep disturbances, difficulty
concentrating, sense of inertia, anhedonia, tearfulness, crying spells and possible suicidality.

Generalized anxiety disorder (Choice D) is not correct because, while the patient IS demonstrating anxiety,
obsessive thinking and compulsive behaviors are not amongst the symptoms of generalized anxiety
disorder. Generalized anxiety disorder is characterized by a pattern of frequent, constant worry and
anxiety over many different activities and events, difficulty concentrating, difficulty controlling worry,
disproportionate anxiety and worry, nonspecific physical symptoms that include sweating, palpitations,
dyspnea, abdominal discomfort, fatigue, irritability, muscle tension, a sense of inner restlessness and sleep
disturbance.

Paranoid personality disorder (Choice E) is incorrect because this condition is a chronic psychiatric illness
similar to paranoid schizophrenia that usually results in the person becoming low functioning and socially
isolated. These people are generally suspicious and distrustful. Additionally, they tend to believe that the
people around them have hidden motives and that they are going to use and exploit the person suffering
from the condition. Such flawed thinking can lead to poor self image, detachment, hostility and even
violence.

101
102

Das könnte Ihnen auch gefallen